NCLEX RN - Acid-Base Imbalance (+Practice), Acid/Base NCLEX Questions, IBD Summer Test 5, ABG Test Questions, Medical-Surgical: Gastrointestinal, LIVER (lippencott questions), ATI Gastrointestinal, Peptic Ulcer Disease, med surg 6, Med Surg Ch 58 Coo...

Pataasin ang iyong marka sa homework at exams ngayon gamit ang Quizwiz!

A patient has just been brought to the emergency department by emergency medical services after a motor vehicle accident. What is the first thing the nurse should do?

Mental status examination and vital signs

a nurse is assessing a client who has upper gastrointestinal blooding. which of the following findings should the nurse except?

hypotension

The nurse monitors for which clinical manifestations in the patient diagnosed with a duodenal ulcer? (Select all that apply.)

- Intermittent abdominal pain, which is relieved after eating and taking antacids but becomes worse at night - Right upper quadrant tenderness and is positive for occult blood in stool - Complaints of heartburn or regurgitation and vomiting

Which interventions should the nurse provide to a patient with a T-tube? (Select all that apply.)

- Routinely monitor drainage output - Provide T-tube maintenance teaching to the patient - monitor stool for color and consistency

2 (clients may devlop folic acid deficiency. Green leafy veges are good for that)

A client has been placed on long term sulfasalazine therapy for treatment of ulcerative colitis. The nurse should encourage the client to eat which foods to help avoid the nutrient deficiencies that may develop as a result of this medication? 1. citrus fruits 2. green leafy vegetables 3. eggs 4. milk products

The nurse is caring for a patient with colorectal cancer who just had a total colectomy with placement of a permanent ileostomy. Which nursing diagnosis is a priority for the immediate postoperative period?

Acute pain

Metabolic Acidosis

A total concentration of buffer base that is lower than normal, with a relative ↑ in the hydrogen ion concentration, resulting from loss of too much base &/or retention of too much acid pH ↓ PCO2 normal HCO3 ↓ >Respiratory compensates by hyperventilating to blow off CO2

4 (Change in baseline. May need ECG, This one first)

After completing assessment rounds, which client should the nurse discuss with the HCP first? 1. A client with cirrhosis who is depressed and has refused to eat for the past 2 days 2. A client with stable VS that has been receiving IV ciprofloxacin following a cholecystectomy for 1 day and has developed a rash on the chest and arms 3. A client with pancreatitis whose family requests to speak with the HCP regarding the treatment plan 4 A client with hepatitis whose pulse was 84 bpm and regular and is now 118 and irregular

A nurse is caring for a client who has acute pancreatitis. Which of the following serum laboratory values should the nurse anticipate returning to the expected reference range within 72 hr after treatment begins?

Amylase

A nurse is assessing a client who is in the early stages of hepatitis A. Which of the following manifestations should the nurse expect?

Anorexia

A nurse is assessing a client who is experiencing perforation of a peptic ulcer. Which of the following manifestations should the nurse expect?

Boardlike abdomen

The patient with which condition is at the highest risk for fistula formations?

Crohn's disease

A community health nurse is planning an educational program about hepatitis A. When preparing the materials, the nurse should identify that which of the following groups is most at risk for developing hepatitis A?

Children

A nurse is caring for a client who is receiving total parenteral nutrition (TPN) therapy and has just returned to the room following physical therapy. The nurse notes that the infusion pump for the client's TPN is turned off. After restarting the infusion pump, the nurse should monitor the client for which of the following findings?

Diaphoresis

The nurse admitting a patient with acute pancreatitis correlates this disease process with which etiology?

Digestion of the pancreas by enzymes

A patient with liver disease is being evaluated for varices. The nurse prepares the patient for which procedure?

EGD

The nurse correlates which laboratory data to decreased liver function secondary to liver cancer or cirrhosis?

Elevated serum ammonia level

A nurse is caring for a client who has a history of cirrhosis and is admitted with manifestations of hepatic encephalopathy. The nurse should anticipate a prescription for which of the following laboratory tests to determine the possibility of recent excessive alcohol use?

Gamma-glutamyl transferase (GGT)

Respiratory Alkalosis

Hyperventilation A deficit of carbonic acid & a ↓ in hydrogen ion concentration that results from the accumulation of base or from a loss of acid without a comparable loss of base in the body fluids pH ↑ PCO2 ↓ HCO3 normal >Kidneys compensate by secreting bicarbonate & retaining hydrogen

Respiratory Acidosis

Hypoventilation The total concentration of buffer base is lower than normal, with a relative ↑ in hydrogen ion concentration; thus a greater number of hydrogen ions is circulating in the blood than can be absorbed by the buffer system pH ↓ PCO2 ↑ HCO3 normal >Kidneys compensate by retaining bicarbonate & secreting hydrogen

Which type of shock is exhibited in the patient with pancreatitis?

Hypovolemic shock

A nurse is caring for a client who has fulminant hepatic failure. Which of the following procedures should the nurse anticipate for this client?

Liver transplant

a nurse is reviewing the prescriptions for a client who has Campylobacter enteritis. which of the following prescriptions should the nurse clarify with the provider?

Milk of magnesia the nurse should clarify a prescription for milk of magnesia with the provider. this medication increases gastrointestinal motility, which can increase the client's risk for an electrolyte imbalance.

The nurse correlates which clinical manifestation to cholecystitis?

RUQ pain

Metabolic Alkalosis - Risk factors/Causes

Results from a dysfunction of metabolism that causes an ↑ amount of available base solution in the blood or a ↓ in available acids in the blood >Diuretics >Ingestion of antacids >GI suction >Excessive vomiting >Hypokalemia >Hypoaldosteronism >TPN >Blood transfusion

Respiratory Alkalosis - Risk factors/Causes

Results from conditions that cause overstimulation of the respiratory system >Altitude sickness >Anxiety >Asphyxiation >Asthma >Brain injury >Hyperventilation >Hypoxemia >Sepsis

A nurse is caring for a client who is 4 hr postoperative following a laparoscopic cholecystectomy. Which of the following findings should the nurse expect?

Right shoulder pain

A nurse is assessing a client who was admitted with a bowel obstruction. The client reports severe abdominal pain. Which of the following findings should indicate to the nurse that a possible bowel perforation has occurred?

Rigid abdomen

1 (hepatoxicity. Death can occur)

The client dx with bipolar disorder has been taking valproic acid (Depakote) an anticonvulsant for 4 months. Which assess,emt data would warrant the medication being discontinued? 1. The clients eyes are yellow 2 The client has mood swings 3. The clients BP is 164/94 4. The clients serum level is 75 mcg/mL

A nurse is caring for a client who is dehydrated and is receiving continuous tube feeding through a pump at 75 mL/hr. When the nurse assesses the client at 0800, which of the following findings requires intervention by the nurse?

The head of the bed is elevated 20 degrees

1 3 2 4 (The nurse should first assess the client with cirrhosis to ensure the clients safety and assess the client for onset of hepatic encephalopathy. The nurse should then assess the client with acute pancreatitis who is requesting pain med, and deliver the needed med. The nurse should then assess the client who underwent a cholecystectomy and is 1 day post op to make sure the T tube is draining and the client is performing post op breathing exercises. The nurse can speak last with the client with Hep B who has questions about discharge instructions because this is not urgent)

The nurses assignment consists of 4 clients. From highest to lowest priority, in which order should the nurse assess the clients after receiving the morning report? 1. The client with cirrhosis who became confused and disoriented last night 2. The client who is 1 day post op following a cholecystectomy and has a T tube inserted 3. The client with acute pancreatitis who is requesting pain medication 4. The client with Hep B who has questions about discharge instructions

A nurse is caring for a client who is scheduled to undergo an esophagogastroduodenoscopy (EGD). The nurse should identify that this procedure is used to do which of the following?

To detect an ulceration in the stomach

A nurse in the emergency department is caring for a client who has bleeding esophageal varies. The nurse should anticipate a prescription for which of the following medications?

Vasopressin

A nurse is caring for a client who has celiac disease. Which of the following foods should the nurse remove from the client's meal tray?

Wheat toast

a,b,c,e (Complications associated with cirrhosis include esophageal​ varices, splenomegaly,​ ascites, and hepatic encephalopathy. Hypertension is not a complication associated with cirrhosis.)

Which complications are associated with​ cirrhosis? ​(Select all that​ apply.) a Hepatic encephalopathy b Splenomegaly c Esophageal varices d Hypertension e Ascites

The nurse correlates which clinical manifestation to the pathophysiology of a gastric ulcer?

acute gastric pain after eating

a nurse is reviewing the laboratory results of a client who has hepatic cirrhosis. which of the following laboratory findings should the nurse report to the provider?

ammonia 180 mcg/dl the nurse should report an increased serum ammonia level because it can indicate portal-systemic encephalopathy

The nurse correlates which clinical manifestation to type I hiatal hernia?

anorexia

When a physical assessment of gastrointestinal function is completed, which procedure follows inspection?

auscultation

a nurse is providing discharge teaching for an older adult client who has mild diverticulitis. which of the following client statements indicates an understanding of the teaching?

i should eat foods that are lower in fiber

a nurse if providing discharge teaching for a client who has gastritis and a new prescription for famotidine. which of the following client statements indicates the teaching was effective?

i should make sure the water I drink is filtered

a nurse is providing discharge teaching for a client who has GERD. which of the following client statements indicates the teaching was effective?

i will decrease the amount of carbonated beverages i drink

a nurse is assessing a client who has acute hepatitis B. which of the following findings should the nurse except?

joint pain

a nurse is developing a plan of care for a client who has cirrhosis and ascites. which of the following interventions should the nurse include?

measure abdominal girth daily

The nurse correlates poor dentition in patients to which digestive disorder?

mechanical digestion of protein

Fully compensated

pH normal PCO2 & HCO3 are abnormal

Partially compensated

pH, PCO2 or HCO3 are all abnormal

Uncompensated

pH, PCO2 or HCO3 is abnormal

pH: 7.41, PCO2: 26, HCO3: 18

pH: N, PCO2: ↓, HCO3: ↓ Fully compensated Respiratory Alkalosis

pH: 7.50, PCO2: 40, HCO3: 32

pH: ↑, PCO2: N, HCO3: ↑ Uncompensated Metabolic Alkalosis

pH: 7.86, PCO2: 30, HCO3: 20

pH: ↑, PCO2: ↓, HCO3: ↓ Partially compensated Respiratory Alkalosis

In preparing a patient for esophageal manometry, which content does the nurse include in preprocedure teaching?

this test measures the pressure and action of the esophagus

a nurse is providing discharge teaching for a client who has a new colostomy and is concerned about flatus and order. which of the following foods should the nurse recommend?

yogurt

a nurse is caring for a client who has hepatic encephalopathy. the client asks the nurse if she can have a larger portion of beef for dinner. which of the following responses by the nurse is appropriate?

you should limit your animal protein intake. can i get you a veggie burger instead?

A nurse is preparing a community education program about hepatitis B. Which of the following statements should the nurse include in the teaching?

"A hepatitis B immunization is given to infants and children."

Which patient statement indicates effective teaching related to acute gastritis?

"I need to avoid using aspirin or nonsteroidal medications for routine pain relief."

The nurse is caring for a postoperative patient who has had surgery for a ventral hernia repair. The nurse is reinforcing postoperative care teaching and asks the patient to verbalize what was taught to him. Which statement requires the nurse to clarify the patient's perception of the teaching?

"I should turn, cough, and deep breathe every 2 hours."

Which statement by a patient diagnosed with liver trauma indicates understanding of the prescribed plan of care?

"I will need a blood transfusion."

Which statement by the patient scheduled for an EGD indicates the need for further teaching?

"It is important that I eat extra fiber after the test."

The nurse incorporates which information into the teaching plan for a patient diagnosed with a duodenal ulcer?

"Smoking cigarettes can make the PUD worse."

A nurse is teaching a client who has Barrett's esophagus and is scheduled to undergo an esophagogastroduodenoscopy (EGD). Which of the following statements should the nurse include in the teaching?

"This procedure can determine how well the lower part of your esophagus works."

Which statement by the patient undergoing external beam radiation indicates the need for further teaching?

"my grandchildren will not be able to visit because I will be radioactive"

The nurse caring for a client with an ileostomy understands that the client is *most* at risk for developing which acid-base disorder? 1. Metabolic acidosis 2. Metabolic alkalosis 3. Respiratory acidosis 4. Respiratory alkalosis Reference: Saunders Comprehensive Review for the NCLEX-RN examination (6th ed.)

*1. Metabolic acidosis* 2. Metabolic alkalosis 3. Respiratory acidosis 4. Respiratory alkalosis Rationale: Metabolic acidosis is defined as a total concentration of buffer base that is lower than normal, with a relative ↑ in the hydrogen ion concentration. This results from loss of buffer bases or retention of too many acids without sufficient bases, & occurs in conditions such as kidney disease; DKA; high fat diet; insufficient metabolism of carbohydrates; malnutrition; ingestion of toxins, such as acetylsalicylic acid (aspirin); malnutrition; or severe diarrhea. Intestinal secretions are high in bicarbonate & may be lost through enteric drainage tubes, an ileostomy, or diarrhea.

The nurse reveals the blood gas results of a client with atelectasis. The nurse analyzes the results & determines that the client is experiencing respiratory acidosis. Which result validates the nurse's findings? 1. pH: 7.25, PCO2: 50 mm Hg 2. pH: 7.35, PCO2: 40 mm Hg 3. pH: 7.50, PCO2: 52 mm Hg 4. pH: 7.52, PCO2: 28 mm Hg Reference: Saunders Comprehensive Review for the NCLEX-RN examination (6th ed.)

*1. pH: 7.25, PCO2: 50 mm Hg* 2. pH: 7.35, PCO2: 40 mm Hg 3. pH: 7.50, PCO2: 52 mm Hg 4. pH: 7.52, PCO2: 28 mm Hg Rationale: Atelectasis is a condition characterized by the collapse of the alveoli, preventing therespiratory exchange of oxygen & carbon dioxide in a part of the lungs. The normal pH is 7.35 to 7.45. The normal PCO2 is 35 to 45 mm Hg. In respiratory acidosis, the pH is ↓ & the PCO2 is elevated. Option 2 identifies normal values. Option 3 identifies an alkalotic condition, & option 4 identifies respiratory alkalosis.

A nurse is caring for a client who was in a motor-vehicle accident. He is reporting chest pain & difficulty breathing, A chest x-ray reveals the client has a pneumothorax, & arterial blood gases are obtained. Which of the following findings should the nurse expect? A.) pH: 7.06 PaO2: 86 mm Hg PaCO2: 52 mm Hg HCO3-: 24 mEq/L B.) pH: 7.42 PaO2: 38 mm Hg PaCO2: 38 mm Hg HCO3-: 23 mEq/L C.) pH: 6.98 PaO2: 100 mm Hg, PaCO2: 30 mm Hg HCO3-: 18 mEq/L D.) pH: 7.58 PaO2: 96 mm Hg, PaCO2: 38 mm Hg HCO3-: 29 mEq/L

*A.)* *pH: 7.06* *PaO2: 86 mm Hg* *PaCO2: 52 mm Hg* *HCO3-: 24 mEq/L* B.) pH: 7.42 PaO2: 38 mm Hg PaCO2: 38 mm Hg HCO3-: 23 mEq/L C.) pH: 6.98 PaO2: 100 mm Hg, PaCO2: 30 mm Hg HCO3-: 18 mEq/L D.) pH: 7.58 PaO2: 96 mm Hg, PaCO2: 38 mm Hg HCO3-: 29 mEq/L Rationale: Pneumothorax can cause alveolar hyperventilation & ↑ carbon dioxide levels, resulting in a state of *respiratory acidosis*

A client is scheduled for blood to be drawn from the radial artery for an ABG determination. A nurse assists with performing Allen's test before drawing the blood to determine the adequacy of the: 1. Ulnar circulation 2. Carotid circulation 3. Femoral circulation 4. Brachial circulation

1.

A nurse is caring for a client with severe diarrhea. The nurse monitors the client closely, understanding that this client is at risk for developing which acid-base disorder? 1. Metabolic acidosis 2. Metabolic alkalosis 3. Respiratory acidosis 4. Respiratory alkalosis

1.

1. George Kent is a 54 year old widower with a history of chronic obstructive pulmonary disease and was rushed to the emergency department with increasing shortness of breath, pyrexia, and a productive cough with yellow-green sputum. He has difficulty in communicating because of his inability to complete a sentence. One of his sons, Jacob, says he has been unwell for three days. Upon examination, crackles and wheezes can be heard in the lower lobes; he has a tachycardia and a bounding pulse. Measurement of arterial blood gas shows pH 7.3, PaCÓ 68 mm Hg, HCǑ 28 mmol/L, and PaÓ 60 mm Hg. How would you interpret this? A. Respiratory Acidosis, Uncompensated B. Respiratory Acidosis, Partially Compensated C. Metabolic Alkalosis, Uncompensated D. Metabolic Acidosis, Partially Compensated

1. Answer: B. Respiratory Acidosis, Partially Compensated The patient has respiratory acidosis (raised carbon dioxide) resulting from an acute exacerbation of chronic obstructive pulmonary disease, with partial compensation.

The nurse is caring for a client with a nasogastric tube that is attached to low suction. The nurse monitors the client, knowing that the client is at risk for which acid-base disorder? 1. Metabolic acidosis 2. Metabolic alkalosis 3. Respiratory acidosis 4. Respiratory alkalosis Reference: Saunders Comprehensive Review for the NCLEX-RN examination (6th ed.)

1. Metabolic acidosis *2. Metabolic alkalosis* 3. Respiratory acidosis 4. Respiratory alkalosis Rationale: Metabolic alkalosis is defined as a deficit or loss of hydrogen ions or acids or an excess of base (bicarbonate) that results from the accumulation of base or from a loss of acid without a comparable loss of base in the body fluids. This occurs in conditions resulting in hypovolemia, the loss of gastric fluid,excessive bicarbonate intake, the massive transfusion of whole blood, & hyperaldosteronism. Loss of gastric fluid via nasogastric suction or vomiting causes metabolic alkalosis as a result of the loss of hydrochloric acid.

The nurse plans care for a client with chronic obstructive pulmonary disease (COPD), understanding that the client is most likely to experience what type of acid-base imbalance? 1. Metabolic acidosis 2. Metabolic alkalosis 3. Respiratory acidosis 4 Respiratory alkalosis Reference: Saunders Comprehensive Review for the NCLEX-RN examination (6th ed.)

1. Metabolic acidosis 2. Metabolic alkalosis *3. Respiratory acidosis* 4. Respiratory alkalosis Rationale: Respiratory acidosis is most often caused by hypoventilation in a client with COPD. Other acid-base disturbances can occur in a client with COPD during exacerbation of the disease, but the most likely imbalance is respiratory acidosis. COPD is a respiratory condition, not a metabolic one. Respiratory alkalosis is associated with hyperventilation.

The nurse reviews the arterial blood gas results of a client & notes the following: pH: 7.45 PCO2: 30 mm Hg HCO3-: 20 mEq/L The nurse analyzes these results as indicating which condition? 1. Metabolic acidosis, compensated 2. Respiratory alkalosis, compensated 3. Metabolic alkalosis, uncompensated 4. Respiratory acidosis, uncompensated Reference: Saunders Comprehensive Review for the NCLEX-RN examination (6th ed.)

1. Metabolic acidosis, compensated *2. Respiratory alkalosis, compensated* 3. Metabolic alkalosis, uncompensated 4. Respiratory acidosis, uncompensated Rationale: The normal pH is 7.35 to 7.45. In a respiratory condition, an opposite effect will be seen between the pH & the PCO2. In this situation, the pH is at the high end of the normal value & the PCO2 is low. In an alkalotic condition, the pH is elevated. Therefore the values identified in the question indicate a respiratory alkalosis that is compensated by the kidneys through the renal excretion of bicarbonate. Because the pH has returned to a normal value, compensation has occurred. **In a respiratory imbalance you will find an opposite response between the pH & the PCO2 as indicated in the question. Therefore, you can eliminate the options reflective of a primary metabolic problem. Remember that the pH ↑ in an alkalotic condition & compensation can be evidenced by a normal pH.

The nurse is caring for a client who is on a mechanical ventilator. Blood gas results indicate a pH of 7.50 & a PCO2 of 30 mm Hg. The nurse has determined that the client is experiencing respiratory alkalosis. Which laboratory value would *most likely* be noted in this condition? 1. Sodium level of 145 mEq/L 2. Potassium level of 3.0 mEq/L 3. Magnesium level of 2.0 mg/dL 4. Phosphorus level of 4.0 mg/dL Reference: Saunders Comprehensive Review for the NCLEX-RN examination (6th ed.)

1. Sodium level of 145 mEq/L *2. Potassium level of 3.0 mEq/L* 3. Magnesium level of 2.0 mg/dL 4. Phosphorus level of 4.0 mg/dL Rationale: Respiratory alkalosis is defined as a deficit of carbonic acid or a ↓ in hydrogen ion concentration that results from the accumulation of base or from a loss of acid without acomparable loss of base in the body fluids. This occurs inconditions that cause over-stimulation of the respiratory system. Clinical manifestations of respiratory alkalosis include lethargy, lightheadedness, confusion, tachycardia, dysrhythmias r/t hypokalemia, nausea, vomiting, epigastric pain, & numbness & tingling of the extremities. All three incorrect options identify normal laboratory values.

3 (This client has experienced a physiological problem and the nurse must assess the client and the emesis to decide on possible interventions.)

1. The nurse is caring for clients on a medical unit. Which task should the nurse implement first? 1. change the abd surgical dressing for a client who ambulated in the hall 2. discuss the correct method of placing Montgomery straps on the client with the UAP 3. Assess the male client who called the desk to say he is nauseated and just vomited 4. Place a call to the extended care facility to give the report on a discharged client

1 ( Nausea and vomiting are common adverse effects of interferon alfa-2a, but continued vomiting should be reported to the physician, because dehydration may occur. The medication may be given by either the subcutaneous or intramuscular route. Flulike symptoms such as a mild temperature elevation, headache, muscle aches, and anorexia are common after initiation of therapy but tend to decrease over time. Focus: Prioritization)

10. A patient with chronic hepatitis C has been receiving interferon alfa-2a (Roferon-A) injections for the last month. Which information gathered during a home visit is most important to communicate to the physician? 1. The patient has persistent nausea and vomiting. 2. The patient injects the medication into the thigh by the intramuscular route. 3. The patient's temperature is 99.7° F (37.6° C) orally. 4. The patient reports chronic fatigue, muscle aches, and anorexia.

10. Anne, who is drinking beer at a party, falls and hits her head on the ground. Her friend Liza dials "911" because Anne is unconscious, depressed ventilation (shallow and slow respirations), rapid heart rate, and is profusely bleeding from both ears. Which primary acid-base imbalance is Anne at risk for if medical attention is not provided? A. Metabolic Acidosis B. Metabolic Alkalosis C. Respiratory Acidosis D. Respiratory Alkalosis

10. Answer: C. Respiratory Acidosis One of the risk factors of having respiratory acidosis is hypoventilation which may be due to brain trauma, coma, and hypothyroidism or myxedema. Other risk factors include COPD, Respiratory conditions such as pneumothorax, pneumonia and status asthmaticus. Drugs such as Morphine and MgSÒ toxicity are also risk factors of respiratory acidosis.

4 (The client has a urinary output of less than 30 mL/hr, therefore this client may be going into renal failure and should be assessed first. : definitions: tenesmus: straining to empty bowels, Jaundice and ascites are expected in client with liver failure, so that client is not first. Barretts esophagus is expected to have dysphagia and pyroisis aka heartburn.)

10. Which client should the nurse assess first after receiving the pm shift assessment? 1. The client with Barretts esophagus who has dysphagia and pyrosis 2. The client with proctitis who has tenesmus and passage of mucus through the rectum 3. the client with liver failure who is jaundiced and has ascites 4. The client with abd pain who has an 8 hr urinary output of 150 mL

3 ( Patients taking immunosuppressive medications are at increased risk for development of cancer. A nontender swelling or lump may signify that the patient has lymphoma. The other data indicate that the patient is experiencing common side effects of the immunosuppressive medications. Focus: Prioritization)

11. A patient with a history of liver transplantation is receiving cyclosporine (Sandimmune), prednisone (Deltasone), and mycophenolate (CellCept). Which finding is of most concern? 1. Gums that appear very pink and swollen 2. A blood glucose level that is increased to 162 mg/dL 3. A nontender lump above the clavicle 4. Grade 1+ pitting edema in the feet and ankles

11. Dave, a 6-year-old boy, was rushed to the hospital following her mother's complaint that her son has been vomiting, nauseated and has overall weakness. After series of tests, the nurse notes the laboratory results: potassium: 2.9 mEq. Which primary acid-base imbalance is this boy at risk for if medical intervention is not carried out? A. Respiratory Acidosis B. Respiratory Alkalosis C. Metabolic Acidosis D. Metabolic Alkalosis

11. Answer: D. Metabolic Alkalosis Vomiting, hypokalemia, overdosage of NaHCǑ and NGT suctioning are considered risk factors of metabolic alkalosis.

12. An old beggar was admitted to the emergency department due to shortness of breath, fever, and a productive cough. Upon examination, crackles and wheezes are noted in the lower lobes; he appears to be tachycardic and has a bounding pulse. Measurement of arterial blood gas shows pH 7.2, PaCÓ 66 mm Hg, HCǑ 27 mmol/L, and PaÓ 65 mm Hg. As a knowledgeable nurse, you know that the normal value for pH is: A. 7.20 B. 7.30 C. 7.40 D. 7.50

12. Answer: C. 7.40 Normal blood pH must be maintained within a narrow range of 7.35-7.45 to ensure the proper functioning of metabolic processes and the delivery of the right amount of oxygen to tissues. Acidosis refers to an excess of acid in the blood that causes the pH to fall below 7.35, and alkalosis refers to an excess of base in the blood that causes the pH to rise above 7.45.

3 (mucosal barrier agent must be administered before the client eats in order to coat the gastric mucosa. Administer this first.)

12. The nurse is preparing to administer morning medications to client on a medical unit. Which medication should then nurse administer first? 1. Methylprenisolon (Solu-Medrol), a steroid to a client with Crohns disease 2. Donepezil (Aricept) an acetylcholinesterase inhibitor, to a client with dementia 3. Sucralfate (Carafate) a mucosal barrier agent to a client dx with ulcer disease 4. Enoxaparin (Lovenox) an anticoagulant to a client on bed rest after abdominal surgery

13. Liza's mother is seen in the emergency department at a community hospital. She admits that her mother is taking many tablets of aspirin (salicylates) over the last 24-hour period because of a severe headache. Also, the mother complains of an inability to urinate. The nurse on duty took her vital signs and noted the following: Temp = 97.8 °F; apical pulse = 95; respiration = 32 and deep. Which primary acid-base imbalance is the gentleman at risk for if medical attention is not provided? A. Respiratory Acidosis B. Respiratory Alkalosis C. Metabolic Acidosis D. Metabolic Alkalosis

13. Answer. C. Metabolic Acidosis Salicylate overdose causes a high anion gap metabolic acidosis in both children and adults. Adults commonly develop a mixed acid-base disorder as a respiratory alkalosis due to direct respiratory centre stimulation occurs as well. This second disorder is uncommon in children.

3 (HARD RIGID ABD)

13. The nurse has received morning shift report on a surgical unit in a community hospital. Which client should the nurse assess first? 1. The client 6 hrs post op small bowel resection who has hypoactive bowel sounds x4 quads 2. The client who is scheduled for abdominal-peritoneal resection this morning who is crying and upset 3. The client who is 1 day post op for abd surgery and has a hard rigid abdomen 4. The client who is 2 days postop for emergency appendectomy and is complaining of abd pain rating 8 on scale 1-10

14. A patient who is hospitalized due to vomiting and a decreased level of consciousness displays slow and deep (Kussmaul breathing), and he is lethargic and irritable in response to stimulation. The doctor diagnosed him of having dehydration. Measurement of arterial blood gas shows pH 7.0, PaÓ 90 mm Hg, PaCÓ 22 mm Hg, and HCǑ 14 mmol/L; other results are Na+ 120 mmol/L, K+ 2.5 mmol/L, and Cl- 95 mmol/L. As a knowledgeable nurse, you know that the normal value for PaCÓ is: A. 22 mm Hg B. 36 mm Hg C. 48 mm Hg D. 50 mm Hg

14. Answer: B. 36 mm Hg The normal range for PaCÓ is from 35 to 35 mm Hg.

3 (The location of the incision for a cholecystectomy the general anesthesia needed, and a heavy smoking history make this client high risk for pulmonary complications. WRONG: #2 The clients high BP should be monitored closely and meds administered but would not cause to have a higher risk for complications.)

15 The nurse is preparing clients for surgery. which client has the greatest potential for experiencing complications? 1. The client scheduled for removal of an abdominal mass who is overweight 2. The client scheduled for a gastrectomy who has arterial hypertension 3. The client scheduled for an open cholecystectomy who smokes two packs of cigs a day 4. The client scheduled for an emergency appendectomy who smoke marijuana on a daily basis

15. A company driver is found at the scene of an automobile accident in a state of emotional distress. He tells the paramedics that he feels dizzy, tingling in his fingertips, and does not remember what happened to his car. Respiratory rate is rapid at 34/minute. Which primary acid-base disturbance is the young man at risk for if medical attention is not provided? A. Respiratory Acidosis B. Respiratory Alkalosis C. Metabolic Acidosis D. Metabolic Alkalosis

15. Answer: B. Respiratory Alkalosis Hyperventilation is typically the underlying cause of respiratory alkalosis. Hyperventilation is also known as overbreathing. When someone is hyperventilating, they tend to breathe very deeply or very rapidly.

16. An elderly client was admitted to hospital in a coma. Analysis of the arterial blood gave the following values: PCÓ 16 mm Hg, HCǑ- 5 mmol/L and pH 7.1. As a well-rounded nurse, you know that the normal value for HCǑ is: A. 20 mmol/L B. 24 mmol/L C. 29 mmol/L D. 31 mmol/L

16. Answer: B. 24 mmol/L The normal value for bicarbonate (HCǑ) is 22-26 mmol/L or mEq/L. It may vary slightly among different laboratories. The given values show the common measurement range of results for these tests. Some laboratories use different measurements or may test different specimens.

17. In a patient undergoing surgery, it was vital to aspirate the contents of the upper gastrointestinal tract. After the operation, the following values were acquired from an arterial blood sample: pH 7.55, PCÓ 52 mm Hg and HCǑ- 40 mmol/l. What is the underlying disorder? A. Respiratory Acidosis B. Respiratory Alkalosis C. Metabolic Acidosis D. Metabolic Alkalosis

17. Answer: D. Metabolic Alkalosis NGT suctioning, vomiting, hypokalemia and overdosage of NaHCǑ are considered risk factors of metabolic alkalosis.

18. A mountaineer attempts an assault on a high mountain in the Andes and reaches an altitude of 5000 meters (16,400 ft) above sea level. What will happen to his arterial PCÓ and pH? A. Both will be lower than normal. B. The pH will rise and PCÓ will fall. C. Both will be higher than normal due to the physical exertion. D. The pH will fall and PCÓ will rise

18. Answer: B. The pH will rise and PCÓ will fall. The mountaineer will suffer from a respiratory alkalosis. The decline in the PÓ with altitude will stimulate breathing to offset the hypoxia. Carbon dioxide is driven from the blood faster than it is produced in the tissues so PCÓ falls and pH rises.

3 (Wound dehiscence is the premature bursting open of a wound along surgical suture and is an emergency that would require first assessment.)

19 The med surg nurse has just received the am shift report. Which client should the nurse assess first? 1. The client who has a parlytic ileus and absent bowel sounds 2. The client who is 2 days post op abdominal surgery and has a soft tender abdomen 3. The client who is 6 hrs postop and has an abdominal wound dehiscence 4. The client who had a liver transplant and is being transferred to the rehab unit

19. A young woman is found comatose, having taken an unknown number of sleeping pills an unknown time before. An arterial blood sample yields the following values: pH 6.90, HCǑ- 13 meq/liter and PaCÓ 68 mmHg. This patient's acid-base status is most accurately described as: A. Metabolic Acidosis B. Respiratory Acidosis C. Simultaneous Respiratory and Metabolic Acidosis D. Respiratory Acidosis with Complete Renal Compensation

19. Answer: C. Simultaneous Respiratory and Metabolic Acidosis Whenever the PCÓ and HCǑ are abnormal in opposite directions, ie, one above normal while the other is reduced, a mixed respiratory and metabolic acid-base disorder exists. When the PCÓ is elevated and the [HCǑ-] reduced, respiratory acidosis and metabolic acidosis coexist.

1 ( The UAP should use infection control precautions for the protection of self, employees, and other clients. Monitoring is an RN responsibility. UAPs can report valuable information; however, they are not responsible for detecting signs and symptoms that can be subtle or hard to detect, such as skin changes. While playing games with the client may be ideal, it is rarely possible on a medical-surgical unit. Focus: Delegation)

19. In the care of a client with acute viral hepatitis, which task should be delegated to the UAP? 1. Emptying the bedpan while wearing gloves 2. Playing games or engaging the client in diversional activities 3. Monitoring dietary preferences 4. Reporting signs and symptoms of jaundice

A nurse is caring for a client with a nasogastric tube that is attached to low suction. The nurse monitors the client closely for which acid-base disorder that is most likely to occur in this situation? 1. Metabolic acidosis 2. Metabolic alkalosis 3. Respiratory acidosis 4. Respiratory alkalosis

2

A nurse is caring for a client with respiratory insufficiency. The ABG results indicate a pH of 7.50 and a PCO2 of 30 mm Hg, and the nurse is told that the client is experiencing respiratory alkalosis. Which of the following additional laboratory values would the nurse expect to note? 1. A sodium level of 145 mEq/L 2. A potassium level of 3.2 mEq/L 3. A magnesium level of 2.4 mg/dL 4. A phosphorus level of 4.0 mg/dL

2

What combination of medications is used to eradicate H. pylori?

2 antibiotics and 1 PPI

2. Carl, an elementary student, was rushed to the hospital due to vomiting and a decreased level of consciousness. The patient displays slow and deep (Kussmaul breathing), and he is lethargic and irritable in response to stimulation. He appears to be dehydrated—his eyes are sunken and mucous membranes are dry—and he has a two week history of polydipsia, polyuria, and weight loss. Measurement of arterial blood gas shows pH 7.0, PaÓ 90 mm Hg, PaCÓ 23 mm Hg, and HCǑ 12 mmol/L; other results are Na+ 126 mmol/L, K+ 5 mmol/L, and Cl- 95 mmol/L. What is your assessment? A. Respiratory Acidosis, Uncompensated B. Respiratory Acidosis, Partially Compensated C. Metabolic Alkalosis, Uncompensated D. Metabolic Acidosis, Partially, Compensated

2. Answer: D. Metabolic Acidosis, Partially, Compensated The student was diagnosed having diabetes mellitus. The results show that he has metabolic acidosis (low HCǑ -) with respiratory compensation (low CÓ).

20. A mother is admitted in the emergency department following complaints of fever and chills. The nurse on duty took her vital signs and noted the following: Temp = 100 °F; apical pulse = 95; respiration = 20 and deep. Measurement of arterial blood gas shows pH 7.37, PaÓ 90 mm Hg, PaCÓ 40 mm Hg, and HCǑ 24 mmol/L. What is your assessment? A. Hyperthermia B. Hyperthermia and Respiratory Alkalosis C. Hypothermia D. Hypothermia and Respiratory Alkalosis

20. Answer: A. Hyperthermia An individual is considered to have hyperthermia if he or she has a temperature of >37.5 or 38.3 °C (99.5 or 100.9 °F). Measurement of arterial blood gases are normal.

1 ( There is a potential for sudden rupture of fragile blood vessels with massive hemorrhage from straining that increases thoracic or abdominal pressure. The client could have fluid accumulation in the abdomen (ascites) that can be mild and hard to detect or severe enough to cause orthopnea. Dependent peripheral edema can also be observed but is less urgent. Focus: Prioritization)

20. You are caring for a client with cirrhosis and portal hypertension. Which statement by the client concerns you the most? 1. "I'm very constipated and have been straining during bowel movements." 2. "I can't button my pants anymore because my belly is so swollen." 3. "I have a tight sensation in my lower legs when I forget to put my feet up." 4. "When I sleep, I have to sit in a recliner so that I can breathe more easily."

2 (Pain is priority because the nurse must first determine if this is expected pain or complication of surgery. Assess this client first. #1 the ABGs reflect metabolic alkalosis which is expected with excessive vomiting)

21 The nurse is caring for clients on a surgical unit. Which client should the nurse assess first? 1. The client who has been vomiting for 2 days and has an ABG of pH 7.47, PaO2 95, PacO2 44, HCO3 30 2. The client who is 8 hrs postop for splenectomy and who is complaining of abdominal pain, rating it as a 9 on a scale of 1-10 3. The client who is 12 hrs postop abd surgery and has dark green bile draining in the NG tube 4. The client who is 2 days post op for hiatal hernia repair and is complaining of feeling constipated

HCO3

22 - 26 mEq/L

2, 3 ( Both clients will need frequent pain assessments and medications. Clients with copious diarrhea or vomiting will frequently need enteric isolation. Cancer clients receiving chemotherapy are at risk for immunosuppression and are likely to need protective isolation. Focus: Assignment)

23. You must rearrange the room assignments for several clients. Which two clients would be best to put in the same room? 1. 35-year-old woman with copious intractable diarrhea and vomiting 2. 43-year-old woman who underwent cholecystectomy 2 days ago 3. 53-year-old woman with pain related to alcohol-associated pancreatitis 4. 62-year-old woman with colon cancer receiving chemotherapy and radiation _____, _____

3 (The clients apical pulse is above normal and the BP is low which are signs of hypovolemic shock, which warrants immed intervention. #1 Hemoglobin 9 think tranfusion time, this H/H is ok #4 coffee ground indicates old blood which would not be unexpected in the client with esophageal bleeding)

25 The client is dx with esophageal bleeding.. Which of the following assessment data warrants immediate intervention by the nurse? 1. The clients H/H is 11.4/32 2. The clients abdomen is soft to touch and non-tender 3. The clients VS are T 99, AP 114, RR 18, BP 88/60 4. The clients nasogastric tube has coffee ground drainage

3 ( The UAP can take vital signs and report all of the values to the RN. In this case, all of the values are needed in order to detect trends. In other cases, you may decide to give parameters for reporting. The RN should assess skin temperature and pain, and closely monitor the urine because quantity is an indicator of perfusion and red/pink urine can signal damage to the urinary system, transfusion reaction, or rhabdomyolysis. Focus: Delegation)

25. You are caring for a client who was admitted to your medical-surgical unit for observation after being evaluated in the emergency department for blunt trauma to the abdomen. Which instructions are appropriate to give to the UAP? 1. Check the client's skin temperature and report if the skin feels cool. 2. Check the urine in the urometer every hour and observe for red- or pink-tinged urine. 3. Check vital signs every hour and report all of the values. 4. Check the client's pain and report worsening of pain or discomfort.

1 ( Refeeding syndrome occurs when aggressive and rapid feeding results in fluid retention and heart failure. Electrolytes, especially phosphorus, should be monitored, and the client should be observed for signs of fluid overload. Changes in bowel sounds, nausea, and distention may occur but are also appropriate for any client with nutritional issues or for clients receiving enteral feedings. Observing for purging and water ingestion would be appropriate for a client with an eating disorder. Change in stool patterns may occur, but are not related to refeeding syndrome. Focus: Prioritization)

27. Clients who are undernourished or starved for prolonged periods are at risk for refeeding syndrome when nourishment is first given. What is the priority nursing assessment to prevent complications associated with this syndrome? 1. Monitor for peripheral edema, crackles in the lungs, and jugular vein distention. 2. Monitor for decreased bowel sounds, nausea, bloating, and abdominal distention. 3. Observe for signs of secret purging and ingestion of water to increase weight. 4. Assess for alternating constipation and diarrhea and pale clay-colored stools.

3 ( All of these measures should be performed for total care of the client; however weighing the client every day is considered the single best indicator of fluid volume. Focus: Prioritization)

28. You are caring for a client who was admitted for advanced cirrhosis. The client has massive ascites, peripheral dependent edema in the lower extremities, nausea and vomiting, and dyspnea related to pressure on the diaphragm. For the nursing diagnosis of Excess Fluid Volume, which indicator is the most reliable for tracking fluid retention? 1. Auscultating the lung fields for crackles every day 2. Measuring the abdominal girth every morning 3. Performing daily weights with the same amount of clothing 4. Checking the extremities for pitting edema and comparing to baseline

3 ( Substance abuse may exclude a person from the transplant list, so the nurse should conduct additional assessment about this comment. The comment about difficulty in taking prescription medications should also be investigated because a true inability to follow the treatment regimen would also exclude the client from the list. Focus: Prioritization)

29. A client with end-stage liver disease is talking to you about being on the transplant list. Which statement by the client concerns you the most? 1. "I have a family history of diabetes." 2. "I had symptoms of asthma when I was a kid." 3. "I am going to cut down on my drinking very soon." 4. "I am not very good about taking prescribed medication."

The RN reviews the results of the ABG with the LPN and tells the LPN that the client is experiencing respiratory acidosis. The LPN would expect to note which of the following on the laboratory result form? 1. pH 7.50, PCO2 52 mm Hg 2. pH 7.35, PCO2 40 mm Hg 3. pH 7.25, PCO2 50 mm Hg 4. pH 7.50, PCO2 30 mm Hg

3

A nurse is caring for a client with a diagnosis of COPD. The nurse monitors the client for which acid-base imbalance that most likely occurs in clients with this condition? 1. Metabolic acidosis 2.Metabolic alkalosis 3. Respiratory acidosis 4. Respiratory alkalosis

3.

3. A cigarette vendor was brought to the emergency department of a hospital after she fell into the ground and hurt her left leg. She is noted to be tachycardic and tachypneic. Painkillers were carried out to lessen her pain. Suddenly, she started complaining that she is still in pain and now experiencing muscle cramps, tingling, and paraesthesia. Measurement of arterial blood gas reveals pH 7.6, PaÓ 120 mm Hg, PaCÓ 31 mm Hg, and HCǑ 25 mmol/L. What does this mean? A. Respiratory Alkalosis, Uncompensated B. Respiratory Acidosis, Partially Compensated C. Metabolic Alkalosis, Uncompensated D. Metabolic Alkalosis, Partially Compensated

3. Answer: A. Respiratory Alkalosis, Uncompensated The primary disorder is acute respiratory alkalosis (low CÓ) due to the pain and anxiety causing her to hyperventilate. There has not been time for metabolic compensation.

4 (The client with Crohns disease should be asymptomatic, so pain and diarrhea warrant intervention by the nurse, pain could indicate a complication. WRONG: The other answers all have expected symptoms for the condition/diagnoses definitions: stretorrhea=fat frothy stools, pyrexia=fever, dyspepsia=upset stomach, eructation=belching)

3. Which client warrants immediate intervention from the nurse on the medical unit? 1. The client diagnosed with dyspepsia who has eructation and bloating 2. The client diagnosed with pancreatitis who has steatorrhea and pyrexia 3. The client with diverticulitis who has LLQ pain and fever 4. The client with Crohns disease who has right lower abd pain and diarrhea

1 (The nurse should first assess the clients neurological status to determine the status of the client.)

30 The LPN tells the nurse the client dx with liver failure is getting more confused. Which intervention should the nurse implement first? 1. Assess the clients neurological status 2. Notify the clients healthcare provider 3. Request a stat ammonia serum level 4. Tell the LPN to obtain the clients VS

3 ( T-tubes should not be irrigated, aspirated, or clamped without a specific order from the physician. All of the other actions are appropriate in the care of this client. Focus: Supervision)

30. You are supervising a nursing student who is caring for a client who had a cholecystectomy. There is a T-tube in place. You would intervene if the student performs which action? 1. Maintains the client in a semi-Fowler position 2. Checks the amount, color, and consistency of the drainage 3. Gently aspirates the drainage from the tube 4. Inspects the skin around the tube for redness or irritation

PCO2

35 - 45 mm Hg

A client has the following lab values: a pH of 7.55, an HCO3- of 22 mm Hg, and a PCO2 of 30 mm Hg. What should the nurse do? 1. Perform Allen's test 2. Prepare the client for dialysis 3. Administer insulin as ordered 4. Encourage the client to slow down breathing

4.

A nurse is caring for a client with diabetic ketoacidosis and documents that the client is experiencing Kussmaul's respirations. Based on this documentation, which of the following did the nurse most likely observe? 1. Respirations that cease for several seconds 2. Respirations that are regular but abnormally slow 3. Respirations that are labored and increased in depth and rate 4. Respirations that are abnormally deep, regular, and increased in rate

4.

The nurse is told that the blood gas results indicate a pH of 7.55 and a PCO2 of 30 mm Hg. The nurse determines that these results indicate: 1. Metabolic acidosis 2. Metabolic alkalosis 3. Respiratory acidosis 4. Respiratory alkalosis

4.

4. Ricky's grandmother is suffering from persistent vomiting for two days now. She appears to be lethargic and weak and has myalgia. She is noted to have dry mucus membranes and her capillary refill takes >4 seconds. She is diagnosed as having gastroenteritis and dehydration. Measurement of arterial blood gas shows pH 7.5, PaÓ 85 mm Hg, PaCÓ 40 mm Hg, and HCǑ 34 mmol/L. What acid-base disorder is shown? A. Respiratory Alkalosis, Uncompensated B. Respiratory Acidosis, Partially Compensated C. Metabolic Alkalosis, Uncompensated D. Metabolic Alkalosis, Partially Compensated

4. Answer: C. Metabolic Alkalosis, Uncompensated The primary disorder is uncompensated metabolic alkalosis (high HCǑ -). As CÓ is the strongest driver of respiration, it generally will not allow hypoventilation as compensation for metabolic alkalosis.

1 (The nurse should not delegate to the UAP feeding a client who is not stable and at risk for complications during feeding, as a result of dysphagia. This requires judgement the UAP is not expected to possess.)

4. The nurse and the UAP are caring for clients on a med-surg unit. Which task should not be assigned to the UAP? 1. Instruct the UAP to feed the 69 year old who is experiencing dysphagia 2 REquest the UAP change the linens for the 89 year old client with fecal incontinence 3. Tell the UAP to assist the 54 year old client with a bowel management program 4. Ask the UAP to obtain VS on the 72 year old client diagnosed with cirrhosis

2

40 The significant other of a client dx with liver cancer and who is dying asks the nurse, what is bereavement counseling? Which statement is the nurses best response? 1. Bereavement counseling helps the client accept the terminal illness 2. It provides support to you and your family in the transition to a life without your loved one 3. We provide counseling to you and your loved one during the dying process 4. It is a group counseling for family members whose loved ones have died

3

41 The nurse is working in a digestive disease disorder clinic. Which nursing action is an example of evidence-based practice? 1. Turn on the tap water to help a client urinat 2. Use two identifiers to identify a client before a procedure 3. Educate a client based on current published information 4. Read nursing journals about the latest procedures

2

43 The administrative supervisor is staffing the hospitals med surg units during an icestorm and has received many calls from staff who are unable to get to the hospital. Which action should the supervisor implement first? 1. inform the cheif nursing officer 2. notify the on duty staff to stay 3. call staff members who live close to the facility 4. implement the emergency disaster protocol

3 (This client is exhibiting symptoms asthma a complication of GERD therefore the client should be assigned to the most experienced nurse. #4 PAin is expected with a surgical procedure and a less experienced nurse could admin pain meds #1 regurgitation is a common manifestation of gerd, so not the most experienced nurse #2 Barretts esophagitis is a complication of gerd, new grads can prepare the client for a diagnostic procedure)

46 The charge nurse is making assignments on a med surg unit. Which client should be assigned to the most experienced nurse? 1 the client with lower esophageal dysfunction who is experiencing regurgitation 2. The client dx with Barretts esophagitis who is scheduled for an endoscopy 3. The client dx with gastroesophageal reflux disease who has bilateral wheezed 4. The client dx with 1 day post op hiatal hernia who has pain rated a 4 on a scale of 0-10

3 (high risk due to hypokalemia. Assess the cardiac status and then implement other interventions)

47 The client is experiencing severe diarrhea and has a serum potassium level of 3.3 Which intervention should the nurse implement first? 1. Notify the HCP 2. Assess for leg cramps 3. Place on telemetry 4. Prepare to admin IV potassium

5. Mrs. Johansson, who had undergone surgery in the post-anesthesia care unit (PACU), is difficult to arouse two hours following surgery. Nurse Florence in the PACU has been administering Morphine Sulfate intravenously to the client for complaints of post-surgical pain. The client's respiratory rate is 7 per minute and demonstrates shallow breathing. The patient does not respond to any stimuli! The nurse assesses the ABCs (remember Airway, Breathing, Circulation!) and obtains ABGs STAT! Measurement of arterial blood gas shows pH 7.10, PaCÓ 70 mm Hg and HCǑ 24 mEq/L. What does this mean? A. Respiratory Alkalosis, Partially Compensated B. Respiratory Acidosis, Uncompensated C. Metabolic Alkalosis, Partially Compensated D. Metabolic Acidosis, Uncompensated

5. Answer: B. Respiratory Acidosis, Uncompensated The results show that Mrs. Johansson has respiratory acidosis because of decreased pH and increased PaCÓ which mean acidic in nature. Meanwhile, it is uncompensated because HCǑ is within the normal range.

3 (A client with a continuous feeding tube should be in the Fowlers or high Fowlers position to prevent aspiration pneumonia. This action requires immediate intervention)

5. Which behavior by the UAP requires immediate intervention by the nurse? 1 The UAP is refusing to feed the client Dx with acute diverticulitis 2 The UAP would not place the client on the bedside commode who was on bed rest 3. The UAP placed the client with continuous feeding tube in supine position 4. The UAP placed sequential compression devices on the client who is on strict bed rest.

3 (Direct pressure is applied to the site and the client is placed on the right side to maintain site pressure for at least 2 hours. Turning the client to the left side warrants intervention by the nurse so the client will not hemorrhage. #1 The client should stay on his or her right side for at least 2 hrs post procedure so giving a urinal to void is ok. #2 the client is NOT NPO after, so they can have water)

58 The male client is 30 mins post procedure liver biopsy. Which action by the UAP requires the nurse to intervene? 1. The UAP offered the client a urinal to void 2. The UAP gave the client a glass of water 3. The UAP turned the client on the left side 4. The UAP took the clients VS

3 (Mittens will help prevent the client from scratching the skin and causing skin breakdown, which is a priority for the client with liver failure. The client has decreased Vitamin K, which will lead to bleeding. The client is also immunosuppressed which will lead to infection. WRONG: #1 Hot water increases pruritis and soap will cause dry skin, #2 this will help dry skin but this is not the first intervention to protect #4 benedryl will help decrease pruritis but will take 30 mins to work, protection is the priority)

59 The client dx with liver failure is experiencing pruritus secondary to sever jaundice and is scratching the upper extremities. Which intervention should the nurse implement first? 1. Request the UAP to assist the client to take a hot soapy shower 2. Apply emollient to the clients upper extremities 3. Place mittens on both hands of the client 4. Administer benedryl 25 mg PO to the client

6. Baby Angela was rushed to the Emergency Room following her mother's complaint that the infant has been irritable, difficult to breastfeed and has had diarrhea for the past 3 days. The infant's respiratory rate is elevated and the fontanels are sunken. The Emergency Room physician orders ABGs after assessing the ABCs. The results from the ABG results show pH 7.39, PaCÓ 27 mmHg and HCǑ 19 mEq/L. What does this mean? A. Respiratory Alkalosis, Fully Compensated B. Metabolic Acidosis, Uncompensated C. Metabolic Acidosis, Fully Compensated D. Respiratory Acidosis, Uncompensated

6. Answer: C. Metabolic Acidosis, Fully Compensated Baby Angela has metabolic acidosis due to decreased HCǑ and slightly acidic pH. Her pH value is within the normal range which made the result fully compensated.

2 (Milk thistle has an active ingredient silymarin, which has been used to treat liver disease for more than 2000 years. It is a powerful oxidant and promotes liver cell growth. This response gives the client factual information)

60 The client with hepatitis asks the nurse "Is there any herb I can take to help my liver get better?" Which statement is the nurses best response? 1. You should ask your HCP about taking herbs 2. Milk thistle is a powerful oxidant and promotes liver cell growth 3. You should not take any medication that is not prescribed 4. Why would you want to take any herbs?

1 (Pain should be assessed even if it is expected for the clients dx if other clients are stable)

64 The nurse has received the am shift report. Which client should the nurse assess first? 1. the client with peptic ulcer disease who is complaining of acute epigastric pain 2. The client with acute gastroenteritis who is upset and wants to go home 3. The client with inflammatory bowel disease who is receiving TPN 4. The client with hep B who is complaining and who is jaundiced and anorexic

2 (The client was just transferred from the PACU therefore the nurse should assess this client first to perform a baseline assessment to ensure the client is stable)

67 The nurse is caring for the following clients on a surgical unit. Which client should the nurse assess first? 1. The client with an inguinal hernia repair who has a urine output of 160 mL in 4 hrs 2. The client with an emergency appendectomy who was transferred from PACU 3. The client who is 4 hours post op abd surgery who has flatulence 4. The client who is 6 hrs post procedure colonoscopy and is being discharged

7. Mr. Wales, who underwent post-abdominal surgery, has a nasogastric tube. The nurse on duty notes that the nasogastric tube (NGT) is draining a large amount (900 cc in 2 hours) of coffee ground secretions. The client is not oriented to person, place, or time. The nurse contacts the attending physician and STAT ABGs are ordered. The results from the ABGs show pH 7.57, PaCÓ 37 mmHg and HCǑ 30 mEq/L. What is your assessment? A. Metabolic Acidosis, Uncompensated B. Metabolic Alkalosis, Uncompensated C. Respiratory Alkalosis, Uncompensated D. Metabolic Alkalosis, Partially Compensated

7. Answer: B. Metabolic Alkalosis, Uncompensated The postoperative client's ABG results show that he has metabolic alkalosis because of an increased pH and HCǑ. It is uncompensated due to the normal PaCÓ which is within 35 to 45 mmHg.

pH

7.35 - 7.45

8. Client Z is admitted to the hospital and is to undergo brain surgery. The client is very anxious and scared of the upcoming surgery. He begins to hyperventilate and becomes very dizzy. The client loses consciousness and the STAT ABGs reveal pH 7.61, PaCÓ 22 mmHg and HCǑ 25 mEq/L. What is the ABG interpretation based on the findings? A. Metabolic Acidosis, Uncompensated B. Respiratory Alkalosis, Partially Compensated C. Respiratory Alkalosis, Uncompensated D. Metabolic Alkalosis, Partially Compensated

8. Answer: C. Respiratory Alkalosis, Uncompensated The results show that client Z has respiratory alkalosis since there is an increase in the pH value and a decrease in PaCÓ which are both basic. It is uncompensated due to the normal HCǑ which is within 22-26 mEq/L.

2 (This client is being prepared for a test in the morning and is the least acute of the clients listed. The new grad should be assigned to this client.)

8. The charge nurse is making assignments on a medical unit. Which client should the nurse assign to the graduate nurse? 1. The client who has received 3 units of PRBCs 2. The client scheduled for an esophagogastroduodenoscopy in the morning 3. The client with short bowel syndrome who has diarrhea and a K+ level of 3.3 4. The client who has just returned from surgery for a sigmoid colostomy

PO2

80% - 100%

9. Three-year-old Adrian is admitted to the hospital with a diagnosis of asthma and respiratory distress syndrome. The mother of the child reports to the nurse on duty that she has witnessed slight tremors and behavioral changes in her child over the past four days. The attending physician orders routine ABGs following an assessment of the ABCs. The ABG results are pH 7.35, PaCÓ 72 mmHg and HCǑ 38 mEq/L. What acid-base disorder is shown? A. Respiratory Acidosis, Uncompensated B. Respiratory Acidosis, Fully Compensated C. Respiratory Alkalosis, Fully Compensated D. Metabolic Alkalosis, Partially Compensated

9. Answer: B. Respiratory Acidosis, Fully Compensated The patient has respiratory acidosis (raised carbon dioxide) resulting from asthma and respiratory distress syndrome, with compensation having normal pH value within 7.35to 7.45, increased PaCÓ which is acidic and increased HCǑ which is basic.

SaO2

95% - 100%

Metabolic Acidosis - Signs/Symptoms

>Bradycardia >Weak pulses >Hypotension >Tachypnea >Flaccid paralysis >Confusion >Headache >Hyporeflexia >Lethargy >Warm, flushed, dry skin >Kussmaul's respirations >Hyperkalemia >Diarrhea

Metabolic Acidosis - Risk factors/Causes

>Diarrhea >Fever >Hypoxia >Starvation >Seizure >Overdose, salicylates or ethanol >Renal failure >DKA >Dehydration >Shock

Metabolic Alkalosis - Signs/Symptoms

>Dizziness >Restlessness -> Lethargy >Paresthesias >Hypertonic muscles (tremors) >↓ respirations >Tachycardia >Hypoventilation >Hypokalemia >N/V, Diarrhea

Respiratory Acidosis - Signs/Symptoms

>Dizziness, drowsiness >Headache >Palpitations >Muscle twitching, hyperreflexia >Convulsions >Hypoventilation -> Hypoxia >Rapid, shallow respirations >Dyspnea >Hypotension w/ vasodilation >Hyperkalemia >Dysrhythmias

Respiratory Alkalosis - Signs/Symptoms

>Lethargy, confusion >Lightheadedness >Tachypnea >Hyperventilation >Anxiety, tetany >Paresthesias >Numbness/tingling of extremities >Tachycardia >Palpitations >Chest pain >↓ BP or normal >Seizures >N/V

Respiratory Acidosis - Interventions

>Maintain patent airway >Reversal agents for narcotics >Regulation ventilation therapy >Bronchodilators >Mucolytics

Respiratory Alkalosis - Interventions

>Regulate O2 therapy >Reduce anxiety >Rebreathing techniques

Metabolic Acidosis - Interventions

>Treat underlying cause >Administer fluids, electrolytes

Metabolic Alkalosis - Interventions

>Treat underlying causes >Administer fluids, electrolytes

The nurse recognizes which patient is at greatest risk for type 1 autoimmune hepatitis?

A 16-year-old female with type 1 diabetes mellitus

The nurse is screening patients for their risk of developing acute gastritis. The nurse should consider which patient at greatest risk?

A 32-year-old man who takes ibuprofen daily (NSAIDS)

4 (Rationale: bowel perforation, obstruction or hemorrhage and toxic megacolon are common complications of ulcerative colitis that may require surgery. )

A HCP and nurse are discussing options with a client diagnosed with severe ulcerative colitis. When providing client teaching during early treatment the symptoms of which diagnosis would be discussed? 1. gastritis 2. bowel herniation 3. bowel outpouching 4. bowel perforation

a,e (Rationale: Edematous tissue must receive meticulous care to prevent tissue breakdown. When jaundice is present, bile salts can deposit on the skin, causing pruritus and scratching by the client to relive itching, which promotes skin breakdown. Warm water should be used for bathing rather than hot water as the latter increases itching. An air pressure mattress and careful repositioning can prevent skin breakdown, and having the client in different positions, such as chair-sitting, can relieve pressure on the skin. However, having the client sit in a chair for 30 minutes each shift may be too disruptive to rest and sleep and may not be possible for the severely ill bedridden individual. Range of motion exercises preserve joint function but do not prevent skin breakdown. )

A client diagnosed with chronic cirrhosis has jaundice, ascites, and pitting peripheral edema as well as hepatic encephalopathy. Which nursing interventions are most appropriate to prevent skin breakdown? (Select all that apply.) a Turning and repositioning every 2 hours b Using hot water to bathe to relieve pruritus c Asking client to sit in a chair for 30 minutes each shift d Range of motion every 4 hours e Alternating air pressure mattress

c (Rationale Hepatic encephalopathy may be aggravated by sepsis secondary to​ infection, due to increased buildup of toxic​ substances, in clients with cirrhosis. Portal​ hypertension, esophageal​ varices, and Wilson disease are not caused or aggravated by infection.)

A client diagnosed with liver cirrhosis is being treated for an infection. For which complication should the nurse monitor the​ client? a Esophageal varices b Wilson disease c Hepatic encephalopathy d Portal hypertension

1 (because the biopsy needles insertion site is close to the lung, there is a risk of lung puncture and pneumothorax; therefore immed after the procedure the nurse should determine diminished or absent lung sounds in the right lung. Although fever indicates infection, a rise in temp is not seen immed. A CBC is warranted if the VS and client symptoms indicate potential hemorrhage. The needle insertion site is covered with a pressure dressing there is no need for a dressing requiring packing.)

A client had a liver biopsy 1 hr ago. The nurse should first: 1. auscultate lung sounds 2 check for fever 3. obtain CBC 4. apply packing to the biopsy site

3 (Clients with Hep C should receive geno-type testing to determine the most effective treatment approach. and it must be done prior to starting drug treatment with alph-interferon. There are six types of hep C genotypes and clients have different responses to drugs depending on their genotype. The recommended course of treatment depends on the genotype)

A client has a positive serologic test for anti-HCV (hep C virus). The nurse should instruct the client: 1. how to self administer alpha interferon 2. that the HCV will resolve in approximately 3 months 3. That a follow up appt for HCV genotype testing is required 4. to take alpha-interferon as prescribed

4 (Portal Hypertension and hypoalbuminemia is a result of cirrhosis cause a fluid shift into the peritoneal space causing ascites. In a cardiac or kidney problem, NOT CIRRHOSIS, sodium can promote edema formation and subsequent decreased urine output. Edema does not migrate upward toward the heart to enhance circ. Although diuretics promote the excretion of excess fluid, occasionally forgetting or omitting a dose will not yield the ascites found in cirrhosis of the liver)

A client has advanced cirrhosis of the liver. The clients spouse asks the nurse why his abdomen is swollen, making it very difficult for him to fasten his pants. How should the nurse respond to provide the most accurate explanation of the disease process? 1. He must have been eating too many foods with salt in them. Salt pulls water with it 2. The swelling in his ankles must have moved up closer to his heart so the fluid circulates better 3. He must have forgotten to take his daily water pill 4. Blood is not able to flow readily through the liver now, and the liver cannot make protein to keep fluid inside the blood vessels

d (Rationale: The highest priority is protecting the airway so if the client develops respiratory distress, the nurse would deflate the esophageal balloon to avoid compression on the airway. The physician would be notified following this action. The nurse would not remove the NG tube, and the client would not be placed in a supine position. An appropriate syringe should be kept at the bedside to deflate the esophageal balloon in case respiratory distress occurs.)

A client hospitalized with cirrhosis and bleeding varices is being treated with esophageal balloon and gastric balloons using a multiple-lumen nasogastric tube to apply pressure to the varices. An endotracheal tube has already been inserted/is being treated. Which of the following is the priority action by the nurse if the client develops respiratory distress? a Contact the physician. b Place the client supine. c Remove the nasogastric tube. d Deflate the esophageal balloon.

c (Rationale: The nurse should assess bowel sounds and palpate for tenderness since spontaneous bacterial infection (spontaneous bacterial peritonitis) can develop with ascites, producing abdominal discomfort, fever, and worsening encephalopathy. Headache and nuchal rigidity are symptoms of meningitis. Neck vein distention is associated with right-sided heart failure. Abdominal girth and shifting dullness are important in monitoring progress of ascites, not infection. )

A client hospitalized with severe ascites due to cirrhosis develops abdominal pain, fever, and confusion. As part of the initial plan for care, the nurse should first: a Inquire about headache and check for nuchal rigidity. b Measure abdominal girth and percuss for shifting dullness. c Auscultate bowel sounds and palpate the abdomen for tenderness. d Observe for neck vein distention and auscultate lung sounds.

1 (The immediate problem is controlling the diarrhea. Addressing this problem is a step toward correcting the nutritional imbalance and decreasing the diarrheal cramping. Self-care and compliance with the treatment plan are important long-term goals that can be addressed when the client is feeling better physically. Focus: Prioritization)

A client hospitalized with ulcerative colitis reports 10 to 20 small diarrhea stools per day, with abdominal pain before defecation. The client appears depressed and underweight and is uninterested in self-care or suggested therapies. What is the priority nursing diagnosis? 1. Diarrhea related to irritated bowel 2. Imbalanced Nutrition: Less than Body Requirements related to nutrient loss 3. Acute Pain related to increased GI motility 4. Ineffective Self-Health Management related to treatment plan

4,5 (IBS (Crohns) is an inflammatory bowel disease caused by the inflammation of the lining of the digestive tract, which can lead to abdominal pain, severe diarrhea, and even malnutrition. Corticosteroids such as prednisone reduce the S/S of diarrhea, pain and bleeding by decreasing inflammation. Antidiarrheals, combat diarrhea by decreasing peristalsis. Lactulose is used to treat chronic constipation and would aggravate the symptoms of Crohns, A high fiber diet and milk and milk products are contraindicated in clients with Crohns because the may promote diarrhea.)

A client is admitted with inflammatory bowel syndrome (Crohns disease). When planning care for the health care team, which would be included? Select all that apply 1. Lactulose therapy 2. high-fiber diet 3. high protein milkshakes 4. corticosteroid therapy 5. antidiarrheal medications

a,c,d,e (Portals of entry for infection that may lead to sepsis​ include, but are not limited​ to, intravenous​ catheters, surgical​ wounds, sexually transmitted​ infections, and peptic ulcerations. Pulse oximetry is not an invasive procedure and is not a portal of entry for infectious sepsis)

A client is at risk for infectious sepsis through which portals of​ entry? ​(Select all that​ apply.) a Surgical wounds b Pulse oximetry monitoring c Sexually transmitted infections d Intravenous catheters e Peptic ulcerations

4

A client is recieving TPN. The nurse should assess the clients ability to metabolize the solution adequately by monitoring the client for which sign? select all that apply 1. tachycardia 2. hypertension 3. elevated BUN 4. hyperglycemia

2 (The taste of lactulose is a problem for some clients. Mixing it with fruit juice, water or milk can make it more palatable. Lactulose should NOT be given with antacids which would inhibit action. Lactulose should not be taken with a laxative because increased stooling is an adverse effect of the drug and would be potentiated by using a laxative. Lactulose comes in the form of syrup for oral and rectal administration)

A client is to be discharged with a prescription for lactulose. The nurse teaches the client and the clients souse how to administer this medication. Which statement would indicate the client has understood the information? 1. I will take it with an antacid 2. I will mix it with apple juice 3. I will take it with a laxative 4 I will mix the crushed tablets in some gelatin

2

A client newly diagnosed with ulcerative colitis who has been places on steroids asks the nurse why steroids are prescribed. The nurse should tell the client: 1. ulcerative colitis can be cured with steroids 2. steroids are used in severe flare ups because they can decrease the incidence of bleeding 3. long term use of steroids will prolong periods of remission 4. the side effects of steroids outweigh their benefits to clients with ulcerative colitis

1 (Current therapy includes a combination of IV interferon and ribavirin that often includes unpleasant side effects and requires frequent monitoring. The recent approval of oral, directly acting antiviral agents (telaprevir, boceprevir, sofosbuvir, simeprevir) is expected to decrease monitoring rates and increase cure rates, though these drugs are currently very expensive. Though answers 3 and 4 may be true, it is not appropriate to make judgements about a clients health insurance and lifestyle choices)

A client recently dx with Hep C states: Now that you know what is wrong with me, you can just get me the new drugs to take care of it right? The nurse tells the client: 1. The Tx is complex. There are new antiviral drugs available that may make Tx more effective and help you tolerate it better 2. There are drugs to help with the symptoms, but once you have Hep C you will never be cured 3. The medicine currently used to treat Hep C is very expensive, and your insurance probably will not pay for it 4. If you continue to make the same lifestyle choices the medicine will not make any difference

c (Food will be withheld from the client with severe symptoms of ulcerative colitis to rest the bowel. To maintain the client's nutritional status, the client will be started on TPN. Enteral feedings or dividing the diet into 6 small meals does not allow the bowel to rest. A high-calorie, high-protein diet will worsen the client's symptoms.)

A client who has ulcerative colitis has persistent diarrhea. He is thin and has lost 12 pounds since the exacerbation of his ulcerative colitis. The nurse should anticipate that the physician will order which of the following treatment approaches to help the client meet his nutritional needs? A Initiate continuous enteral feedings B Encourage a high protein, high-calorie diet C Implement total parenteral nutrition D Provide six small meals a day

a,c,e (Rationale: A client with ulcerative colitis having a total colectomy with an ileal pouch-anal anastomosis and ileostomy can expect that the anal anastomosis will be healed in 2-3 months. The ileostomy is then closed. Meanwhile, the client will not have control of the bowels and must always wear a collection device. .Once the bowel is healed and the ileostomy closed, the client can expect to have 6-8 bowel movements a day)

A client who has ulcerative colitis is scheduled for a total colectomy with an ileal pouch-anal anastomosis (IPAA) and a temporary ileostomy. When the client asks the nurse what to expect related to bowel function and care after surgery, the nurse accurately responds with the following: (Select all that apply.) a "The stoma will require that you wear a collective device at all times." b "You will be able to have some control over your bowel movements with the ileostomy." c "The temporary ileostomy is usually maintained for 2-3 months while the bowel heals, then is closed." d "After the stoma heals, you can irrigate your bowel so you won't have to wear a collection pouch." e "Once the ileostomy is closed, you can expect to have 6-bowel movements through the anus daily."

4 (during the convalescent or posticteric stage of hepatitis fatigue and malaise are the most common problems. These symptoms usually disappear within 2-4 months. Fatigue and malaise are not evidence of a secondary infection. Hep A is not treated by drug therapy. It is important that the client continue to balance activity with periods of rest)

A client who is recovering from Hep A has fatigue and malaise. The client asks the nurse, When will my strength return? Which response by the nurse is most appropriate? 1 Your fatigue should be gone by now. We will evaluate you for a secondary infection 2. Your fatigue is an adverse effect of your drug therapy and will disappear when your treatment regimen is complete 3. It is important for you to increase your activity level That will help decrease your fatigue 4. It is normal for you to feel fatigued. The fatigue should go away in the next 2-4 months

1 3 2 4 (The nurse should first assess the client to determine if the tube is obstructing the airway; assessment is done by assessing the airflow. Once obstruction is established, the tube should be deflated and then quickly removed. A set of scissors should always be at bedside to allow for emergency deflation of the ballon. Oxygen via face mask should then be applied once the tube is removed)

A client with a Sengstaken-Blakemore tube has a sudden drop in SpO2 and an increase in respiratory rate to 40 breaths a min. What should the nurse do in order from first to last? 1. Affirm airway obstruction by the tube 2 Remove the tube 3. deflate the tube by cutting with bedside scissors 4. apply oxygen via face mask

2

A client with a history of crohns has concentrated urine, decreased urinary output, dry skin with decreased turgor, hypotension and weak thready pulses. What should the nurse do first? 1. Encourage client to drink at least 1000 mL/day 2. Provide parental rehydration therapy as prescribed 3. Turn and reposition q2h 4. Monitor VS q8

b (Rationale: Hepatic encephalopathy results from cerebral edema, the accumulation of neurotoxins in the blood; therefore, the nurse wants to assess for signs of neurological involvement. Tremoring or flapping of the hands (asterixis) when the arms are extended and wrists dorsiflexed, agitation, confusion, and changes in mentation are common. These clients typically have ascites and edema, so they also experience weight gain, although they may actually be malnourished due to compromised liver functioning and nutrient absorption. Urinary urgency and stomatitis are not related to hepatic encephalopathy. )

A client with advanced cirrhosis has been diagnosed with hepatic encephalopathy. The nurse expects to assess for: a Weight loss b Hand tremors c Urinary urgency d Stomatitis

3 (Edematous tissue is easily traumatized. An alternating air pressure mattress will help decrease pressure on the edematous tissue. ROM exercise is for joint function. When abdominal skin is stretched taut due to ascites, it must be cleaned very carefully, it should not be massaged. Elevation of the LE extremities promotes venous return and decreases swlling)

A client with ascites and peripheral edema is at risk for impaired skin integrity. To prevent skin breakdown the nurse should: 1. institute ROM exercise q4h 2. massage the abd once a shift 3. use an alternating air pressure mattress 4. elevate the LE

2,4,5 (Clients with Chronic C should abstain from alcohol as it can spread cirrhosis and end stage liver disease. Clients should also check with their HCPs before taking any non prescription medications, or herbal supplements. It is also important that clients who are infected with HCV be tested for HIV as clients who have both HIV and HCV have a more rapid disease progression than those with HCV alone. Clients with HCV and N should be instructed to eat 4-5 times a day to help reduce anorexia and N. The client should obtain sufficient rest to manage fatigue)

A client with chronic hepatitis C is experiencing N/V anorexia, and fatigue During the health history, the client states that he is homosexual drinks one to two glasses of wine with dinner, is taking St Johns wort for a bit of depression, and takes acetaminophen for frequent headaches. What should the nurse do? SATA 1. Instruct the client that the wine with meals can be beneficial for cardiovascular health 2. Instruct the client to ask the HCP about taking any other medications as they may interact with medications the client is currently taking 3. Instruct the client to increase the protein in the diet and eat less freq 4. Advise the client of the need for additional testing for HIV 5. Encourage the client to obtain sufficient rest

2 (Spronolactone spares K. therefore the client should be watched for hyperkalemia. Other common adverse effects include abd cramping, D, dizziness, headache and rash. Constipation and dysuria are not common adverse effects. An irreg pulse is not an adverse effect of spironolactone but could develop if serum potassium levels are not closely watched)

A client with cirrhosis begins to develop ascites. Spironolactone is prescribed to treat the ascites. The nurse should monitor the client closely for which drug related adverse effect? 1. constipation 2. hyperkalemia 3. irregular pulse 4. dysuria

d (Rationale For the client with cirrhosis who successfully achieves identified goals and​ outcomes, the nurse should observe improved coagulation studies. Slight​ bruising, disorientation, and elevated liver function tests do not indicate successful achievement of goals.)

A client with cirrhosis is being evaluated for discharge. Which outcome and nursing observation indicate the client is ready for discharge​ home? a Only slightly elevated liver function tests b Easily reoriented to person c Only slight bruising d Improved coagulation studies

1 (The client with cirrhosis can develop hepatic encephalopathy caused by increasing ammonia levels Asterixis, a flapping tremor is a characteristic symptom of increasing ammonia levels. Bacterial action on increased protein in the bowel will increase ammonia levels and cause the encephalopathy to worsen. GI bleeding and protein consumed in the diet increase protein in the intestine and can elevate ammonia levels. Lactulose is given to reduce ammonia formation in the intestine and should not be held since neurological symptoms are worsening. Bilirubin is assoc with jaundice)

A client with cirrhosis is receiving lactulose. The nurse notes the client is more confused and has asterixis. The nurse should: 1. assess for GI bleeding 2. withhold the lactulose 3. increase protein in the diet 4. monitor serum bilirubin levels

1 (Normal serum albumin is administered to reduce ascites. Hypoalbuminemia, a mechanism underlying ascites formation results in decreased colloid osmotic pressure. Administering serum albumin increases the plasma colloid osmotic pressure, which causes fluid to flow from the tissue space into the plasma. Increased urine output is the best indication the albumin is having the desired effect. An increased serum albumin level and increased ease of breathing may indirectly imply that the administration of albumin is effective in relieving the ascites, However it is not as direct an indication as increased urine output and reduced ascites. Anorexia is not affected by the administration of albumin)

A client with cirrhosis who has ascites receives 100 mL of 25% serum albumin IV. Which finding would best indicate that the albumin is having its desired effect? 1. reduced ascites 2. increased serum albumin level 3. decreased anorexia 4. increased ease of breathing

2,3,4 (Baking soda baths can decrease pruritus, keeping nails short and rubbing the area with knuckles can decrease breakdown when scratching. Calamine lotions help relieve itching. Alcohol will increase skin dryness. Sodium in the diet will increase edema and weaken skin integrity)

A client with jaundice has pruritus and areas of irritation from scratching. What measures can the nurse suggest the client use to prevent skin breakdown? SATA 1. Avoid lotions containing calamine 2. Add baking soda to the water in a tub bath 3. keep nails short and clean 4. Rub the skin when it itches with knuckles instead of nails 5. massage skin with alcohol 6. Increase sodium intake in diet

c (Salicylate compounds act by inhibiting prostaglandin synthesis and reducing inflammation. The nurse teaches the client to take the medication with a full glass of water and to increase fluid intake throughout the day. This medication needs to be taken after meals to reduce GI irritation.)

A client with ulcerative colitis has an order to begin salicylate medication to reduce inflammation. The nurse instructs the client to take the medication: A 30 minutes before meals B On an empty stomach C After meals D On arising

1,2,4 (sulfasalazine may cause dizziness, acute intol s/s cramping, acute abd pain, bloody diarrhea, fever, headache, rash, D/C!, med may make urine change color to orange=ok. Take missed doses ASAP unless almost time for next dose)

A client with ulcerative colitis taking sufasalazine. which instructions should the nurse give the client about taking this med at home? select all 1. Drink enough fluids to maintain a urine output of 1200-1500 mL/day 2. D/C therapy if symptoms of acute intolerance develop and notify HCP 3. Stop taking the med if the urine turns orange-yellow 4. avoid activities that require alertness 5. If dose is missed, skip and continue with next dose

d (Rationale Use of the transjugular intrahepatic portosystemic shunt​ (TIPS) relieves portal hypertension and reduces the onset of esophageal varices and ascites. The​ Sengstaken-Blakemore and Minnesota tubes are used for bleeding​ varices, and paracentesis is done to relieve severe ascites that does not respond to diuretic therapy.)

A client with​ end-stage cirrhosis is brought to the emergency department with declining functional status. Which treatment will relieve the client​'s symptoms of portal hypertension and reduce the onset of esophageal varices and​ ascites? ​a Sengstaken-Blakemore tube b Paracentesis c Minnesota tube d Transjugular intrahepatic portosystemic shunt​ (TIPS)

3 (LActulose increases intestinal motility thereby trapping and expelling ammonia in the feces. An increase in the number of bowel movements is expected as an adverse effect. Lactulose does not affect urine output. Any improvements in mental status would be the result of increased ammonia elimination, not an adverse effect of the drug. N/V are not common adverse effects of lactulose)

A clients serum ammonia level is elevated and the HCP prescribes 30 mL of lactulose. Which effect is common for this drug? 1. Increase urine output 2. improved LOC 3 increase bowel movements 4. N/V

Metabolic Alkalosis

A deficit of carbonic acid & a ↓ in hydrogen ion concentration that results from the accumulation of base or from a loss of acid without a comparable loss of base in the body fluids pH ↑ PCO2 normal HCO3 ↑ >Respiraory compensates by hypoventilating to hold onto CO2

b,c,d (Rationale: In the United States, the greatest risk factors for developing cirrhosis and chronic liver disease include high rates of alcohol use and abuse and being of Native American or Hispanic/Latino origin. Rather than higher triglycerides causing cirrhosis, excessive alcohol consumption causes metabolic changes in the liver, which leads to higher triglyceride synthesis. While hepatitis can be contracted from food handlers with the disease, this is not a primary risk factor for developing cirrhosis/chronic liver disease. )

A nurse invited to present to high school adolescents in a biology class about health issues would identify which factors as having the greatest risk for developing cirrhosis and chronic liver disease? (Select all that apply.) a Exposure to food handlers who may or may not be immunized against hepatitis b Being of Native American descent c Being of Hispanic/Latino descent d Excessive alcohol consumption e Having high triglyceride levels

a (Rationale Diuretics are used to reduce fluid retention and ascites. While furosemide​ (Lasix) may be​ used, the drug of choice is spironolactone​ (Aldactone). Neomycin sulfate reduces the number of​ ammonia-forming bacteria in the​ bowel, and oxazepam​ (Serax) is used for acute agitation.)

A nurse is caring for a client with ascites secondary to cirrhosis. Which medication is the treatment of​ choice? a Spironolactone​ (Aldactone) b Oxazepam​ (Serax) c Furosemide​ (Lasix) d Neomycin sulfate

d (Rationale Abdominal​ distention, which is an imbalance of fluid within the portal​ system, might mean ascites in a client with cirrhosis. The vital signs are all within normal limits.)

A nurse is caring for a client with cirrhosis. Which assessment finding warrants immediate​ attention? a Pulse of 60 bpm b Oxygen saturation of​ 92% c Blood pressure of​ 110/72 mmHg d Abdominal distention

a,b,c (Rationale The client with inflammatory bowel disease is at greatest risk for deficient fluid​ volume, diarrhea and constipation. This client does not generally display acute confusion or risk for falls.)

A nurse is caring for a client with inflammatory bowel disease and is planning for the most appropriate client interventions. Which nursing diagnosis best supports the interventions needed for the client with inflammatory bowel​ disease? ​(Select all that​ apply.) a Diarrhea b Constipation c Risk for deficient fluid volume d Acute confusion e Risk for injury

b,c,d,e (Rationale Interventions for a client with inflammatory bowel disease primarily focus on preventing​ infection, monitoring​ weight, maintaining skin​ integrity, promoting nutritional​ balance, and maintaining fluid balance. Encouraging deep breathing and coughing may be an appropriate​ intervention, but this is not a primary focus for this client.)

A nurse is caring for a client with inflammatory bowel disease. When planning care for this​ client, which interventions take primary​ focus? ​(Select all that​ apply.) a Encourage deep breathing and coughing b Monitor weight c Promote nutritional balance d Maintain skin integrity e Maintain fluid balance

b (Rationale: The client with Crohn's disease is likely to have diarrhea with no obvious blood or mucus in the stools. Frothy stools are not characteristic of Crohn's disease. Abdominal pain in Crohn's disease is typically cramping or steady and located in the lower right quadrant or around the umbilicus. A clinical manifestation of ulcerative colitis is cramping in the lower left quadrant relieved by defecation. )

A nurse is caring for the client with Crohn's disease who was admitted last night. Which of the following manifestations would the nurse expect to note for this client? a Cramping in the lower left quadrant with relief with defecation b Diarrhea c Frothy stools d Bloody stools

1,2,3,4,5 (Constipation leads to increased ammonia production, Lactulose is a hyperosmotic laxative that reduces blood ammonia by acidifying the colon contents, which retards diffusion of nonionic ammonia from the colon to the blood while promoting its migration from the blood to the colon. Hepatic encephalopathy is considered a toxic or metabolic condition that causes cerebral edema, it affects a persons coordination and pupil reaction to light and accommodation. Food and fluids high in carbs should be given because the liver is not synthesizing and storing glucose. Because exercise produces ammonia as a by product of metabolism, activity should be limited, not encouraged)

A nurse is developing a care plan for a client with hepatic encephalopathy. Which are goals for the care of this client? SATA 1. PRevent constipation 2. administer lactulose to reduce blood ammonia 3. monitor coordination while walking 4. check the pupil reaction 5. provide food and fluids high in carbs 6. encourage physical activity

a (Rationale The most correct goal or outcome for the client with inflammatory bowel disease is the client recognized the early signs of a flare up. The client should be free from infection with no loss of skin integrity or weight loss.)

A nurse is planning care for a client with inflammatory bowel disease. What goal or outcome is the best choice for this​ client? a Client recognized the early signs of a flare up. ​b Client's symptoms of infection have not worsened. c Client lost less than​ 5% of​ pre-illness body weight. ​d Client's skin excoriation has not worsened.

d (The answer is D. Pancolitis affects all the colon and is a very severe form of ulcerative colon. The patient is at risk for toxic megacolon. In toxic megacolon, the large intestine dilates due to the overwhelming inflammation. The large intestine is unable to function properly and becomes paralyzed. Typical signs and symptoms of toxic megacolon include: abdominal distention, fever, diarrhea, abdominal pain, dehydration, and tachycardia.)

A patient diagnosed with pancolitis is experiencing extreme abdominal distension, pain 10 on 1-10 scale in the abdomen, temperature of 103.6 'F, HR 120, and profuse diarrhea. What complication due you suspect the pain is experiencing? A. Fistulae B. Stricture C. Bowel obstruction D. Toxic megacolon

d (The answer is D. Left-sided colitis (distal colitis) starts in the rectum and goes to the sigmoid and descending colon. Ulcerative proctitis affects the rectum only. Proctosigmoiditis affects the rectum and sigmoid colon. Right-sided colitis is NOT a type of ulcerative colitis.)

A patient is admitted with ulcerative colitis. In the physician's notes, it is stated that the patient's barium enema results showed the patient has colitis that starts in the rectum and extends into the sigmoid and descending colon. As the nurse, you know that this is what type of ulcerative colitis? A. Right-sided colitis B. Proctosigmoiditis C. Ulcerative procotitis D. Left-sided colitis

a (The answer is A. 5-Aminosalicylates (Sulfasalazine) are usually prescribed for mild to moderate cases of ulcerative colitis as first-line treatment. If Aminosalicylates are not working (or the patient is allergic to sulfa) corticosteriods are prescribed. Corticosteriods may be used in combination with immunosupressors. Immunosupressors and immunomodulators are used in severe cases of ulcerative colitis when other medications have not worked.)

A patient is newly diagnosed with mild ulcerative colitis. What type of anti-inflammatory medication is typically prescribed as first-line treatment for this condition? A. 5-Aminosalicylates (Sulfasalazine) B. Immunomodulators (Adalimumab) C. Corticosteroids (Prednisone) D. Immunosupressors (Azathioprine)

d (Rationale: Young adult clients typically have concerns about managing sexual activity with an ileostomy. Validating these concerns and helping these individuals obtain the knowledge they are seeking will them cope more effectively with this significant threat to self-image. The nurse should indicate that she understands this client's concern and then ensure that the two of them can have privacy to discuss sensitive questions. The United Ostomy Association and its local chapters have printed materials available on this topic, but simply handing the patient a brochure is not the best strategy to address the client's needs. )

A young adult client with inflammatory bowel disease has just undergone a temporary ileostomy. He tells the nurse that he is not sure how he can handle any future sexual encounters. What is the best response the nurse might provide? a "Granted, this will be a challenge, but you'll figure it out over time." b "It's a bit early to be thinking about this...let's plan to talk about it on a follow-up visit." c "Here's a pamphlet from the United Ostomy Association that you can read about sex and ostomies." d "I can understand how this can be a real concern. Let's go find a place to talk where we can have privacy."

A stroke patient who primarily uses a wheelchair for mobility has diarrhea with fecal incontinence. What should the nurse assess first? A. Fecal impaction B. Perineal hygiene C. Dietary fiber intake D. Antidiarrheal agent use

A. Fecal impaction Patients with limited mobility are at risk for fecal impactions due to constipation that may lead to liquid stool leaking around the hardened impacted feces, so assessing for fecal impaction is the priority. Perineal hygiene can be assessed at the same time. Assessing the dietary fiber and fluid intake and antidiarrheal agent use will be assessed and considered next.

A patient with a history of peptic ulcer disease has presented to the emergency department reporting severe abdominal pain and has a rigid, boardlike abdomen that prompts the health care team to suspect a perforated ulcer. What intervention should the nurse anticipate? A. Providing IV fluids and inserting a nasogastric (NG) tube B. Administering oral bicarbonate and testing the patient's gastric pH level C. Performing a fecal occult blood test and administering IV calcium gluconate D. Starting parenteral nutrition and placing the patient in a high-Fowler's position

A. Providing IV fluids and inserting a nasogastric (NG) tube A perforated peptic ulcer requires IV replacement of fluid losses and continued gastric aspiration by NG tube. Nothing is given by mouth, and gastric pH testing is not a priority. Calcium gluconate is not a medication directly relevant to the patient's suspected diagnosis, and parenteral nutrition is not a priority in the short term.

A nurse is admitting a client who has been vomiting for 24 hr. Arterial blood gases are obtained. Based on laboratory findings, which of the following conditions should the nurse expect? A. Respiratory acidosis B. Respiratory alkalosis C. Metabolic acidosis D. Metabolic alkalosis

A. Respiratory acidosis B. Respiratory alkalosis C. Metabolic acidosis *D.* *Metabolic alkalosis* Rationale: Excessive vomiting causes a loss of gastric acids & an accumulation of bicarbonate in the blood, resulting in *metabolic alkalosis*

A nurse is assessing a client who has pancreatitis. His arterial blood gases reveal metabolic acidosis. Which of the following is an expected findings? (Select all that apply.) A. Tachycardia B. Hypertension C. Bounding pulses D. Hyperreflexia E. Dysrhythmia F. Tachypnea

A. Tachycardia B. Hypertension *C. Bounding pulses* *D. Hyperreflexia* *E. Dysrhythmia* *F. Tachypnea*

A patient complains of nausea. When administering a dose of metoclopramide (Reglan), the nurse should teach the patient to report which potential adverse effect? A. Tremors B. Constipation C. Double vision D. Numbness in fingers and toes

A. Tremors Extrapyramidal side effects, including tremors and tardive dyskinesias, may occur as a result of metoclopramide (Reglan) administration. Constipation, double vision, and numbness in fingers and toes are not adverse effects of metoclopramide.

Two days following a colectomy for an abdominal mass, a patient reports gas pains and abdominal distention. The nurse plans care for the patient based on the knowledge that the symptoms are occurring as a result of A. impaired peristalsis. B. irritation of the bowel. C. nasogastric suctioning. D. inflammation of the incision site.

A. impaired peristalsis. Until peristalsis returns to normal following anesthesia, the patient may experience slowed gastrointestinal motility leading to gas pains and abdominal distention. Irritation of the bowel, nasogastric suctioning, and inflammation of the surgical site do not cause gas pains or abdominal distention.

A nurses completing a history and physical assessment for a client who has chronic pancreatitis. Which of the following findings should the nurse identify as a likely cause of the client's condition?

Alcohol use

c (With Crohn​ disease, the bowel lumen begins to appear as​ "cobblestones," as fissures and ulcers surround islands of intact tissue over edematous submucosa. The inflammatory lesions are not continuous and often occur as​ "skip" lesions with intervals of​ normal-appearing bowel. Clients with ulcerative colitis have a bowel that appears​ red, edematous, and friable. The inflammation of ulcerative colitis begins at the crypts of​ Lieberkühn in the distal large intestine and rectum.)

Alicia​ Meritt, a​ 22-year-old woman with complaints of fatigue and persistent​ diarrhea, is suspected of having Crohn disease and is scheduled to have a colonoscopy. Which finding would​ Alicia's nurse expect to see from the colonoscopy if Alicia has Crohn​ disease? ​a Red, edematous, and friable tissue b Continuous inflammatory lesions of bowel c Cobblestone appearance of bowel d Inflammation that begins at the crypts of​ Lieberkühn in the distal large intestine and rectum

b,e (Rationale: Essential content with TPN instruction includes teaching about central venous line dressing changes using sterile technique and the need to carefully monitor the client's intake and output. While monitoring for fever is important to identify the onset of a possible infection, an alert adult will able to inform the caregiver if s/he begins to feel warm or has fever. Routine listening for bowel sounds and palpating the abdomen are not aspects of routine home care for the alert adult client on TNP therapy. )

An alert adult client with severe inflammatory bowel disease (IBD) requires short-term total parenteral nutrition (TPN) following discharge home. Which nursing actions are particularly important when teaching this client and family members about TPN? (Select all that apply.) a Periodically palpate the abdomen for tenderness. b Carefully document the client's intake and output. c Listen for bowel sounds regularly using the stethoscope provided to the family. d Take the client's temperature every 4 hours whether or not the client feels warm to the touch. e Make sure the client and family members use sterile technique when changing the central venous catheter dressing.

What other priority information may the nurse assess in order to care for a patient presenting with abdominal pain and watery, bloody stools? A. Psychosocial history, family history, vital signs, previous bowel history B. Vaccine history, medication history, heart rate C. Orientation status, ability to participate in sports, blood pressure D. Cranial nerves, sensation, skin assessment

Answer: A Rationale: All of these are important assessments for the nurse. However, IBD often show a familial tendency and it must be noted whether there is any family history. Psychosocial history is important as IBD patients often have many psychosocial issues and symptoms are exacerbated during stress. Vital signs are important as during the inflammatory process, the patient if often febrile. The patient's previous bowel history is used to determine the onset. A is the correct answer.

The patient with which condition is at the highest risk for fistula formations? A. Crohn's disease B. Ulcerative colitis C. Diverticulitis D. Irritable bowel disease

Answer: A Rationale: Because Crohn's disease is transmural, affecting all layers of the bowel, it can develop sinus tracts leading to fistula formation.

The nurse is caring for a postoperative patient who has had surgery for a ventral hernia repair. The nurse is reinforcing postoperative care teaching and asks the patient to verbalize what was taught to him. Which statement requires the nurse to clarify the patient's perception of the teaching? A. "I should turn, cough, and deep breathe every 2 hours." B. "I should avoid heavy lifting until my provider tells me that I can lift again." C. "I can use ice packs and scrotal support for scrotal swelling." D. "I will observe my incision for redness, heat, swelling, and drainage. I will report these signs to my provider at once."

Answer: A Rationale: Coughing will cause undue pressure on the surgical site and possibly lead to recurrence of the hernia. If coughing is necessary, the surgical site should be splinted with pillows to prevent pressure on the site.

What clinical manifestations are consistent with a diagnosis of ulcerative colitis? (Select all that apply.) A. Bloody stools B. Constipation C. Belching D. Chest pain E. Dysphagia

Answer: A Rationale: The stool of patients with ulcerative colitis is usually watery diarrhea, with blood in the stool. Constipation is associated with disorders that slow GI motility, such as opioids. Belching is usually associated with disorders of the upper gastrointesti- nal system, and chest pain is associated with GERD and hiatal hernias.

The nurse recognizes which findings as diagnostic for IBS? A. Rome III and/or Manning criteria B. CT scan of the abdomen shows inflammatory process C. Blood urea nitrogen and creatinine are elevated D. Patient has abdominal pain and a psychiatric diagnosis

Answer: A Rationale: There are two tools used for the diagnosis of IBS: the Rome III Diagnostic Criteria for Functional Gastrointestinal Disorders and the Manning criteria for the diagnosis of irritable bowel syndrome. The other tests may be used, but are not as specific.

The nurse correlates strangulated hernias to which finding? A. Impede blood flow of the intestines B. Result from pressure on an old surgical incision C. Are more common in infants D. A hernia in which contents can be placed back into place

Answer: A Rationale: When the intestine protruding through an abnormal opening cannot be placed easily back into the abdominal cavity manually or by lying down, it is known as irreducible or incarcerated. If the blood sup- ply is obstructed, it is then known as a strangulated hernia, and the patient may present with symptoms of an intestinal obstruction. This is a medical emergency, and the patient must be prepared for surgery immedi- ately to prevent gangrene from developing

Which interventions are considered conservative treatments for hemorrhoids? (Select all that apply.) A. Hemorrhoid creams and ointments B. Sitz baths C. Over-the-counter analgesics D. Cryosurgery E. Rubber-band ligation

Answer: A, B, and C Rationale: Treatment of hemorrhoids is usually conser- vative and involves relief of symptoms and associated pain. Cold packs and sitz baths (warm water baths covering the hips and buttocks) three or four times a day reduce some swelling and pain. There are a num- ber of over-the-counter preparations available in creams and suppositories used to treat hemorrhoids. Rubber band ligation and cryosurgery are surgical in- terventions for higher grade hemorrhoids.

25. What two organs in the body serve as a compensatory function to maintain acid base balance? A. Kidneys and Lungs B. Lungs and Spleen C. Heart and Liver D. Gallbladder and Appendix

Answer: A. Kidneys and Lungs The carbonic acid concentration is controlled by the amount of carbon dioxide excreted by the lungs. The bicarbonate concentration is controlled by the kidneys, which selectively retain or excrete bicarbonate in response to the body's needs.

23. Decreased plasma bicarbonate (HCǑ-) A. Metabolic Acidosis B. Respiratory Alkalosis C. Metabolic Alkalosis D. Respiratory Acidosis

Answer: A. Metabolic Acidosis The body compensates by using body fat for energy, producing abnormal amounts of ketone bodies. In an effort to neutralize the ketones and maintain the acid-base balance of the body, plasma bicarbonate is exhausted. This condition can develop in anyone who does not eat an adequate diet and whose body fat must be burned for energy. Symptoms include headache and mental dullness.

37. pH 7.45, PaCÓ 50, HCǑ- 30 A. Metabolic Alkalosis, Fully Compensated B. Respiratory Alkalosis, Fully Compensated C. Metabolic Alkalosis, Partially Compensated D. Respiratory Acidosis, Partially Compensated

Answer: A. Metabolic Alkalosis, Fully Compensated

38. pH 7.6, PaCÓ 53, HCǑ- 38 A. Metabolic Alkalosis, Partially Compensated B. Metabolic Alkalosis, Fully Compensated C. Respiratory Acidosis, Partially Compensated D. Respiratory Alkalosis, Fully Compensated

Answer: A. Metabolic Alkalosis, Partially Compensated

26. Arterial blood gas (ABG) measurement will give the information needed to determine if the primary disturbance of acid-base balance is respiratory or metabolic in nature. A. True B. False C. Both Carbonic Acid Excess and Deficit Only D. Both Bicarbonate Excess and Deficit Only

Answer: A. True ABG's are blood tests that are useful in identifying the cause and extent of the acid-base disturbance and in guiding and monitoring treatment.

29. The human body functions optimally in a state of homeostasis. A. True B. False C. Maybe D. Homeostasis has nothing to do with metabolic balance.

Answer: A. True The maintenance of acid-base balance, which in one part of homeostasis, is evidenced by an arterial plasma pH value of 7.35-7.45. Many mechanisms in the body work together to achieve and maintain this delicate narrow range of pH that is essential for normal cell function.

28. Alkalosis is characterized by overexcitement of the nervous system. A. True B. False C. The major effect of Alkalosis is a depression of the central nervous system. D. Both Acidosis and Alkalosis result in overexcitement of the central nervous system.

Answer: A. True The muscles may go into a state of tetany and convulsions.

A patient has just been brought to the emergency department by emergency medical services after a motor vehicle accident. What is the first thing the nurse should do? A. Ask the patient if he or she is in pain B. Mental status examination and vital signs C. Ask the patient to move all extremities D. Order laboratory tests

Answer: B

The nurse is caring for a patient with colorectal cancer who just had a total colectomy with placement of a permanent ileostomy. Which nursing diagnosis is a priority for the immediate postoperative period? A. Disturbed body image B. Acute pain C. Potential for infection D. Knowledge deficit

Answer: B Rationale: Adequate pain management is necessary for the patient to return to an optimal level of functioning and to prevent postoperative complications, such as atelectasis. The other nursing diagnoses are relevant, but acute pain is the priority in the immediate post- operative period.

The nurse is caring for a patient in the emergency department with abdominal pain, fever, nausea, and vomiting. The patient is suspected of having appendicitis. What assessments may the provider perform to confirm diagnosis? A. Flat-plate x-ray of the abdomen, chemistry panel B. CT scan, complete blood count (CBC), abdominal assessment for rebound tenderness C. Give patient a laxative to see if symptoms improve D. Colonoscopy, esophagogastroduodenoscopy (EGD), and endoscopic retrograde cholangiopancreatogram (ERCP)

Answer: B Rationale: In patients with appendicitis, as the inflam- matory process proceeds, pain is shifted to the right lower quadrant of the abdomen and becomes more severe and steady in the area of McBurney's point. When applying and releasing pressure to this area, if the patient notes increased pain when pressure is released, this is called rebound tenderness and is another indication of appendicitis. An abdominal CT may indicate inflammation or enlargement of the appendix. While a flat plate x-ray of the abdomen and serum chemistries may be ordered, they are diagnosti- cally definitive. Laxatives are contraindicated due to the risk of perforation. There is no indication for colonoscopy, EGD, or ERCP in the patient with appendicitis.

A patient has just been brought to the emergency department by emergency medical services after a motor vehicle accident. What is the first thing the nurse should do? A. Ask the patient if he or she is in pain. B. Mental status examination and vital signs. C. Ask the patient to move all extremities. D. Order laboratory tests.

Answer: B Rationale: Once the trauma patient's airway, breathing, and circulation have been thoroughly assessed, then the patient needs to be evaluated for signs of hemorrhage, shock, and peritonitis. Vital signs and mental status examination are priority nursing assessments. The clinical manifestations vary widely according to the organ of injury, and assessment includes the presence, location, and quality of any pain experienced by the patient.

A patient is admitted to the hospital for treatment for diverticulitis. The nurse recognizes which interventions appropriate for this patient? A. High-fiber diet, ambulate frequently, IV fluids, pain medications B. Antibiotics, IV fluids, NPO, NG tube, pain medications C. Laxatives, enemas, diet, pain medications D. Surgery with follow-up physical therapy

Answer: B Rationale: Patients with diverticulitis who are hospital- ized are treated with broad-spectrum antibiotics, IV fluids, and placed NPO to allow the bowel to rest. The patient may have a nasogastric (NG) tube for bowel decompression. Laxatives and enemas should be avoided because they increase intestinal motility. Pain medications may be given as needed, and opiates are frequently needed. If patients develop complications such as perforation, hemorrhage, obstruction, or abscess, they may require surgery to remove the diseased portion of the colon.

Which information does the nurse include in the teaching to Jack related to his diagnosis of ulcerative colitis? A. "Decrease fluid intake to decrease diarrhea." B. "Spread out your meals to six times per day." C. "Avoid foods high in potassium." D. "Increase your intake of simple sugars for energy."

Answer: B Rationale: The patient is encouraged to eat small, frequent meals in order to decrease gastric motility and decrease diarrhea. The patient is at risk for fluid volume deficit, so fluids are not decreased. Loss of potassium may be increased with diarrhea and may require supplementation. Simple sugars increase gastric motility and can exacerbate diarrhea.

A nurse cares for a client who is recovering from a hemorrhoidectomy. The client states, "I need to have a bowel movement." The nurse is aware this is the first bowel movement. Which priority safety action should the nurse take? A. Obtain a bedside commode for the client to use B. Stay with the client while providing privacy C. Make sure the call light is in reach to signal completion D. Gather supplies to collect a stool sample for the laboratory

Answer: B The first bowel movement after surgery may be painful, and the patient needs to take an analgesic prior. Fainting has occurred during bowel movements early after surgery because of the intensity of the pain along with vagal stimulation. Because of the fainting risk, staying with the patient is an important safety measure.

Where is the primary location of nutrient absorption in the gastrointestinal system? A. Stomach B. Small intestine C. Large intestine D. Pancreas

Answer: B The small intestine's primary function is digestion and absorption of nutrients across the intestinal wall into the circulation.

30. Acids have no hydrogen ions and are able to bind in a solution. A. True B. False C. Acid is a substance that is not capable of donating hydrogen ions. D. Acids and bases have nothing to do with hydrogen ions.

Answer: B. False Acids are substances having one or more hydrogen ions that can be liberated into a solution. Bases are substances that can bind hydrogen ions in a solution.

27. The major effect of acidosis is overexcitement of the central nervous system. A. True B. False C. Maybe D. Both Acidosis and Alkalosis result in overexcitement of the central nervous system

Answer: B. False The major effect is a depression of the central nervous system, as evidenced by disorientation followed by coma.

35. pH 7.34, PaCÓ 24, HCǑ- 20 A. Respiratory Acidosis, Partially Compensated B. Metabolic Acidosis, Partially Compensated C. Metabolic Acidosis, Uncompensated D. Metabolic Alkalosis, Partially Compensated

Answer: B. Metabolic Acidosis, Partially Compensated

21. Low plasma PaCÓ A. Metabolic Acidosis B. Respiratory Alkalosis C. Metabolic Alkalosis D. Respiratory Acidosis

Answer: B. Respiratory Alkalosis Excessive pulmonary ventilation decreases hydrogen ion concentration and thus causes respiratory alkalosis. It can become dangerous when it leads to cardiac dysrhythmias caused partly by a decrease in serum potassium levels.

36. pH 7.64, PaCÓ 25, HCǑ- 19 A. Respiratory Acidosis, Uncompensated B. Respiratory Alkalosis, Partially Compensated C. Respiratory Alkalosis, Uncompensated D. Metabolic Alkalosis, Partially Compensated

Answer: B. Respiratory Alkalosis, Partially Compensated

33. pH 7.55, PaCÓ 25, HCǑ- 22 A. Respiratory Acidosis, Partially Compensated B. Respiratory Alkalosis, Uncompensated C. Metabolic Alkalosis, Partially Compensated D. Metabolic Acidosis, Uncompensated

Answer: B. Respiratory Alkalosis, Uncompensated

A patient presents to the ED with complaints of abdominal pain and watery, bloody stools. What diagnostic tests does the nurse expect to be ordered on the basis of these symptoms? A. CBC, MRI, electrolytes, stool analysis B. CT scan, MRI, chemistry panel, ERCP C. Colonoscopy, CBC, wireless capsule endoscopy, upper GI endoscopy D. BUN, creatinine, ultrasound, chest x-ray

Answer: C Rationale: Bloody stools are more common with ulcerative colitis. Therefore a CBC should be ordered. Wireless capsule endoscopy evaluates the portion between what can be seen with upper GI endoscopy and the colonoscopy. Ulcerative colitis affects only the large intestine but Crohn's disease affects anywhere from mouth to anus and must be ruled out before a surgical decision can be made. Blood, mucus, and pus are common with ulcerative colitis but not with Crohn's.

An 80-year-old patient is admitted to the hospital for diverticulitis. The family states that the family member isn't acting normally. The patient does not have specific complaints. The nurse correlates this data to which characteristics of older adults? A. They typically complain of a lot of pain with diverticulitis. B. They often have referred pain to another site. C. They may exhibit a change in mental status before any other symptoms occur. D. They will be having other symptoms such as nausea and vomiting.

Answer: C Rationale: The first sign that may appear in the elderly is a change in mental status. Baseline temper- ature is often decreased from normal in the older adult. Therefore, one of the most common signs of infection may not be apparent in the older adult and the patient may present with increased confusion, falling, and anorexia.

The nurse is caring for a postoperative patient who has had surgery for hemorrhoids. What information should be taught to the patient in the postoperative period? A. Apply dry heat to the surgical area as soon as you return home. B. Maintain a low-fiber diet. C. Contact the provider if urinary retention occurs. D. No bowel movement is expected for 3 days.

Answer: C Rationale: Urinary retention may occur because of rectal spasms and pain. The nurse needs to make sure the patient has urinated prior to leaving the outpatient facility. Patients also need to be taught to call the physician immediately if they are unable to urinate once they return home.

Reabsorption of the majority of the water from the gastrointestinal tract occurs in the: A. Liver B. Small intestine C. Large intestine D. Rectum

Answer: C The large intestine's primary function is absorption of water. There is also some absorption of electrolytes although not to the extent of absorption in the small intestine.

What is the most distal section of the small intestine? A. Cecum B. Duodenum C. Ileum D. Jejunum

Answer: C There are three sections to the small intestine: • Duodenum—attaches to the pylorus and is approximately 10 in. (25 cm) long • Jejunum—approximately 8 ft (2.5 m) • Ileum—approximately 12 ft (3.5 m)

24. Increased plasma bicarbonate (HCǑ-) A. Metabolic Acidosis B. Respiratory Alkalosis C. Metabolic Alkalosis D. Respiratory Acidosis

Answer: C. Metabolic Alkalosis In metabolic alkalosis, breathing becomes depressed in an effort to conserve carbon dioxide for combination with water in the blood to raise the blood level of carbonic acid. Symptoms include confusion, dizziness, numbness or tingling of fingers or toes.

40. pH 7.4, PaCÓ 59, HCǑ- 35 A. Respiratory Acidosis, Uncompensated B. Metabolic Alkalosis, Uncompensated C. Respiratory Acidosis, Fully Compensated D. Metabolic Alkalosis, Partially Compensated

Answer: C. Respiratory Acidosis, Fully Compensated

The nurse is caring for a patient who has had surgery for colon cancer and has a permanent colostomy. Which assessment must the nurse report to the provider immediately? A. The stoma is reddish pink and moist. B. The stoma is flat against the skin. C. The stoma has not expelled any flatus or stool in the first 24 hours. D. The stoma is dark and bluish in appearance.

Answer: D Rationale: If the stoma begins to show signs of ischemia (dark red, purplish, or black color) or unusual bleeding, the physician should be notified immediately. This is a sign that there is little or no blood flow to the stoma.

On the basis of the presenting symptoms of abdominal pain with watery, bloody stools, what will the nurse consider a priority to notify the provider? A. Height and weight B. Temperature C. Blood pressure D. Hemoglobin and hematocrit

Answer: D Rationale: The patient's hemoglobin is 7.6 and hematocrit of 21.2. This probably accounts for the pulse of 133. His weight is on the low end of normal for his height but not a priority of care. His temperature is 100.5 F (38 C) due to the inflammatory process and the body's immune system at work. The blood pressure is of no concern. The nurse should expect the physician to address the potential of Jack to receive blood products for his hemoglobin and hematocrit.

32. pH 7.39, PaCÓ 44, HCǑ- 26 A. Respiratory Acidosis B. Metabolic Acidosis C. Respiratory Alkalosis D. Normal

Answer: D. Normal

22. High plasma PaCÓ A. Metabolic Acidosis B. Respiratory Alkalosis C. Metabolic Alkalosis D. Respiratory Acidosis

Answer: D. Respiratory Acidosis An excess of carbon dioxide (hypercapnia) can cause carbon dioxide narcosis. In this condition, carbon dioxide levels are so high that they no longer stimulate respirations but depress them.

34. pH 7.17, PaCÓ 48, HCǑ- 36 A. Respiratory Acidosis, Uncompensated B. Metabolic Acidosis, Partially Compensated C. Respiratory Alkalosis, Partially Compensated D. Respiratory Acidosis, Partially Compensated

Answer: D. Respiratory Acidosis, Partially Compensated

31. pH 7.57, PaCÓ 22, HCǑ- 17 A. Respiratory Acidosis, Partially Compensated B. Respiratory Alkalosis, Uncompensated C. Metabolic Acidosis, Partially Compensated D. Respiratory Alkalosis, Partially Compensated

Answer: D. Respiratory Alkalosis, Partially Compensated

39. pH 7.5, PaCÓ 19, HCǑ- 22 A. Respiratory Alkalosis, Partially Compensated B. Metabolic Alkalosis, Partially Compensated C. Respiratory Acidosis, Uncompensated D. Respiratory Alkalosis, Uncompensated

Answer: D. Respiratory Alkalosis, Uncompensated

Of the following three patients, which one would the nurse see first: A. 78-year-old female with diverticulitis. The patient had just arrived from the emergency department at the end of the previous shift. In report, the previous nurse states that the patient had arrived 1 hour prior to shift change. In the emergency department, the patient had abdominal x-rays and CT scan completed. A nasogastric tube is in place and the patient is NPO. Pain medication has been given although the patient continues to complain of generalized abdominal pain and is holding their abdomen. The family states that her mental status has changed with increased confusion over the past 48 hours. B. 22-year-old female with complications associated with Crohn's disease. She has a PCA pump for pain management that the patient says is not relieving her pain. She has a medication ordered for breakthrough pain in which she requests every 2 hours. It is now shift change and she is requesting her medication for breakthrough pain. She has called the nurses' station three times in the last 15 minutes. C. 60-year-old male with colon cancer who had surgery three days ago. The previous nurse reports that vital signs are within normal limits, patient slept most of the night with no complaints of pain. He has had training from the Wound Ostomy Continence Nurse (WOCN) on how to care for his ostomy and is emptying it on his own. He ambulates the halls independently.

Answer: Patient A Rationale: Although the patient with Crohn's is calling the nurses' station frequently requesting pain medica- tion, be aware that the patient with diverticulitis may be experiencing a ruptured diverticulum with peritonitis. This patient should be assessed first as the physician needs to be notified of their pain unrelieved from medication, guarding behavior, and change in mental status. The patient requesting pain medication can be delegated to another nurse to take care of while you are assessing a more urgent situation. The patient with colon cancer appears to be the most stable and can be seen last.

A patient is admitted to the hospital for treatment for diverticulitis. The nurse recognizes which interventions appropriate for this patient?

Antibiotics, IV fluids, NPO, NG tube, pain medications

A 61-year-old patient with suspected bowel obstruction had a nasogastric tube inserted at 4:00 AM. The nurse shares in the morning report that the day shift staff should check the tube for patency at what times? A. 7:00 AM, 10:00 AM, and 1:00 PM B. 8:00 AM, 12:00 PM, and 4:00 PM C. 9:00 AM and 3:00 PM D. 9:00 AM, 12:00 PM, and 3:00 PM

B. 8:00 AM, 12:00 PM, and 4:00 PM A nasogastric tube should be checked for patency routinely at 4-hour intervals. Thus if the tube were inserted at 4:00 AM, it would be due to be checked at 8:00 AM, 12:00 PM, and 4:00 PM.

After administering a dose of promethazine (Phenergan) to a patient with nausea and vomiting, what common temporary adverse effect of the medication does the nurse explain may be experienced? A. Tinnitus B. Drowsiness C. Reduced hearing D. Sensation of falling

B. Drowsiness Although being given to this patient as an antiemetic, promethazine also has sedative and amnesic properties. For this reason, the patient is likely to experience drowsiness as an adverse effect of the medication. Tinnitus, reduced hearing, and loss of balance are not side effects of promethazine.

The patient who is admitted with a diagnosis of diverticulitis and a history of irritable bowel disease and gastroesophageal reflux disease (GERD) has received a dose of Mylanta 30 mL PO. The nurse should evaluate its effectiveness by questioning the patient as to whether which symptom has been resolved? A. Diarrhea B. Heartburn C. Constipation D. Lower abdominal pain

B. Heartburn Mylanta is an antacid that contains both aluminum and magnesium. It is indicated for the relief of GI discomfort, such as heartburn associated with GERD. Mylanta can cause both diarrhea and constipation as a side effect. Mylanta does not affect lower abdominal pain.

The nurse is planning care for a 68-year-old patient with an abdominal mass and suspected bowel obstruction. Which factor in the patient's history increases the patient's risk for colorectal cancer? A. Osteoarthritis B. History of colorectal polyps C. History of lactose intolerance D. Use of herbs as dietary supplements

B. History of colorectal polyps A history of colorectal polyps places this patient at risk for colorectal cancer. This tissue can degenerate over time and become malignant. Osteoarthritis, lactose intolerance, and the use of herbs do not pose additional risk to the patient.

What information would have the highest priority to be included in preoperative teaching for a 68-year-old patient scheduled for a colectomy? A. How to care for the wound B. How to deep breathe and cough C. The location and care of drains after surgery D. Which medications will be used during surgery

B. How to deep breathe and cough Because anesthesia, an abdominal incision, and pain can impair the patient's respiratory status in the postoperative period, it is of high priority to teach the patient to cough and deep breathe. Otherwise, the patient could develop atelectasis and pneumonia, which would delay early recovery from surgery and hospital discharge. Care for the wound and location and care of the drains will be briefly discussed preoperatively, but done again with higher priority after surgery. Knowing which drugs will be used during surgery may not be meaningful to the patient and should be reviewed with the patient by the anesthesiologist.

The nurse should administer an as-needed dose of magnesium hydroxide (MOM) after noting what information while reviewing a patient's medical record? A. Abdominal pain and bloating B. No bowel movement for 3 days C. A decrease in appetite by 50% over 24 hours D. Muscle tremors and other signs of hypomagnesemia

B. No bowel movement for 3 days MOM is an osmotic laxative that produces a soft, semisolid stool usually within 15 minutes to 3 hours. This medication would benefit the patient who has not had a bowel movement for 3 days. MOM would not be given for abdominal pain and bloating, decreased appetite, or signs of hypomagnesemia.

Which clinical manifestations of inflammatory bowel disease are common to both patients with ulcerative colitis (UC) and Crohn's disease (select all that apply)? A. Restricted to rectum B. Strictures are common. C. Bloody, diarrhea stools D. Cramping abdominal pain E. Lesions penetrate intestine.

C, D. Clinical manifestations of UC and Crohn's disease include bloody diarrhea, cramping abdominal pain, and nutritional disorders. Intestinal lesions associated with UC are usually restricted to the rectum before moving into the colon. Lesions that penetrate the intestine or cause strictures are characteristic of Crohn's disease.

After the nurse teaches a patient with gastroesophageal reflux disease (GERD) about recommended dietary modifications, which statement by the patient indicates that the teaching has been effective? A. "I can have a glass of low-fat milk at bedtime." B. "I will have to eliminate all spicy foods from my diet." C. "I will have to use herbal teas instead of caffeinated drinks." D. "I should keep something in my stomach all the time to neutralize the excess acids."

C. "I will have to use herbal teas instead of caffeinated drinks." Rationale: Patients with gastroesophageal reflux disease should avoid foods (such as tea and coffee) that decrease lower esophageal pressure. Patients should also avoid milk, especially at bedtime, as it increases gastric acid secretion. Patients may eat spicy foods, unless these foods cause reflux. Small, frequent meals help prevent overdistention of the stomach, but patients should avoid late evening meals and nocturnal snacking.

The patient with chronic gastritis is being put on a combination of medications to eradicate H. pylori. Which drugs does the nurse know will probably be used for this patient? A. Antibiotic(s), antacid, and corticosteroid B. Antibiotic(s), aspirin, and antiulcer/protectant C. Antibiotic(s), proton pump inhibitor, and bismuth D. Antibiotic(s) and nonsteroidal antiinflammatory drugs (NSAIDs)

C. Antibiotic(s), proton pump inhibitor, and bismuth To eradicate H. pylori, a combination of antibiotics, a proton pump inhibitor, and possibly bismuth (for quadruple therapy) will be used. Corticosteroids, aspirin, and NSAIDs are drugs that can cause gastritis and do not affect H. pylori.

The nurse determines that a patient has experienced the beneficial effects of therapy with famotidine (Pepcid) when which symptom is relieved? A. Nausea B. Belching C. Epigastric pain D. Difficulty swallowing

C. Epigastric pain Famotidine is an H2-receptor antagonist that inhibits parietal cell output of HCl acid and minimizes damage to gastric mucosa related to hyperacidity, thus relieving epigastric pain. Famotidine is not indicated for nausea, belching, and dysphagia.

A female patient has a sliding hiatal hernia. What nursing interventions will prevent the symptoms of heartburn and dyspepsia that she is experiencing? A. Keep the patient NPO. B. Put the bed in the Trendelenberg position. C. Have the patient eat 4 to 6 smaller meals each day. D. Give various antacids to determine which one works for the patient.

C. Have the patient eat 4 to 6 smaller meals each day. Eating smaller meals during the day will decrease the gastric pressure and the symptoms of hiatal hernia. Keeping the patient NPO or in a Trendelenberg position are not safe or realistic for a long period of time for any patient. Varying antacids will only be done with the care provider's prescription, so this is not a nursing intervention.

Following administration of a dose of metoclopramide (Reglan) to the patient, the nurse determines that the medication has been effective when what is noted? A. Decreased blood pressure B. Absence of muscle tremors C. Relief of nausea and vomiting D. No further episodes of diarrhea

C. Relief of nausea and vomiting Metoclopramide is classified as a prokinetic and antiemetic medication. If it is effective, the patient's nausea and vomiting should resolve. Metoclopramide does not affect blood pressure, muscle tremors, or diarrhea.

Following bowel resection, a patient has a nasogastric (NG) tube to suction, but complains of nausea and abdominal distention. The nurse irrigates the tube as necessary as ordered, but the irrigating fluid does not return. What should be the priority action by the nurse? A. Notify the physician. B. Auscultate for bowel sounds. C. Reposition the tube and check for placement. D. Remove the tube and replace it with a new one.

C. Reposition the tube and check for placement. The tube may be resting against the stomach wall. The first action by the nurse (since this is intestinal surgery and not gastric surgery) is to reposition the tube and check it again for placement. The physician does not need to be notified unless the tube function cannot be restored by the nurse. The patient does not have bowel sounds, which is why the NG tube is in place. The NG tube would not be removed and replaced unless it was no longer in the stomach or the obstruction of the tube could not be relieved.

1 (pyrosis is heartburn and is expected in a client dx with GERD. The new grad can care for this client and administer an antacid.)

CS 1 Ms Kathy is making assignments. Which client should be assigned to the graduate nurse on the unit for 1 month? 1. The client dx with lower esophageal dysfunction who is complaining of pyrosis 2. The client who had an endoscopy this morning with absent bowel sounds 3. The client with gastroesophageal reflux disease who has bilateral wheezing 4. The client who is 1 day post op open cholecystectomy and refuses to deep breath

3 (This client should be placed on the right side, same side as liver, to prevent hemorrhaging after liver biopsy)

CS 7 Ms Kathy observes the UAP turning the client who has just had a liver biopsy to the supine position. Which action should Ms Kathy implement first? 1. Tell the UAP to keep the client on bed rest for 2 hrs 2. Praise the UAP for placing the client in the supine position 3. Instruct the UAP to place the client on the Right side 4. Complete an incident report on the UAPs behavior

2 (Ms Kathy must first have the nurse complete an adverse occurrence report so there is written documentation concerning the situation. Then Ms Kathy should notify the infection control nurse who will arrange for post-exposure prophylaxis and determine if the client has hepatitis.)

CS 8 One of the primary nurses tells Ms Kathy she stuck herself in the finger with a used needle and cleaned the site with soap and water. Which intervention should Ms. Kathy implement first? 1.. Notify the infection control nurse 2. Complete an adverse occurrence report 3. Request post exposure prophylaxis 4. Check the hepatitis status of the client

3 (The client with pruritis is stable and the UAP can assist with showering and am care, therefore this task can be delegated. WRONG: #1 The client with an inflated Sengstaken-Blakemore tube has acute esphageal varices bleeding and is NOT stable, therefore this task cannot be delegated. )

CS 9 Which nursing task is most appropriate to delegate to the UAP? 1. Bathe the client with liver failure who has a Sengstaken-Blakemore tube inflated 2. Teach the client with an open cholesectomy to splint the incision when coughing 3. Assist the client with pruritis to the bathroom for shower and am care 4. Tell the UAP to assist the nurse performing a paracentesis on the client with liver failure

The nurse is caring for a patient in the emergency department with abdominal pain, fever, nausea, and vomiting. The patient is suspected of having appendicitis. What assessments may the provider perform to confirm diagnosis?

CT scan, complete blood count (CBC), abdominal assessment for rebound tenderness

Respiratory Acidosis - Risk factors/Causes

Caused by primary defects in the function of the lungs or changes in normal respiratory patterns Any condition that causes an obstruction of the airway or depresses the respiratory system can cause respiratory acidosis >Airway obstruction >Asthma >Atelectasis >Brain trauma >COPD >Hypoventilation >Overdose >Pneumonia >Pneumothorax >Pulmonary edema >Pulmonary emboli >Respiratory depression

A nurse is caring for a client who is 2 days postoperative following a gastric bypass. The nurse notes that bowel sounds are present. Which of the following foods should the nurse provide at the initial feeding?

Clear liquids

4 (Hep B is considered a STD. and students should observe safe sex. Poor sanitary conditions in underdeveloped countries relate to the spread of Hep A and E. Focusing on routes of transmission and avoidance of infection can prevent the spread of hep. There is no vaccine for Hep D.)

College freshmen are participating in a study abroad program. When teaching them about hepatitis B the nurse should instruct the students need for: 1. water sanitation 2. Single dorm rooms 3. Vaccination for hep D 4. safe sex practices

The nurse is caring for a postoperative patient with a colostomy. The nurse is preparing to administer a dose of famotidine (Pepcid) when the patient asks why the medication was ordered since the patient does not have a history of heartburn or gastroesophageal reflux disease (GERD). What response by the nurse would be the most appropriate? A. "This will prevent air from accumulating in the stomach, causing gas pains." B. "This will prevent the heartburn that occurs as a side effect of general anesthesia." C. "The stress of surgery is likely to cause stomach bleeding if you do not receive it." D. "This will reduce the amount of HCl in the stomach until the nasogastric tube is removed and you can eat a regular diet again."

D. "This will reduce the amount of HCl in the stomach until the nasogastric tube is removed and you can eat a regular diet again." Famotidine is an H2-receptor antagonist that inhibits gastric HCl secretion and thus minimizes damage to gastric mucosa while the patient is not eating a regular diet after surgery. Famotidine does not prevent air from accumulating in the stomach or stop the stomach from bleeding. Heartburn is not a side effect of general anesthesia.

The nurse is caring for a 68-year-old patient admitted with abdominal pain, nausea, and vomiting. The patient has an abdominal mass, and a bowel obstruction is suspected. The nurse auscultating the abdomen listens for which type of bowel sounds that are consistent with the patient's clinical picture? A. Low-pitched and rumbling above the area of obstruction B. High-pitched and hypoactive below the area of obstruction C. Low-pitched and hyperactive below the area of obstruction D. High-pitched and hyperactive above the area of obstruction

D. High-pitched and hyperactive above the area of obstruction Early in intestinal obstruction, the patient's bowel sounds are hyperactive and high-pitched, sometimes referred to as "tinkling" above the level of the obstruction. This occurs because peristaltic action increases to "push past" the area of obstruction. As the obstruction becomes complete, bowel sounds decrease and finally become absent.

The nurse is preparing to administer a dose of bisacodyl (Dulcolax). In explaining the medication to the patient, the nurse would explain that it acts in what way? A. Increases bulk in the stool B. Lubricates the intestinal tract to soften feces C. Increases fluid retention in the intestinal tract D. Increases peristalsis by stimulating nerves in the colon wall

D. Increases peristalsis by stimulating nerves in the colon wall Bisacodyl is a stimulant laxative that aids in producing a bowel movement by irritating the colon wall and stimulating enteric nerves. It is available in oral and suppository forms. Fiber and bulk forming drugs increase bulk in the stool; water and stool softeners soften feces, and saline and osmotic solutions cause fluid retention in the intestinal tract.

The nurse would question the use of which cathartic agent in a patient with renal insufficiency? A. Bisacodyl (Dulcolax) B. Lubiprostone (Amitiza) C. Cascara sagrada (Senekot) D. Magnesium hydroxide (Milk of Magnesia)

D. Magnesium hydroxide (Milk of Magnesia) Milk of Magnesia may cause hypermagnesemia in patients with renal insufficiency. The nurse should question this order with the health care provider. Bisacodyl, lubiprostone, and cascara sagrada are safe to use in patients with renal insufficiency as long as the patient is not currently dehydrated.

A patient reports having a dry mouth and asks for something to drink. The nurse recognizes that this symptom can most likely be attributed to a common adverse effect of which medication that the patient is taking? A. Digoxin (Lanoxin) B. Cefotetan (Cefotan) C. Famotidine (Pepcid) D. Promethazine (Phenergan)

D. Promethazine (Phenergan) A common adverse effect of promethazine, an antihistamine/antiemetic agent, is dry mouth; another is blurred vision. Common side effects of digoxin are yellow halos and bradycardia. Common side effects of cefotetan are nausea, vomiting, stomach pain, and diarrhea. Common side effects of famotidine are headache, abdominal pain, constipation, or diarrhea.

A 72-year-old patient was admitted with epigastric pain due to a gastric ulcer. Which patient assessment warrants an urgent change in the nursing plan of care? A. Chest pain relieved with eating or drinking water B. Back pain 3 or 4 hours after eating a meal C. Burning epigastric pain 90 minutes after breakfast D. Rigid abdomen and vomiting following indigestion

D. Rigid abdomen and vomiting following indigestion A rigid abdomen with vomiting in a patient who has a gastric ulcer indicates a perforation of the ulcer, especially if the manifestations of perforation appear suddenly. Midepigastric pain is relieved by eating, drinking water, or antacids with duodenal ulcers, not gastric ulcers. Back pain 3-4 hours after a meal is more likely to occur with a duodenal ulcer. Burning epigastric pain 1-2 hours after a meal is an expected manifestation of a gastric ulcer related to increased gastric secretions and does not cause an urgent change in the nursing plan of care.

What should the nurse instruct the patient to do to best enhance the effectiveness of a daily dose of docusate sodium (Colace)? A. Take a dose of mineral oil at the same time. B. Add extra salt to food on at least one meal tray. C. Ensure dietary intake of 10 g of fiber each day. D. Take each dose with a full glass of water or other liquid.

D. Take each dose with a full glass of water or other liquid. Docusate lowers the surface tension of stool, permitting water and fats to penetrate and soften the stool for easier passage. The patient should take the dose with a full glass of water and should increase overall fluid intake, if able, to enhance effectiveness of the medication. Dietary fiber intake should be a minimum of 20 g daily to prevent constipation. Mineral oil and extra salt are not recommended.

d (The surgery Ms. Jacobs will undergo is a strictureplasty. Longitudinal incisions are made in the narrow bowel segments to relieve strictures. The total colectomy ileal​ pouch-anal anastomosis​ (IPAA) is usually performed on patients with ulcerative colitis. A bowel resection is aimed at reducing inflammation by removing the diseased part of the bowel. In​ Ms.Jacob's case, there is a less serious option. A sigmoidoscopy is a procedure to inspect the bowels for changes​ and, if​ needed, to take biopsies.)

Danae Jacobs is a​ 35-year-old woman with a history of Crohn​ disease, which manifested itself when she was 21 years old. For the last 2​ years, she has been on numerous antibiotics and steroids with less than desirable effects. After a recent​ colonoscopy, the surgeon has told Danae that strictures have formed in her bowel and he suggested surgery. What surgery does​ Danae's nurse anticipate Danae​ needing? a Sigmoidoscopy b Bowel resection c Total colectomy ileal​ pouch-anal anastomosis​ (IPAA) d Strictureplasty

Following a partial gastrectomy for gastric cancer, a patient complains of nausea, abdominal pain and cramping, and diarrhea after eating. Recognizing manifestations of dumping syndrome, what should the nurse recommend?

Decreasing the amount of fluid taken at meals and avoiding high-carbohydrate foods, including fluids such as fruit nectars

d (​Eli's ulcerative colitis is causing severe​ diarrhea, and he is most likely dehydrated.​ Therefore, the nurse should provide intravenous fluids as prescribed.​ Anti-inflammatory medications should be administered as prescribed to help ameliorate the diarrhea episodes. The nurse should assess the perianal region for irritation or excoriation and apply a protective cream to the irritated areas as indicated. Daily weights should be performed to help assess​ Eli's hydration status.)

Eli Schwartz is a​ 15-year-old boy who was recently diagnosed with ulcerative colitis and is admitted to the hospital with severe diarrhea. Which nursing intervention would​ Eli's nurse provide for​ Eli? a Weigh every other day b Avoid placing any kind of cream on the perianal region c Avoid​ anti-inflammatory medications d Infuse intravenous fluids as prescribed

A nurse is providing dietary teaching to a client who has diverticulitis about preventing acute attacks. Which of the following foods should the nurse recommend?

Foods high in fiber

d (Rationale Hepatorenal syndrome causes sodium​ retention, oliguria, and hypotension. Asterixis develops with hepatic​ encephalopathy, and fever with bacterial peritonitis. Esophageal varices are a complication of cirrhosis.)

For which complication should the nurse monitor a client with portal​ hypertension? a Hepatic encephalopathy b Esophageal varices c Steatohepatitis d Hepatitis C

d (Rationale Hepatorenal syndrome causes sodium​ retention, oliguria, and hypotension. Asterixis develops with hepatic​ encephalopathy, and fever with bacterial peritonitis. Esophageal varices are a complication of cirrhosis.)

For which manifestation should the nurse assess in a client with hepatorenal​ syndrome? a Esophageal varices b Asterixis c Fever d Sodium retention

What are the 2 causes of peptic ulcer disease

H. pylori NSAID use

Which interventions are considered conservative treatments for hemorrhoids? (Select all that apply.)

Hemorrhoid creams and ointments - sitz bath - OTC analgesics

2. The nurse identifies which nursing diagnosis as the highest priority for the patient admitted with PUD and possible perforation?

Impaired tissue integrity (GI)

The nurse correlates strangulated hernias to which finding?

Impede blood flow of the intestines

3 (Interferon Alpha 2b most commonly causes flulike adverse effects such as myalgia, arthralgia, headache, N, fever and fatigue. Retinopathy is a potential adverse effect but not a common one. Diarrhea may develop as an adverse effect. Clients are advised to administer the drug at bedtime and get adequate rest. Medications may be prescribed to treat the symptoms. The drug may also cause hematologic changes, therefore lab tests such as a CBC and differential should be conducted monthly during drug therapy. Blood glucose levels should be monitored for HYPER glycemia.)

Interferon alfa-2b has been prescribed to treat a client with chronic Hep B. The nurse should assess the client for which common adverse side effect? 1. retinopathy 2. constipation 3. flulike symptoms 4. hypoglycemia

The nurse caring for a patient with liver failure notes a change in mental status and elevated ammonia level. Which ordered interventions are most directly related to treating these clinical manifestations?

Lactulose and neomycin

A patient just returned from the procedure area after having a liver biopsy. The primary nursing intervention includes which action?

Monitor blood pressure and assess the site for bleeding

The nurse should question the administration of which medication in the patient admitted with cholecystitis?

Morphine

d

Mr. Manning is a​ 36-year-old client diagnosed with​ end-stage liver disease. He began drinking alcohol heavily at 13 years of age. Which symptoms of cirrhosis caused by alcohol abuse should the nurse anticipate Mr. Manning to exhibit upon​ assessment? a Profuse​ sweating, jaundice, and hypoglycemia ​b Low-grade fever, acute abdominal pain that radiates to distal areas upon​ palpation, nausea, and vomiting c Crushing chest​ pain, jaundice, and shortness of breath d Fluid buildup in the​ abdomen, bleeding in the esophagus and upper​ stomach, abdominal​ inflammation, infection, and portal systemic encephalopathy

d (While a​ beta-blocker can be used for esophageal​ varices, the best therapy at this time is a balloon tamponade​ (either a​ Sengstaken-Blakemore or Minnesota​ tube). A paracentesis is done to relieve severe ascites. A transjugular intrahepatic portosystemic shunt​ (TIPS) relieves portal hypertension and reduces the onset of esophageal varices and ascites.)

Ms. Charlotte is​ 66-years-old and admits to being an alcoholic for most of her adult life. She is brought to the emergency department with bleeding esophageal varices. Which therapy should be the most effective for Ms. Charlotte at this​ time? a Transjugular intrahepatic portosystemic shunt​ (TIPS) b Paracentesis ​c Beta-blocker d Minnesota tube

4 (4 Stopping the diarrhea is a priority for Ms. T. Chronic, frequent diarrhea is demoralizing, and fluid and electrolyte losses cause weakness. If the bowel is allowed to rest, the cramping will stop. The other options also provide accurate information, but the potential resolution of the most disturbing symptom will encourage her to continue. Focus: Prioritization)

Ms. T is discouraged and dispirited about her ulcerative colitis. She is resistant to TPN because "I'm being kept alive with tubes." Which explanation will encourage Ms. T to continue with the TPN therapy? 1. "It will help you regain your weight." 2. "It will create a positive nitrogen balance." 3. "Your physician has ordered this important therapy for you." 4. "Your bowel can rest and the diarrhea will decrease."

3 (Sulfasalazine is potentially nephrotoxic. The other adverse effects are also possible, but are less serious. Focus: Prioritization)

Ms. T is receiving sulfasalazine (Azulfidine) 500 mg by mouth every 6 hours for treatment of ulcerative colitis. Which assessment finding concerns you the most? 1. Urine discoloration 2. Nausea and vomiting 3. Decreased urine output 4. Headache

The nurse assesses for which clinical manifestation in the patient with cholecystitis?

Murphy's sign

The nurse is caring for a 33-year-old woman who has been taking aspirin for back pain and has experienced a sudden episode of tachycardia and feeling faint. She also vomited coffee-ground emesis and passed a tarry stool but has no complaints of pain or heartburn. The patient wants to know why there was no sign of pain as a warning signal prior to the sudden bleeding. What is the nurse's best response?

NSAID-induced gastric injury often is without symptoms, and life-threatening complications such as GI bleeding can occur without warning.

The nurse is caring for a postoperative patient who has had surgery for hemorrhoids. What information should be taught to the patient in the postoperative period?

No bowel movement is expected for 3 days

pH: 7.45, PCO2: 44, HCO3: 22

Normal

The patient receiving chemotherapy rings the call bell and reports the onset of nausea. The nurse should prepare an as-needed dose of which medication? A. Morphine sulfate B. Zolpidem (Ambien) C. Ondansetron (Zofran) D. Dexamethasone (Decadron)

Ondansetron is a 5-HT3 receptor antagonist antiemetic that is especially effective in reducing cancer chemotherapy-induced nausea and vomiting. Morphine sulfate may cause nausea and vomiting. Zolpidem does not relieve nausea and vomiting. Dexamethasone is usually used in combination with ondansetron for acute and chemotherapy-induced emesis.

The nurse recognizes that the treatment of H pylori includes which medications? (Select all that apply.)

PPIs - antibiotics

a,b (Rationale: Clients undergoing paracentesis for the manual removal of excess fluid from the abdomen should be monitored closely for electrolyte imbalance and a drop in intravascular volume (blood pressure) as the pressure of the ascites fluid is relieved. Tachycardia, jaundice, and constipation are not expected complications of paracentesis.)

Paracentesis is prescribed for an adult client with chronic cirrhosis and ascites that is not responding to diuretic therapy. The nurse should monitor the client for which complications of this procedure? (Select all that apply.) a Electrolyte imbalance b Drop in blood pressure c Constipation d Jaundice e Tachycardia

b,c,e (Rationale: Children with ulcerative colitis typically present with pancolitis which tends to be aggressive and may require clients to undergo bowel surgery to manage the disease and its complications. The primary pharmacologic treatment involves systemic and locally-acting anti-inflammatory drugs, corticosteroids, and anti-diarrheal agents. Because of compromised nutrient absorption from the inflamed bowel, dietary supplementation is needed. Children with ulcerative colitis, like adults, are at greater risk for developing cancer of the colon. It is also very important for affected individuals to adhere to a strict medication regimen even when asymptomatic)

Parents of a 9-year-old diagnosed with ulcerative colitis following a hospitalization for pancolitis ask the nurse to teach them more about the disease, its progression, and the typical treatment regimen. The nurse can accurately share what information with the parents? (Select all that apply.) a Nutritional supplements are usually not required since nutrient absorption is adequate for growth during the periods of remission. b These children often require surgery at some point to manage the disease or its complications. c Medications should be continued even when the child has no symptoms. d Children with ulcerative colitis are at not at greater risk for developing colon cancer in adulthood. e Typical treatment regimens include systemic and locally-acting anti-inflammatory drugs, corticosteroids, and anti-diarrheal agents.

A nurse is caring for a client who has a percutaneous endoscopic gastrostomy (PEG) tube and is receiving intermittent feedings. Prior to initiating the feeding, which of the following actions should the nurse take first?

Place the client in semi-Fowler's position

In reviewing diagnostic results of a patient with suspected hepatitis, the nurse correlates which result as consistent with hepatitis A?

Presence of IgM anti-HAV

A nurse is caring for a client who is scheduled to undergo a liver biopsy for a suspected malignancy. Which of the following laboratory findings should the nurse monitor prior to the procedure?

Prothrombin time

*R*espiratory *O*pposite *M*etabolic *E*qual

Respiratory Alkalosis = pH ↑ & PaCO2 ↓ Respiratory Acidosis = pH ↓ & PaCO2 ↑ Metabolic Alkalosis = pH ↑ & HCO3 ↑ Metabolic Acidosis = pH ↓ & HCO3 ↓

Which nursing diagnosis is most appropriate for a patient admitted for acute GI bleed related to acute gastritis?

Risk for deficient fluid volume

The nurse recognizes which findings as diagnostic for IBS?

Rome III and/or Manning criteria

In caring for a patient who sustained a Le Fort fracture, the nurse monitors for which complications? (Select all that apply.)

TMJ pain - airway obstruction - infection

1

TPN is prescribed for a client with Crohns. The TPN solution is having an intended outcome when: 1. The clients nutritional needs are met 2. The client does not have metabolic acidosis 3. The client is hydrated 4 The client is in a negative nitrogen balance

2,4

The HCP has prescribed Cipro for a client who takes warfarin. What should the nurse instruct the client to do? Select all that apply 1. split the tablets and stir them in food 2. avoid exposure to sunlight 3. eleminate caffeine from the diet 4. report unusual bleeding 5. increase fluid intake to 3000 mL.day

3

The HCP instructs a client with alcohol induced cirrhosis to stop drinking alcohol. The expected outcome of this intervention is: 1 absence of deirium tremens 2. having a balanced diet 3. improved liver function 4. reduced weight

2 (Explaining the physiologic reason helps the UAP to understand that rest is part of the therapy. Following physician's orders is important, but it is an inadequate explanation. Depression does not justify bed rest. Using large words to explain common concepts should be avoided, regardless of the audience. Focus: Supervision; QSEN: TC; Concept: Professionalism)

The UAP asks, "Why can't Ms. T get out of bed and do things for herself? She's only 29." What is your best response? 1. "The physician ordered bed rest for a few days." 2. "Decreasing activity helps to decrease the diarrhea." 3. "Acute exacerbations require decreased GI motility." 4. "She is too depressed and malnourished."

2

The basic component of TPN solution is likely to be: 1 isotonic dextrose solution 2 hypertonic dextrose solution 3 hypotonic dextrose solution 4 colloidal dextrose solution

1 (Hot water increases pruritis and soap will cause dry skin, which increases pruritis, therefore the nurse should discuss this with the UAP.)

The client diagnosed with liver failure is experiencing pruritus secondary to severe jaundice. Which action by the UAP warrants intervention by the nurse? 1. The UAP is assisting the client to take a hot, soapy shower 2. The UAP applies an emollient to the clients legs and back 3. The UAP puts mittens on both hands of the client 4. The UAP pats the clients skin dry with a towel

3 (The client is at risk for hypovolemia, therefore VS will be assessed freq to monitor for signs of hemorrhaging. The procedure is done in the clients room with the client seated on the bed or chair The client should empty the bladder prior, no need for Foley The client does not have to hold the breath when the cath is inserted into the peritoneum. This is done when obtaining a liver biopsy)

The client dx with end stage liver failure and ascites is scheduled for a paracentesis. Which client teaching should the nurse discuss with the patient? 1. Explain the procedure will be done in the OR 2. Instruct the client a Foley catheter will have to be inserted 3. Tell the client VS will be taken frequently after the procedure 4. Provide instructions on holding breath when the HCP inserts the catheter

2 (While the balloons are inflated, the client must not be left unattended in case they become dislodged and occlude the airway. This is a safety issue. The clients throat is not anesthetized during the insertion of a NG tube so the gag reflex does not need to be assessed. This laxative is administered to decrease the ammonia level, but the question does not say the ammonia level is elevated Esophageal bleeding does not cause the ammonia level to be elevated)

The client dx with end stage liver failure is admitted with esophageal bleeding. The HCP inserts and inflates a triple-lumen nasogastric tube (Sengstaken-Blakemore). Which nursing intervention should the nurse implement for this treatment? 1. Assess the gag reflex every shift 2. Stay with the client at all times 3. Administer the laxative lactulose (Chronulac) 4. Monitor the clients ammonia level

4 (Ammonia is a byproduct of protein metabolism and contributes to hepatic encephalopathy Reducing protein intake should decrease ammonia levels. Sodium is restricted to reduce ascites and generalized edema, not for hepatic encephalopathy Fluids are calculated based on diuretic therapy, urine output and serum electrolyte values; fluids do not affect hepatic encephalopathy A diet high in calories and moderate in fat intake is recommended to promote healing)

The client dx with end stage liver failure is admitted with hepatic encephalopathy. Which dietary restriction should be implemented by the nurse to address this complication? 1. Restrict the sodium intake to 2 g/day 2. Limit oral fluids to 1500 mL/day 3. Decrease the daily fat intake 4. Reduce protein intake to 60-80 g/day

2 (Bilirubin the by product of RBC destruction, is metabolized in the liver and excreted via the feces. which causes the feces to be brown in color. If the liver is damaged, the bilirubin is excreted via the urine and the skin. The serum ammonia level is increased but does not cause clay colored stools The liver excretes bile into the gallbladder and the body uses the bile to digest fat, but does not affect the feces Vitamin deficiency, resulting from the livers inability to detoxify vitamins may cause steatorrhea, but does not cause clay colored stool)

The client dx with liver problems asks the nurse why are my stools clay colored? On which scientific rationale should the nurse base the response? 1. There is an increase in serum ammonia level 2. The liver is unable to excrete bilirubin 3. The liver is unable to metabolize fatty foods 4 A damaged liver cannot detoxify vitamins

3 (Direct pressure is applied to the site and then the client is placed on the R side to maintain pressure. The client should empty the bladder prior to the procedure Foods and fluids are usually withheld 2 hrs after the biopsy, after which the client can resume the usual diet BUN and creatinine levels are monitored for kidney function and the renal system is not affected by liver biopsy)

The client has a liver biopsy. Which postprocedure intervention should the nurse implement? 1. Instruct the client to void immediately 2. Keep the client NPO for 8 hrs 3. Place the client on the right side 4. Monitor the BUN and creatinine level

1 (Blood in the GI tract is digested as a protein, which increases serum ammonia levels and increases the risk of developing hepatic encephalopathy. Decreased albumin causes ascites An enlarged spleen increases the rate at which RBCs, WBCs, and platelets are destroyed, causing the client to develop anemia, leukopenia, and thrombocytopenia, but not hepatic encephalopathy. An increase in aldosterone causes sodium and water retention, resulting in ascites and general edema)

The client has end stage liver failure secondary to alcoholic cirrhosis. Which complication indicates the client is at risk for developing hepatic encephalopathy? 1 GI bleeding 2. Hypoalbuminemia 3. Splenomegaly 4. Hyperaldosteronism

1,2,3,4 (Vitamin K deficiency causes impaired coagulation. no rectal temps Soft bristle toothbrushes help prevent gums bleeding Platelet count, PTT, PT and INR should be monitored to assess coagulation status. Injections should be avoided if at all possible, because the client is unable to clot, but if they are absolutely necessary, the nurse should use small gauge needles. Asterixis is a flapping tremor of the hands when the arms are extended and indicates an elevated ammonia level not associated with vitamin K deficiency)

The client in end stage liver failure has vitamin K deficiency. Which intervention should the nurse implement? SATA 1 Avoid rectal temperatures 2. Use only a soft toothbrush 3. Monitor the platelet count 4. Use small gauge needles 5. Assess for asterixis

2 (There is no instrument used at home to test daily ammonia levels. the ammonia level is a serum level requiring venipuncture and laboratory diagnostic equipment. 2-3 soft stools indicates the med is effective. Diarrhea indicates an OD of the med, possibly requiring the dosage to be decreased. The HCP needs to make this change in dosage, so the client understands the teaching. The client should check the stool for bright red blood as well as dark tarry stool)

The client is admitted with end stage liver failure and is prescribed the laxative lactulose (Chronulac). which statement indicates the client needs more teaching concerning this medication? 1. I should have 2-3 soft stools a day 2. I must check my ammonia level daily 3. If I have diarrhea I will call my doctor 4. I should check my stool for any blood

3 (This is the main reason the HCP decreases dosage of clients medications. #4 is the medical explanation of why, but not the layperson answer)

The client is dx with end stage liver failure. The client asks the nurse. "Why is my doctor decreasing the doses of my medications?" Which statement by the nurse is the best response? 1. You are worried because the doctor has decreased dosage 2. You really should ask the doctor I am sure there is a good reason 3. You may have an overdose of the medication because your liver is damaged 4. The half life of the medications is altered because the liver is damaged

4 (Flu like symptoms are the first complaints of the client in the preicteric phase of hepatitis. Which is the initial phase and may begin abruptly or insidiously. Clay colored stools and jaundice occur in the icteric phase Normal appetite and itching occur in the icteric phase Fever subsides in the icteric phase, and the pain is in the RUQ)

The client is in the preicteric phase of hepatitis. Which S/s should the nurse expect the client to exhibit during this phase? 1. Clay colored stools and jaundice 2. Normal appetite and pruritis 3. Being afebrile and LUQ pain 4. Complaint of fatigue and diarrhea

1,2,3,4 (TElemetry should be monitored during therapy, client is at risk for ARDS and pulmonary toxicity, When the IV med, monitoring the renal and hepatic function is appropriate, this drug can cause hepatomegaly. IV vasoactive drugs are inherently dangerous so should confirm orders with another nurse.)

The client is showing ventricular ectopy, and the HCP orders amiodarone IV. Which interventions should the nurse implement? SATA 1. Monitor telemetry continuously 2. Assess the clients respiratory status 3. Evaluate the clients liver function studies 4. Confirm the original order with another nurse 5 Prepare to defib the client at 200 joules

4 (The most appropriate goal for this client with Hep A is to increase activity tolerance gradually as tolerated. Periods of alternating rest and activity should be included in the plan of care There is no evidence the client is physically immobile, is unable to provide self care or needs to adapt to new energy levels)

The client with Hep A is experiencing fatigue, weakness, and a general feeling of malaise. The client tires rapidly during morning care. The most appropriate goal for this client is to: 1. increase mobility 2. learn new self care skills 3. adapt to new levels of energy 4. gradually increase activity tolerance

2 (Milk thistle has an active ingredient silymarin, which has been used to treat liver disease for more than 2000 years. It is a powerful oxidant and promotes liver cell growth.)

The client with hepatitis asks the nurse "I went to a herbalist who recommended I take milk thistle. What do you think about that herb? Which statement by the nurse is the best response? 1. You are concerned about taking an herb? 2. The herb has been used to treat liver disease 3. I would not take anything that is not prescribed 4. Why would you want to take any herb?

3 (The vaccine is given in 3 doses - initially, and then at 1 month, then again at 6 months. The vaccine is administered in a series of 3 injections and is reported to be effective for life, but boosters may be given every 5 years. It is given IM into the deltoid muscle)

The employee health nurse is discussing hepatitis B vaccines with new employees. Which statement best describes the proper administration of the hepatitis B vaccine? 1. The vaccine must be administered once per year 2. Two (2) mL of vaccine should be given each hip 3. The vaccine is given in three (3) doses over a six (6) month period 4. The vaccine is administered intradermally into the deltoid muscle

2 (First clean the needlestick with soap and water and attempt stick bleed to remove any virus injected into the skin. The other interventions will be done, and the infection control nurse/employee health nurse will check the hepatitis status of the client)

The female nurse sticks herself with a contaminated needle. Which action should the nurse implement first? 1. notify the infection control nurse 2. Cleanse the area with soap and water 3. REquest postexposure prophylaxis 4. Check the hepatitis status of the client

c (Rationale: The signs and symptoms of Crohn disease vary widely since any portion of the GI tract, from the mouth to the anus, can be affected. Early manifestations of the disease can be insidious, and it is not uncommon for individuals to learn they have the disease when they develop an intestinal obstruction or abscess. Careful adherence to drug therapy plays a key role in terminating acute illness episodes and in preventing relapse. In spite of frequent diarrhea, clients with Crohn disease on long-term corticosteroid therapy may experience Cushingoid effects (e.g., abnormal fat deposition in the face and trunk, osteoporosis). )

The nurse and pharmacist are planning an educational session for families of newly-diagnosed clients with Crohn's disease. Content they should cover include which facts about this chronic condition? a Corticosteroids used to reduce inflammation and induce remission in Crohn's disease have few systemic side effects because of their rapid transit through the gut. b Adherence to a prescribed treatment regimen does not significantly improve rates of reoccurrences or disease flares. c The manifestations of Crohn's disease vary widely among individuals with this condition. d It is rare for clients to first learn they have Crohn's disease by experiencing a bowel obstruction or intestinal abscess.

b (For a client with​ cirrhosis, the nurse assesses recent weight loss during the heath history portion of the nursing assessment. Vital​ signs, mental​ status, and skin color are assessed during the physical examination portion of the nursing assessment.)

The nurse assesses for which item during the health history for a client with​ cirrhosis? a Mental status b Weight loss c Skin color d Vital signs

3 (hypokalemia is a precipitating factor in hepatic encephalopathy. A decrease in creatinine results from muscle atrophy, an increase in creatinine would indicate renal insufficiency. With liver dysfunction, increased aldosterone levels are seen. A decrease in serum protein will decrease colloid osmotic pressure and promote edema)

The nurse assessing the client with cirrhosis who has developed hepatic encephalopathy. The nurse should notify the HCP of a decrease in which serum lab value that is a potential precipitating factor for hepatic encephalopathy? 1. aldosterone 2. creatinine 3. potassium 4. protein

d (Rationale: The nurse would expect the client's respiratory rate and oxygen saturation to fall within normal limits, and the client's abdominal girth should decrease by 1-2 cm/day. A decrease in flatulence is unrelated to this procedure.)

The nurse caring for a client recently undergoing abdominal paracentesis for ascites would expect all of the following indicators of successful treatment except: a a respiratory rate within normal range. b improved oxygen saturation. c a reduction in abdominal girth of 1-2 cm/day. d reduction in excess flatulence.

3

The nurse changing the subclavian dressing of a client who is receiving TPN. When assessing the cath site the nurse notes the presence of yellow drainage around the sutures that are anchoring the cath. What should the nurse do first? 1. clean the insertion site and redress the area 2. document the assessment findings in the clients chart 3. request a prescription to obtain a culture of the drainage 4. check the clients temp.

3 (The hep A virus is transmitted via the fecal-oral route. It spreads thru contaminated hands, water and food, especially shellfish growing in contaminated water. Certain animal handlers are at risk for Hep A. particularly those handling primates. Freq hand washing is probably the single most preventative action. Insects do not transmit Hep A. Family members do not need to stay away from the client with hep. It is not necessary to disinfect food or clothing)

The nurse develops a teaching plan for the client about how to prevent the transmission of hepatitis A. Which discharge instruction is appropriate for the client? 1 spray the house to eliminate infected insects 2. tell family members to try to stay away from the client 3. Ask the family members to wash their hands freq 4. Disinfect all clothing and eating utensils

2 (Excess fluid volume could be secondary to portal hypertension. Therefore no increase in abdominal girth would be an appropriate short term goal, indicating excess of fluid volume. 2 kg is more than 4 lbs, which indicates severe fluid retention and is not an appropriate goal. VS are appropriate to monitor, but do not yield specific information on fluid volume status Having the client receive a low sodium diet does not ensure the client will comply with the diet. The short term goal must evaluate if the FV is wnl)

The nurse identifies the client problem "excess fluid volume" for the client in liver failure. Which short term goal would be most appropriate for this problem? 1. The client will not gain more than 2 kg a day 2. The client will have no increase in abdominal girth 3. The clients VS will remain within normal limits 4. The client will receive a low sodium diet

a (Rationale A total colectomy IPAA is a treatment for a client with ulcerative colitis. It entails the removal of the entire colon and rectum and the formation of a temporary or loop ileostomy at the same time. The ileostomy is used for 2 to 3 months. A strictureplasty is used to treat bowel strictures and does not involve the removal of the bowel or creation of an ostomy. A gastric resection is the removal of part of the​ stomach, not the bowel. It does not involve the creation of an ostomy. A pyloroplasty is a surgical procedure to widen the opening of the pyloric valve at the lower portion of the stomach. It does not involve the removal of the bowel or the creation of an ostomy.)

The nurse is admitting a client with ulcerative colitis who is scheduled for surgery. The client tells the nurse that he is having his bowel removed and will have a temporary ostomy for 2 to 3 months. Which upcoming surgical procedure is the client​ describing? a Total colectomy ileal​ pouch-anal anastomosis​ (IPAA) b Gastric resection c Pyloroplasty d Strictureplasty

3 (early manifestations of cirrhosis are subtle and usually include GI symptoms, such as anorexia, N/V, and changes in bowel patterns. These changes are caused by the livers altered ability to metabolize carbs, proteins, and fats. Peripheral edema, ascites, and jaundice are LATER signs of liver failure and portal hypertension)

The nurse is assessing a client who is in the early stages of cirrhosis of the liver Which focused assessment is appropriate? 1. peripheral edema 2. ascites 3. anorexia 4. jaundice

1 (A client diagnosed with portal hypertension should be assessed for a tympanic (fluid) wave to check for ascites. High BP is not an etiology of portal hypertension In portal hypertension percussion is difficult and will not provide information about the clients condition Weighing the client should be done daily, not twice a week)

The nurse is assessing the client in end stage liver failure who is diagnosed with portal hypertension. Which intervention should the nurse include in the plan of care? 1. Assess the abdomen for tympanic wave 2. Monitor the clients BP 3. Percuss the liver for size and location 4. WEigh the client twice each week

4 (Tylenol is toxic to the liver and should be avoided. Increased periods of rest allow for liver regeneration. A low fat high carb diet and dry toast to relieve nausea are appropriate)

The nurse is assessing the client with hepatitis A and notices that the AST and ALT lab values have increased. Which statement by the client indicates the need for further instruction by the nurse? 1. I require increased periods of rest 2. I follow a low-fat high carb diet 3. I eat dry toast to relieve my nausea 4. I take acetaminophen for arthritis pain

2,5

The nurse is assigning clients for the evening shift. Which clients are appropriate to assign to a LPN to provide client care? Select all that apply 1. a client with Crohns who is receiving TPN 2. a client who had inguinal hernia repair surgery 3 hours ago, VS are stable 3. a client with an intestinal obstruction who needs a cantor tube inserted 4. a client with diverticulitis who needs teaching about take home meds 5. a client who is experiencing an exacerbation of ulcerative colitis

4 (Although primarily bloodborne, unprotected sex with multiple partners and a Hx of STD's are risk factors for transmission of Hep C virus. Other risk factors include blood transfusions, past Tx with chronic hemodialysis, being a child born to a woman with Hep C, past/current illicit drug IV use, or needlestick injuries to health care workers. It is important the nurse is aware of the clients Hx in order to determine the clients level of understanding of the disease, promote a healthy lifestyle and discuss the role of viral transmission of the disease)

The nurse is caring for a client recently diagnosed with Hep C. In reviewing the clients Hx what info will be most helpful as the nurse develops a teaching plan? The client: 1. has a Hx of exercise induced asthma 2. is a scientist and frequently exposed to multiple chemicals 3. traveled to Central America recently and ate uncooked vegetables 4 has a known Hx of STDs

d (Rationale Probiotics help reduce inflammation in the bowel and would be used in the treatment of inflammatory bowel disease. Glucose is needed for cellular energy. It is not considered an​ anti-inflammatory agent. Vitamin K is used for clotting. It is not considered an​ anti-inflammatory agent. Energy drinks contain sugars and electrolytes. They are not considered​ anti-inflammatory agents.)

The nurse is caring for a client who has chronic inflammatory bowel disease. The client tells the nurse the she is taking nutritional supplements that help reduce the inflammation. Which supplements are used in the treatment of chronic inflammatory bowel​ disease? a Vitamin K tablets b Energy drinks c Glucose tablets d Probiotics

a,b,d (Rationale Mesalamine is a​ sulfa-based medication that has the same active ingredients as sulfasalazine and therefore should not be administered to a client with a sulfa allergy. Cipro is a​ fluoroquinolone, not a sulfa​ drug, and could be given to a client with a sulfa allergy. Sulfasalazine is a sulfonamide and therefore should not be administered to a client with a sulfa allergy. Olsalazine is a​ sulfa-based medication that has the same active ingredients as sulfasalazine and therefore should not be administered to a client with a sulfa allergy. Clarithromycin is a​ macrolide, not a sulfa​ drug, and could be given to a client with a sulfa allergy.)

The nurse is caring for a client with chronic inflammatory bowel disease who is prescribed antibiotics to treat the condition. The client reports an allergy to sulfa. Which medication orders will the nurse​ question? ​(Select all that​ apply.) a Olsalazine​ (Dipentum) b Mesalamine​ (Asacol, Rowasa) c Ciprofloxacin​ (Cipro) d Sulfasalazine​ (Azulfidine) e Clarithromycin​ (Biaxin)

a,c,e (Rationale Dietary support is essential because dietary needs change as hepatic function fluctuates. Nutrition therapy includes sodium restricted to under 2​ g/day, fluids restricted to​ 1,500 mL/day, vegetable proteins provided with restricted red meat​ consumption, parenteral nutrition as​ needed, and vitamin supplements that include B​ complex, A,​ D, and E. A regular diet or​ high-fiber diet is not appropriate for a client with cirrhosis.)

The nurse is caring for a client with cirrhosis of the liver. Which dietary support does this client​ need? ​(Select all that​ apply.) ​a Fluid-restricted diet ​b High-fiber diet ​c Sodium-restricted diet d Regular diet e Vitamin supplements

b,d,e (Rationale Risk factors for cirrhosis of the liver include excessive alcohol​ use; infection with Hepatitis​ B, C, or​ D; and injection drug use. Biliary atresia​ (poorly formed bile​ ducts) and hepatitis E are not risk factors for cirrhosis of the liver.)

The nurse is caring for a client with cirrhosis of the liver. Which risk factors should the nurse expect to find in the​ client's history? ​(Select all that​ apply.) a Hepatitis E infection b Excessive alcohol use c Biliary atresia d Injection drug use e Hepatitis C infection

a,b,c,e (Rationale Assessment findings that correlate with expected laboratory findings in the client with cirrhosis include bruising​ easily, frequent​ infections, peripheral​ edema, and confusion. Although spider angiomas can be found in clients with​ cirrhosis, their presence is not associated with any laboratory testing.)

The nurse is caring for a client with cirrhosis. Which assessment findings correlate with expected laboratory findings in the​ client? ​(Select all that​ apply.) a Peripheral edema b Frequent infections c Bruising easily d Spider angiomas e Confusion

a,b,d (Rationale Corticosteroids should be taken with food or at mealtimes to reduce the gastrointestinal side effects. A client taking corticosteroids tends to retain​ fluid, and sodium tends to exacerbate the retention.​ Therefore, the client should reduce the intake of foods that are high in sodium. Corticosteroids should be taken consistently and not be stopped abruptly to reduce the possibility of adrenal shutdown. A client taking corticosteroids tends to​ gain, not​ lose, weight. A client taking corticosteroids tends to have​ high, not​ low, blood pressure.)

The nurse is caring for a client with inflammatory bowel disease who is being treated with corticosteroids. Which information would the nurse provide for the client about the​ medications? ​(Select all that​ apply.) a Reduce intake of foods high in sodium b Take medication consistently and do not stop abruptly c It may cause low blood pressure. d Take with food or at mealtimes e Expect weight loss

3 (An increase in abdominal girth indicates increasing ascites. meaning the clients condition is becoming more serious and should be reported. The normal direct bilirubin level is 0.1-0.4 but a decrease in a value although still elevated would not be reported)

The nurse is caring for the client dx with ascites secondary to hepatic cirrhosis. Which information should the nurse report to the HCP? 1. A decrease in the clients weight by 1 lb 2. An increase in urine after administration of diuretic 3. An increase in abdominal girth of 2 inches 4. A decrease in the serum direct bilirubin to 0.6

2 (The inability to circle food items on the menu may indicate deterioration in the clients cognitive status. The clients neurological status is impaired with hepatic encephalopathy the nurse should investigate this behavior. An increase in ammonia is seen in clients dx with hepatic encephalopathy and coma)

The nurse is caring for the client dx with hepatic encephalopathy Which sign and symptom indicate the disease is progressing? 1. The client has a decrease in serum ammonia 2. The client is not able to circle choices on the menu 3. The client is able to take deep breaths as directed 4. The client is able to eat previously restricted food items

4 (a client with esophageal varices is at an even higher risk of bleeding with elevated PT/INR. The nurse should collaborate with the HCP to prevent bleeding. The other lab findings are not as life threatening. A decreased serum albumin can cause fluid to move into the interstitial tissues. Increased ammonia levels are toxic to the brain. Calcium loss is more common in pancreatitis.)

The nurse is caring for the client with esophageal varices. The nurse should discuss which laboratory report finding with the HCP? 1. normal serum albumin 2. decreased ammonia 3. slightly decreased levels of Calcium 4. elevated PT/INR

1 (Tx of hep consists primarily of bed rest with bathroom privileges. Bed rest is maintained during the acute phase to reduce metabolic demands on the liver, thus increasing its blood supply and promoting liver cell regeneration. When activity is gradually resumed, the client should be taught to rest before becoming overly tired. Although adequate fluid intake is important, it is not necessary to force fluids to tx hep. Antibiotics are not used. Electrolyte imbalances are not typical of hep)

The nurse is developing a plan of care for the client with viral hepatitis. The nurse should instruct the client to : 1. obtain adequate bed rest 2 increase fluid intake 3 take antibiotic therapy as prescribed 4. drink 8 oz of an electrolyte solution every day

a,b,c,d (Rationale Chronic inflammatory bowel disease tends to run in​ families, with​ 15% to​ 25% of clients having a family member with one of the diseases. Although the peak incidence of chronic inflammatory bowel disease is in adolescence and young​ adulthood, it can also affect older adults. Chronic inflammatory bowel disease is not caused by environmental factors​ alone, but these factors do play a role in the etiology of the disease. The peak incidence of chronic inflammatory bowel disease is in adolescence and young​ adulthood, between the ages of 15 and 30 years. Chronic inflammatory bowel disease occurs more frequently in the United States and northern European nations. As many as 1 million Americans have chronic inflammatory bowel disease.)

The nurse is discussing the incidence of inflammatory bowel disease with a group of nursing students. Which information would the nurse include in the​ discussion? ​(Select all that​ apply.) a The disease occurs more frequently in the United States and northern European nations. b Environmental factors can contribute to the etiology of chronic inflammatory bowel disease. c The disease is often linked to heredity. d The peak incidence of the disease is in adolescence and young adulthood. e Chronic inflammatory bowel disease does not affect older adults.

b (Rationale: Frequent diarrhea results in irritated or denuded perianal skin. Gentle cleansing agents such as Peri-Wash, tucks, diaper wipes, and cotton balls saturated with witch hazel can be both soothing and facilitate healing. Protective creams like zinc oxide-based products also help to protect the skin around the anus from breakdown. Antibiotics are not routinely placed around the anus. Maintaining good fluid intake will lessen the risk of having dehydrated skin that is more susceptible to excoriation. )

The nurse is evaluating the client's understanding regarding care of the skin during an acute exacerbation of inflammatory bowel disease with frequent stooling. The nurse determines that teaching goals have been met when the client indicates he will follow which actions for self-care? a Apply antibiotic ointment around the anus. b Carefully clean the perianal skin with a mild cleanser. c Limit intake of fluids. d Use a perfumed or scented lotion around the anus.

b (Rationale Ulcerative colitis begins with inflammation at the rectosigmoid area of the anal canal and progresses proximally. Ulcerative colitis can progress to the entire​ colon, stopping at the ileocecal junction. The duodenum is part of the small intestine and is not affected by ulcerative colitis. The transverse colon is not initially affected by ulcerative colitis.)

The nurse is planning care for a client in the early stages of ulcerative colitis. The nurse understands that which part of the colon is initially affected by ulcerative​ colitis? a Transverse colon b Rectosigmoid area c Ileocecal junction d Duodenum

a (Rationale While all choices are​ problems, which should be addressed in planning care for the client with inflammatory bowel​ disease, impaired fluid balance is priority as this problem may be​ life-threatening if not addressed.)

The nurse is planning care for a client with inflammatory bowel disease. What problem is priority for the nurse to address when caring for this​ client? a Impaired fluid balance b Impaired nutrition c Fatigue d Impaired skin integrity

b,d,e (Rationale Attacks of ulcerative colitis usually last 1 to 3 months. These attacks occur at intervals over months to years. Ulcerative​ colitis, a chronic inflammatory bowel​ disease, affects the mucosa and submucosa of the colon and​ rectum, not the small intestine. A secondary peak of ulcerative colitis occurs between the ages of 60 and 80​ years, not 40 and 50 years. The onset of ulcerative colitis is usually insidious. Most clients with ulcerative colitis have mild to moderate symptoms with six or fewer stools per day.)

The nurse is planning care for a client with ulcerative colitis. Which characteristics associated with ulcerative colitis will the nurse​ recognize? ​(Select all that​ apply.) a A secondary peak of ulcerative colitis occurs between the ages of 40 and 50 years. b Attacks last 1 to 3 months. c It affects the mucosa and submucosa of the small intestine. d Mild to moderate symptoms are present. e The onset is usually insidious.

1 (immed before a paracentesis, the client should empty the bladder to prevent perforation. The client will be placed High Fowlers position or seated on the side of the bed for the procedure. IV sedatives are not usually administered. The client does not need to be NPO)

The nurse is preparing a client for a paracentesis. The nurse should 1. have the client void immediately before the procedure 2. place the client in a side lying position 3. initiate an IV line to administer sedatives 4. place the client on NPO status for 6 hrs before the procedure

4 (Hep B is spread thru exposure to blood or blood products and through high risk sexual activity. Hep B is considered to be a STD. High risk sex activity includes sex with multiple partners, unprotected sex with an infected individual, male homosexual activity, sex with IV drug users, College students are at high risk for development of Hep B, and are encouraged to be immunized. Alcohol intake by itself does not predispose an individual to hep B, but it can lead to high risk behaviors such as unprotected sex. Good personal hygiene alone will not prevent the transmission of Hep B)

The nurse is preparing a community education program about preventing Hep B infection. Which information should be incorporated into the teaching plan? 1 Hep B is relatively uncommon among college students 2. Freq ingestion of alcohol can predispose an individual to development of Hep B 3. Good personal hygiene habits are most effective at preventing the spread of Hep B 4. The use of a condom is advised for sexual intercourse

3 (Clients with cirrhosis should be instructed to avoid constipation and straining at stool to prevent hemorrhage. The client with cirrhosis has bleeding tendencies because of the livers inability to produce clotting factors. A low protein and high carb diet is recommended. Clients with cirrhosis should not take acetaminophen, which is potentially hepatotoxic. Aspirin also should be avoided if esophageal varices are present Cirrhosis is a CHRONIC disease)

The nurse is providing discharge instructions for a client with cirrhosis. Which statement best indicates the client has understood the teaching? 1. I should eat a high-protein high carb diet to provide energy 2. It is safer for me to take acetaminophen for pain instead of aspirin 3. I should avoid constipation to decrease chances of bleeding 4. If I get enough rest and follow my diet it is possible for my cirrhosis to be cured

b,c,d,e (Rationale Blood may or may not be present in the stool of a client with Crohn​ disease, and the nurse would assess for it. The client with Crohn disease will have​ right, not​ left, lower quadrant abdominal pain relieved by defecation. Persistent diarrhea is a symptom of Crohn disease. Malaise may be present in a client with Crohn disease.​ Fissures, ulcers,​ fistulas, and abscesses of the anorectal area may be present in a client with Crohn​ disease, and the nurse would assess the client for them.)

The nurse is questioning a client with Crohn disease about the presence of current symptoms. Which symptoms would the nurse find in the​ client? ​(Select all that​ apply.) a Left lower quadrant abdominal pain relieved by defecation b Blood in the stool c Persistent diarrhea d Malaise ​e Fissures, ulcers,​ fistulas, and abscesses of the anorectal area

B (Crohn's disease is characterized by nonbloody diarrhea of usually not more than four to five stools daily. Over time, the diarrhea episodes increase in frequency, duration and severity. The other options are not associated with diarrhea.)

The nurse is reviewing the record of a client with Crohn's disease. Which of the following stool characteristics would the nurse expect to note documented on the client's record? A Chronic constipation B Diarrhea C Constipation alternating with diarrhea D Stool constantly oozing from the rectum

1 3 4 2 (Silencing the alarm will eliminate stress to the client and allow the nurse to focus The nurse should then assess the access site to note if the needle is inserted in the vein or if there is tissue trauma, infiltration or inflammation. Next the nurse should check for kinks in the tubing Finally the nurse can plug the pump into the wall to allow the battery to recharge)

The nurse is taking care of a client who has an IV infusion pump. The pump alarm rings. What should the nurse do in order from first to last? 1. silence the pump alarm 2. determine if the pump is plugged into the electrical outlet 3. Assess the clients access site for infiltration or inflammation 4. assess the tubing for hindrances to flow of solution

1 (The main transmission route for Hep A is the oral-fecal route. rarely parental. Good handwashing before eating or preparing food is essential to preventing spread of the disease. PErcutaneous transmission is seen with Hep B,C, and D. Alpha interferon is used for treatment of chronic Hep B and C)

The nurse is teaching the client with hepatitis A about preventing transmission of the disease. The nurse should focus teaching on: 1. proper food handling 2. insulin syringe disposal 3 alpha interferon 4. use of condoms

1 (The client should be monitored closely for changes in mental status. Ammonia has a toxic effect on CNS tissue and produces an altered LOC. Marked by drowsiness, and irritability. If this process is unchecked, the client may lapse into a coma. Increasing ammonia levels are not detected by changes in BP urine output, or respirations)

The nurse monitors a client with cirrhosis for the development of hepatic encephalopathy. Which would be an indication that hepatic encephalopathy is developing? 1. decreased mental status 2. elevated BP 3. decreased urine output 4. labored respiration

b (The client with cirrhosis is at risk for​ ascites; therefore it is important to measure the​ client's abdominal girth while providing care. The nurse should encourage small​ meals, provide a diet low in​ sodium, and use warm water for bathing.)

The nurse observes a distinct change in the Mr.​ Dontay's level of consciousness during a routine assessment during a scheduled physical examination. Mr. Dontay is 55 years​ old, a recovering​ alcoholic, and has a primary diagnosis of cirrhosis. Which intervention is appropriate for Mr. Dontay while providing​ care? a Encouraging large meals b Measuring abdominal girth c Using hot water for bathing d Providing a diet high in sodium

4 (Elevate HOB will allow for increasing lung expansion by decreasing the pressure on the diaphragm. The client requires reassessment. A paracentesis is reserved for symptomatic clients with ascites with impaired respiration or abdominal pain not responding to other measures, such as Na restriction and diuretics. There is no indication for blood cultures. Heart sounds are assessed with routine physical assessment)

The nurse reviewing the chart information for a client with increased ascites. The data include the following: T 98.9 (37.2 C) HR 118, shallow respirations @ 26 min, BP 126/76 and SpO2 89% on room air. The nurse should first: 1. assess heart sounds 2. obtain a prescription for blood cultures 3. prepare for a paracentesis 4 raise the HOB

2 (Hep C is usually transmitted thru blood exposure and needlesticks. A hep C vaccine is currently under development, but is not available. The first line of defecnse against hep B is the hep B vaccine. Hep C is not transmitted thru feces or urine. WEaring a gown and mask will not prevent transmission of the hep C virus if the caregiver comes in contact with infected blood or needles)

The nurse should institute which measures to prevent transmission of the Hep C virus to healthcare personnel? 1. administering the Hep C vaccine to all healthcare personnel 2. decreasing contact with blood and blood contaminated fluids 3. wearing gloves when emptying the bed pan 4. wearing a gown and mask when providing direct care

2 (Low fat, high protein, high carb diet is encouraged for a client with Hep to promote liver rejuvenation. Nutrition intake is important because clients may be anorexic and experience weight loss. Activity should be modified and adequate rest maintained to promote recovery. Social isolation should be avoided, and education on preventing transmission should be provided; the client does not need to sleep in a separate room)

The nurse should teach the client with hepatitis A to: 1. limit caloric intake and reduce weight 2. increase carbs and protein in the diet 3. avoid contact with others and sleep in a separate room 4. intensify routine exercise and increase strength

a (Rationale: The client with chronic viral hepatitis resulting in cirrhosis should avoid alcohol and medications like acetaminophen that will further damage the liver. Because the liver becomes fibrotic and there is an extensive loss of liver cells, treatments will not cure or reverse the disease, but they can help to slow the progression to liver failure. Use of medications to regulate protein metabolism is one form of supportive therapy that can help to reduce complications and delay liver failure but these, of course, do not guarantee a long life. )

The nurse would evaluate teaching as effective when a client with chronic viral hepatitis progressing to cirrhosis states which of the following? a "The medications that help to regulate how my body handles protein will help me have a pretty normal lifespan." b "I know I should only use acetaminophen for pain relief." c "I understand that the fibrosis and loss of liver cells can be reversed if I'm really careful with my diet and avoid alcohol and drugs." d "Since this is caused by an infection, not lifestyle, I'm glad to hear that I won't need to reduce my alcohol intake."

1 (sufficient energy is required for healing. Adequate carb intake can spare protein. The client should eat approx. 16 carb kilocalories for each kilogram of ideal body weight daily TPN is not routinely prescribed for the client with hepatitis. The client must lose a large amt of weight and be unable to eat anything for TPN to be ordered.)

The nurse writes the problem "imbalanced nutrition; less than body requirements" for the client dx with hepatitis. Which intervention should the nurse include in the plan of care? 1. Provide a high calorie intake diet 2. Discuss TPN 3. Instruct the client to decrease salt intake 4. Encourage the client to increase water intake

1,2,3 (Hep B can be transmitted by sharing any type of needles, especially those who use drugs. Hep B can be transmitted thru sexual activity, therefore recommend abstinence, mutual monogomy and barrier protection. Three does of Hep B vaccine provide immunity in 90% of healthy adults. Immune globulin shots are administered as postexposure prophylaxis, but encouraging these injections is not a health promotion activity. Hepatoxic meds should be avoided in clients who have Hep or who have had it. )

The public health nurse is discussing hepatitis B with a group in the community. Which health promotion activities should the nurse discuss with the group? SATA 1.. Do not share needles or equipment 2. Use barrier protection during sex 3. Get the Hep B vaccine 4. Obtain immune globulin injections 5. Avoid any type of hepatotoxic medications

1 (The Hepatitis A virus is in the stool of infected people and takes up to two weeks before symptoms develop. Hep B is spread thru contact with body fluids and blood Hep C is transmitted thru infected blood and body fluids Hep D only causes infection in people also infected with Hep B or C)

The public health nurse is teaching day care workers. Which type of hepatitis is transmitted by the oral fecal route via contaminated food, water or direct contact with an infected person? 1. Hep A 2. Hep B 3. Hep C 4. Hep D

4 (Hep A is transmitted by the fecal-oral route. Good handwashing helps to prevent its spread. Singing the happy birthday song takes approx. 30 secs. which is how long an individual should wash their hands Eating after each other should be discouraged, but it is not the most important intervention. Only bottled water should be consumed in 3rd world countries, but not necessary in american schools. Hep B and C are transmitted by sexual activity)

The school nurse is discussing methods to prevent an outbreak of hep A with a group of high school teachers. Which action is the most important to teach the high school teachers? 1. Do not allow students to eat or drink after each other 2. Drink bottled water as much as possible 3. Encourage protected sexual activity 4. Sing the happy birthday song while washing hands

The nurse is caring for a patient who has had surgery for colon cancer and has a permanent colostomy. Which assessment must the nurse report to the provider immediately?

The stoma is dark and bluish in appearance

An 80-year-old patient is admitted to the hospital for diverticulitis. The family states that the family member isn't acting normally. The patient does not have specific complaints. The nurse correlates this data to which characteristics of older adults?

They may exhibit a change in mental status before any other symptoms occur.

1 (Clients with cirrhosis, without complications, a high calorie, high carb diet is preferred to provide adequate supply of nutrients. In the early stages of cirrhosis, there is no need to restrict protein, fat or sodium.)

What diet should be implemented for a client who is in the early stages of cirrhosis? 1. high calorie, high carb 2. high protein, low fat 3. low fat, low protein 4. high carb, low sodium

2 (priority of crohns care in acute exacerbation is to promote bowel rest, this is accomplished by decreasing activity, encouraging rest, and initial NPO status while maintaining nutrition parentally. Diarrhea is nonbloody in crohns and episodes of rectal bleeding are not expected)

What is a priority focus of care for a client experiencing exacerbation of crohns disease? 1. encouraging regular ambulation 2. promoting bowel rest 3. maintaining current weight 4. decreasing episodes of rectal bleeding

a (The purpose of liver functions tests in diagnosing cirrhosis is to determine the degree of elevation of liver enzymes. A CBC is used to determine the presence of anemia. Coagulation studies are used to determine the prothrombin time. Serum glucose and cholesterol levels are used to determine the effect cirrhosis is having on glucose and lipid metabolism)

What is the purpose of liver functions tests in diagnosing​ cirrhosis? a To determine the degree of elevation of liver enzymes b To determine the presence of anemia c To determine the prothrombin time d To determine glucose and lipid metabolism

b,c (Sulfasalazine​ (Azulfidine) blocks the production of prostaglandin to reduce inflammation. The client should use sunscreen when taking​ this, since this medication causes photosensitivity. This medication does not stimulate appetite or reduce the number of stools per day. Clients taking corticosteroids should decrease sodium​ intake, and this medication is an​ antibiotic, which does not require this dietary restriction.)

What is true regarding sulfasalazine​ (Azulfidine) prescribed for clients with inflammatory bowel​ disease? ​(Select all that​ apply.) a The client should reduce intake of sodium when taking this. b It blocks the production of prostaglandin to reduce inflammation. c The client should use sunscreen when taking this. d It reduces the amount of stools per day to maintain fluid balance. e The main side effect is appetite stimulation.

c (The most common age of onset for ulcerative colitis is between 15 and 30​ years, with a secondary peak between the ages of 60 to 80 years. The most common age of onset for ulcerative colitis is not between 5 and​ 15, 25 and​ 55, or 30 and 60 years.)

When does the onset of ulcerative colitis most commonly​ occur? a Between the ages of 5 and 15 years b Between the ages of 30 and 60 years c Between the ages of 15 and 30 years d Between the ages of 25 and 55 years

1 (The prothrombin time may be prolonged because of decreased absorption of Vit K and decreased production of prothrombin by the liver. The client should be assessed carefully for bleeding tendencies. Blood glucose, serum calcium and potassium are not affected by hep)

When planning care for a client with hepatitis A, the nurse should review lab reports for which lab values? 1 prolonged prothrombin time (PT) 2. decreased blood glucose 3. elevated potassium level 4. decreased serum calcium level

a (Rationale: Sulfasalazine makes the client more susceptible to sunburn, so the client is instructed to use sunscreen when outside. The medication should be taken following meals rather than before meals to increase intestinal transit time. It should also be taken with a full glass of water, and daily fluid intake should be increased to a minimum of about 2000 mL (2 quarts) rather than 1.5 mL. There is no known reason to take vitamin C while on this medication, although individuals with IBD are at risk for decreased absorption of vitamins and minerals due to the disease process. )

When teaching a client with inflammatory bowel disease about prescribed oral sulfasalazine, the nurse should instruct the client to: a use sunscreen while taking the medication. b take vitamin C while on this medication. c limit fluids to 1500 mL or less per day. d take the medication on an empty stomach.

3 (The nurse must know when the client had the last alcoholic drink to be able to determine when and if the client will experience delirium tremens the physical withdrawal from alcohol. It really doesnt matter how long the client has been drinking, the dx of alcoholic cirrhosis indicates the client probably has been drinking many years. An advance directive is important for the client that is terminally ill, but is not priority question. And unless the client is very malnourished this is not a typical nursing question)

Which assessment question is priority for the nurse to ask the client diagnosed with end stage liver failure secondary to alcoholic cirrhosis? 1 How many years have you been drinking alcohol? 2 Have you completed an advanced directive? 3. When did you have your last alcoholic drink? 4. What foods did you eat at your last meal?

2

Which diet would be most appropriate for a client with ulcerative colitis? 1. a high calorie, low protein 2. high protein, low residue 3. low fat, high fiber 4. low sodium, high carbohydrate

a (Excessive alcohol use is the leading cause of cirrhosis. Injection drug use increases the risk for contracting bloodborne hepatitis​ (B, C, or​ D), which leads to chronic hepatitis and then cirrhosis. Unprotected sex and hepatitis E infection are not causes of cirrhosis.)

Which factor is the leading cause of​ cirrhosis? a Excessive alcohol use b Injection drug use c Unprotected sex d Hepatitis E

2 (LIver inflammation and obstruction block the normal flow of bile, excess bilirubin turns the skin and sclerae yellow and the urine dark and frothy. Profound anorexia is also common. Tarry stools are indicative of GI bleeding and would not be expected in hepatitis. Light or clay colored stools may occur related to bile duct obstruction. SOB would be unexpected.)

Which finding is normal for a client during the icteric phase of hep A? 1. tarry stools 2. yellowed sclerae 3. sob 4. light, frothy urine

3 (clay colored stools and hemorrhoids are GI effects of liver failure. Hypoalbuminemia (decreased albumin) and muscle wasting are METABOLIC effects not GI effects. Oligomenorrhea is no menses, which is a reproductive effect, and decreased body hair is integumentary effect Dyspnea is a RESP effect, and caput medusae (dilated veins around the umbilicus) is an integumentary effect although it is on the abdomen)

Which gastrointestinal assessment data should the nurse expect to find when assessing the client in end stage liver failure? 1. hypoalbuminemia and muscle wasting 2. oligomenorrhea and decreased body hair 3. clay colored stools and hemorrhoids 4. dyspnea and caput medusae

2

Which goal for the clients care should take priority during the first days of hospitalization for an exacerbation of ulcerative colitis? 1. promoting self care and independence 2. managing diarrhea 3. maintaining adequate nutrition 4. promoting rest and comfort

2 (The client should be able to verbalize the importance of reporting any bleeding tendencies that could be the result of prolonged prothrombin time. Ascites is not typically a clinical manifestation of hep, it is associated with cirrhosis. Alcohol should be eliminated for at least 1 year after the dx of hepatitis to allow the liver time to fully recover. There is no need for the client to be restricted to the home because hepatitis is not spread thru casual contact between individuals)

Which goal is appropriate for a client with Hep A? The client will: 1. demonstrate a decrease in fluid retention related to ascites 2. Verbalize the importance of reporting bleeding gums or bloody stools 3. limit use of alcohol to 2-3 drinks per week 4. restrict activity to within the home to prevent disease transmission

b (Diarrhea is the primary symptom in an exacerbation of ulcerative colitis, and decreasing the frequency of stools is the first goal of treatment. The other goals are ongoing and will be best achieved by halting the exacerbation. The client may receive antidiarrheal medications, antispasmodic agents, bulk hydrophilic agents, or anti-inflammatory drugs)

Which goal of the client's care should take priority during the first days of hospitalization for an exacerbation of ulcerative colitis? A Promoting self-care and independence B Managing diarrhea C Maintaining adequate nutrition D Promoting rest and comfort

2 (General health promotion measures include maintaining good nutrition, avoiding infection, and abstaining from alcohol. It is not necessary to take multivitamins if the client is obtaining adequate nutrition. Rest and sleep are essential, but an impaired liver may not be able to detoxify sedatives and barbiturates. Such drugs must be used cautiously if at all, by clients with cirrhosis. The client does not need to limit contact with others but should exercise caution to stay away from ill people)

Which health promotion activity should the nurse suggest that the client with cirrhosis add to the daily routine at home? 1. supplement diet with multivitamins 2. abstain from drinking alcohol 3. take a sleeping pill at bedtime 4. limit contact with other people whenever possible

2 (Adequate rest ins needed for maintaining optimal immune function The client must avoid all alcohol. Clients are usually anorexic and nauseated in the afternoon and evening, therefore the main meal should be in the morning. Diet drinks and juices provide few calories, and the client needs an increased calorie diet for healing)

Which instruction should the nurse discuss with the client who is in the icteric phase of hep C? 1 decrease alcohol intake 2. Encourage rest periods 3. Eat a large evening meal 4. Drink diet drinks and juices

1 (The increased serum ammonia level associated with liver failure causes the hepatic encephalopathy, which in turn leads to neurological deficit. Administering a loop diuretic is appropriate for ascites and portal hypertension Checking the stool for bleeding is an appropriate intervention for esophageal varices and decreased Vit K Assessing the abdominal fluid wave ius an appropriate intervention for ascites and portal hypertension)

Which intervention should the nurse implement specifically for the client in end stage liver failure who is experiencing hepatic encephalopathy? 1. Assess the clients neurological status 2. Prepare to administer a loop diuretic 3. Check the clients stool for blood 4. Assess for an abdominal fluid wave

b (Rationale Liver biopsy helps distinguish cirrhosis from other forms of liver disease. The O2 ​level, CO2 ​level, and WBC count are not relevant to establishing the diagnosis of cirrhosis.)

Which laboratory test is prescribed for a client with suspected​ cirrhosis? a O2 level b Liver biopsy c CO2 level d WBC count

d (Appropriate nursing diagnoses for a client with cirrhosis include impaired skin​ integrity, diminished protection and impaired nutrition.​ Increased, not​ decreased, fluid volume is appropriate for a client with cirrhosis.)

Which nursing diagnosis is not appropriate for a client with​ cirrhosis? a Diminished protection b Impaired skin integrity c Impaired nutrition d Decreased fluid volume

a (Rationale Clients with cirrhosis deal with a variety of​ problems, but​ fatigue, activity​ intolerance, and anxiety are not among them. A few nursing diagnoses that are appropriate include impaired skin​ integrity, increased risk for acute​ confusion, diminished​ protection, increased fluid​ volume, and reduced​ nutrition, less than body requirements.)

Which nursing diagnosis supports a medical diagnosis of​ cirrhosis? a Increased risk for acute confusion b Fatigue c Activity intolerance d Anxiety

c (Infliximab​ (Remicade), an immune response​ modifier, suppresses tumor necrosis factor​ (TNF) to reduce inflammation in the treatment of inflammatory bowel disease. Olsalazine​ (Dipentum) and metronidazole​ (Flagyl) are antibiotics which are used to treat inflammatory bowel disease and do not suppress tumor necrosis factor​ (TNF). Methylprednisolone​ (Solu-Medrol) is a​ corticosteroid, which is used to treat inflammatory bowel disease and does not suppress tumor necrosis factor​ (TNF).)

Which pharmacologic therapy used in the treatment of inflammatory bowel diseases suppresses tumor necrosis factor​ (TNF) to reduce​ inflammation? a Methylprednisolone​ (Solu-Medrol) b Olsalazine​ (Dipentum) c Infliximab​ (Remicade) d Metronidazole​ (Flagyl)

1 (Ascites can compromise the action of the diaphragm and increase the clients risk of respiratory problems. Ascites also greatly increases the risk of skin breakdown. Freq position changes are important but the preferred position is Fowlers. )

Which position would be appropriate for a client with severe ascites? 1. Fowlers 2. Side lying 3. reverse Trendelenburg 4. Sims

4 (contact precautions are recommended for clients with Hep A. This includes wearing gloves for direct care. A gown is not required unless substantial contact with the client is anticipated. It is not necessary to wear a mask. The client does not need a private room unless incont of stool)

Which precautions should the healthcare team observe when caring for clients with Hep A? 1. gowning when entering the clients room 2. wearing a mask when providing care 3. assigning the client to a private room 4. wearing gloves when giving direct care

2 (The Hepatitis B vaccine will prevent the client from contracting the disease. 1. This intervention appropriate for preventing Hep A 3. This appropriate for preventing Hep A 4. The nurse uses standard precautions, not the client)

Which priority teaching information should the nurse discuss with the client to help prevent contracting hepatitis B? 1. Explain the importance of good handwashing 2. Recommend the client take the Hep B vaccine 3. Tell the client to not ingest unsanitary food or water 4. Discuss the importance of standard precautions

4 (the client needs to understand some types of cough syrup have alcohol and all alcohol must be avoided to prevent further injury to the liver. therefore this requires intervention)

Which statement by the client diagnosed with hepatitis warrants immediate intervention by the clinic nurse? 1. I will not drink any type of beer or mixed drink 2. I will get adequate rest so I dont get exhausted 3. I had a big hearty breakfast this morning 4. I took some cough syrup for a nasty head cold

3

Which task is most appropriate to delegate to the UAP? 1. Draw the serum liver function test 2. Evaluate the clients intake and output 3. Perform the bedside glucometer check 4. Help the ward clerk transcribe orders

a (Nutritional support for cirrhosis includes restricting sodium intake to 2 g per day. Administering vitamin K and recommending antacids is pharmacologic therapy. Decreasing fluid​ intake, not increasing​ it, is considered a nutritional therapy for cirrhosis. )

Which therapy for cirrhosis is considered nutritional​ therapy? a Restricting sodium intake b Recommending antacids c Administering vitamin K d Increasing fluid intake

2 (standard precautions apply to blood, body fluids, secretions, excretions except sweat, regardless of whether they contain visible blood)

Which type of precautions should the nurse implement to protect from being exposed to any of the hepatitis viruses? 1 Airborne 2. Standard 3. Droplet 4. Exposure

3 4 1 2 6 5 (Immediate decontamination is appropriate, because time can affect viral load. The occupational health nurse will direct the UAP in filing the correct forms, getting the appropriate laboratory tests, obtaining appropriate prophylaxis, and following up on results. Focus: Prioritization, supervision)

While transferring a dirty laundry bag, a UAP sustains a puncture wound to the finger from a contaminated needle. The unit has several clients with hepatitis and acquired immunodeficiency syndrome (AIDS); the needle source is unknown. Place in order of priority the instructions that should be given to the UAP. 1. Have blood test(s) performed per protocol. 2. Complete and file an incident report. 3. Perform a thorough aseptic hand washing. 4. Report to the occupational health nurse. 5. Follow up for results and counseling. 6. Begin prophylactic drug therapy. _____, _____, _____, _____, _____, _____

A nurse is teaching a client who has a duodenal ulcer and a new Rx for esomprazole (Nexium). Which of the following should be included in the teaching? (select all) a. take 1 hour before a meal b. limit NSAIDs c. expect skin flushing d. increase fiber intake e. chew the medication before swallowing

a. 1 hour before meals b. limit NSAIDs e. swallow the capsule whole, do not crush or chew

A client with a duodenal ulcer asks the nurse why antibiotics are a part of the treatment plan. Which information should the nurse include in the explanation? a. Most duodenal ulcers are caused by Helicobacter pylori. b. Antibiotics are used to sterilize the stomach. c. Many people have Clostridium difficile, which can lead to ulcer formation. d. Antibiotics decrease the likelihood of infection.

a. H. pylori

The physician has ordered omeprazole 20 mg twice daily, clarithromycin 500 mg twice daily, and amoxicillin 1 gram daily for a client with peptic ulcer disease (PUD). It is most important for the nurse to instruct the client to: a. Stop the drugs and notify the physician if a rash, hives, or itching develops. b. Take the drugs on an empty stomach, 1 hour before breakfast and at least 2 hours after dinner. c. Take the drugs with a full glass of water. d.Consume 8 oz of yogurt or buttermilk daily while on the medication.

a. anaphylaxis Rash, hives, and itching indicate a hypersensitivity reaction and need to be reported immediately. Consuming yogurt and taking the drugs on an empty stomach with a full glass of water are also taught to the client but are not as important as a hypersensitive reaction, which can lead to anaphylaxis and death.

For which of the following clients with PUD is misoprostol (Cytotec) contraindicated? a. 27 y/o who is preggos b. 75 y/o who has osteoarthritis c. 37 y/o who has a kidney stone d. 46 y/o who has a UTI

a. misoprostol can induce labor

A nurse is in the ED is completing an assessment of a client who has suspected stomach perforation due to a peptic ulcer. Which of the following are expected findings? select all a. rigid abdomen b. tachycardia c. elevated BP d. circumoral cyanosis e. rebound tenderness

a. ridgid abdomen b. tachycardia e. rebound tenderness c. HYPOTENSION

The nurse assesses for which clinical manifestations in the patient diagnosed with liver cancer? (Select all that apply.)

anorexia - fatigue - jaundice

a nurse is caring for a client who has GERD and a new prescription for metoclopramide. the nurse should plan to monitor for which of the following adverse effects?

ataxia the nurse should plan to monitor the client for extrapyramdial symtoms, such as ataxia, and should report any positive findings to the provider.

A nurse is providing instructions to a client who has a Rx for metronidazole (Flagyl) to treat peptic ulcer. The client asks the nurse why this medication has been Rx. Which of the following responses is correct? a. to get rid of the infection from giardiasis b. to get rid of the infection from H. pylori c. to increase the pH of gastric juices in the stomach d. to decrease the pH of gastric juices in the stomach.

b. H. pylori (gram - organism) that resides in the stomach and duodenum. This drug greatly reduces the recurrence of PUD.

A client with peptic ulcer disease (PUD) is scheduled for a vagotomy and asks the nurse why the procedure is being performed. The nurse tells the client: a. "A vagotomy causes the stomach to heal." b. "A vagotomy cuts the nerve that stimulates acid secretion." c. "A vagotomy decreases digestion of food." d. "A vagotomy decreases the body's reaction to stress."

b. vagus nerve stimulates acid recreation The vagus nerve stimulates the secretion of hydrochloric acid. Cutting the nerves decreases the production of acid in the stomach. The vagus nerve is not responsible for the reaction to stress, healing of the stomach, or the digestion of food.

a nurse is providing discharge teaching for a client who has chronic cholecystitis. which of the following food selections by the client indicates the teaching was effective?

bananas this food selection by the client indicates the teaching was effective. low-fat options, such as bananas, are recommended due to the decreased risk for causing manifestations of cholecystitis

what are the 4 manifestations of hemorrhaging

blood in stool or vomit fatigue, weakness, dizziness orthostatic hypotension hypovolemic shock

a nurse is assessing a client who has cirrhosis. which of the following findings is the priority for the nurse to report to the provider?

bloody stools the greatest risk to the client is hemorrhaging. bloody stools are indication of bleeding in the gastrointestinal tract. this finding is the priority to the report to the provider.

a nurse is assessing a client who has peritonitis. which of the following findings should the nurse expect?

board-like abdomen a board-like, distended abdomen is an expected finding in this client

A client with peptic ulcer disease (PUD) suddenly complains of severe abdominal pain. The nurse should: (Select all that apply.) a. Obtain an order for narcotics. b, Administer the prescribed proton-pump inhibitor. c. Withhold oral food and fluids. d. Place the client in Fowler's position. e. Notify the physician.

c, d and e

A nurse is completing an assessment of a client who has a gastric ulcer. Which of the following are expected findings? (select all) a. reports pain relived by eating b. reports pain occurs often at night c. reports a sensation of bloating d. reports pain occurs 1/2 to 1 hour after a meal e. reports pain upon palpation of the epigastric region

c. bloating d. pain 1/2 to 1 hour after a meal e. pain in epigastric region p palpation

A nurse is completing d/c teaching for a client who has an infection due to Helicobacter pylori. Which of the following statements by the client indicates understanding of the teaching? a. I will continue my Rx for corticosteroids b. I will schedule a CT scan to monitor improvement c. I will take a combination of medications for treatment d. I will have my throat swabbed to recheck for this bacteria

c. combo of meds antibiotics and a histamine receptor antagonist a. steroid use is a contributing factor to an infection caused by H. pylori. b. EGD not CT d. gastric samples not a throat swab

A client is being evaluated for a possible duodenal ulcer. The nurse monitors the client for which of the following manifestations that would indicate this diagnosis? a. Distended abdomen b. Positive fluid wave c. Epigastric pain relieved by food d. History of chronic aspirin use

c. pain/food The pain of a duodenal ulcer is relieved by eating because the pancreatic juices stimulated by eating are high in bicarbonate. Aspirin use is irritating to the stomach. Distended abdomen and positive fluid wave are unrelated to ulcers.

A client with peptic ulcer disease (PUD) is scheduled for a pyloroplasty and asks the nurse about the procedure. The nurse tells the client which of the following? a. The vagus nerve is cut to reduce irritation. b. The distal portion of the stomach is removed. c. The pylorus is cut to relax the muscle and enlarge the opening. d. The ulcer and tissue producing hydrochloric acid are removed.

c. pylorus is cut Cutting the muscle and enlarging the opening is a pyloroplasty. When the vagus nerve is cut, it is a vagotomy. Removing a portion of the stomach is a subtotal gastrectomy. Cutting the ulcer and surrounding tissue is a Billroth II procedure.

The nurse is caring for a client who was admitted to the hospital with peptic ulcer disease (PUD). Which of the following, if noted by the nurse, is a manifestation of complications from PUD? a. Numbness in the legs b. Bradycardia c. A rigid, board-like abdomen d. Nausea and vomiting

c. rigid, board-like abdomen A risk of peptic ulcer disease is perforation, a surgical emergency. The nurse assesses the client with PUD for this possibility. The client will also have sudden sharp pain, possible nausea and vomiting, and tachycardia. Numbness in the legs is unrelated to perforation.

A client taking sucralfate (Carafate) PO for PUD has been started on phenytoin (Dilantin) to control seizures. Which of the following should be included in the client's teaching? a. take both of these medications at the same time b. take sucralfate with a class of milk c. allow a 2 hour interval between these medications d. chew the sucralfate thoroughly before swallowing

c. sucralfate can interfere with the absorption of phenytoin

The nurse identifies which of the following nursing diagnoses as highest priority for the client with peptic ulcer disease (PUD)? a. Acute Pain b. Ineffective Health Maintenance c. Impaired Tissue Integrity: Gastrointestinal d. Nausea

c. tissue integrity PUD is an interruption of the integrity of the gastric lining. While the other nursing diagnoses will be present and need to be addressed at some point, disruption of the integrity of the lining of the stomach is the highest because of the risk for perforation.

The nurse correlates which clinical manifestation as a normal age-related variant in an older adult?

constipation

The nurse monitors for which complication in the patient who has undergone a barium enema?

constipation

A nurse is providing teaching for a client who has a new dx of dumping syndrome following gastric surgery. Which of the following should be included in the teaching? a. eat 3 moderate-sized meals a day b. drink at least one glass of water with each meal c. eat a bedtime snack that contains a milk product d. increase protein in the diet

d. high protein, high fat, low fiber, and moderate to low carbohydrate diet a. small, frequent meals b. eliminate liquids with meals (as well as 1 hr before and 1 hr after) c. avoid milk products

A nurse is providing a client who has peptic ulcer disease with instructions about managing his condition. Which of the following instructions should the nurse include? (select all) a. eat six small meals a day b. drink milk to aid in healing your ulcer c. low-dose ASA should be avoided d. seek measures to reduce stress e. avoid smoking

d. reduce stress e. avoid smoking a. eating 6 meals a day may stimulate production of gastric acid b. avoid milk/cream it stimulates production of gastric acid c. low-dose ASA therapy is permitted

a nurse is providing discharge teaching for a client following an ileostomy. the nurse should instruct the client to report which of the following findings to the provider?

dark purple stoma

Patients experiencing diarrhea are at risk for which alteration in absorption?

decreased potassium absorption

A nurse is assessing a client immediately following a paracentesis for the treatment of ascites. which of the following findings indicates the procedure was effective?

decreased shortnesss of breath increased abdominal fluid can limit the expansion of the diaphragm and prevent the client from taking a deep breath. once excess peritoneal fluid is removed, the diaphragm will expand more freely. the nurse should identify this finding as an indicator of the effectiveness of the paracentesis.

A 17-year-old patient sustained blunt abdominal trauma (was hit in the abdomen with a baseball bat) and is being discharged home after 24 hours of observation. Discharge planning includes which teaching?

education about abdominal signs and symptoms of increased pain, tenderness

a nurse is providing teaching about dietary management to prevent dumping syndrome for a client who is postoperative following a gastrectomy. the nurse should encourage the client to include which of the following food in his diet?

eggs

The nurse recognizes which gastric disorder as a complication of inadequate mucosal perfusion secondary to intense physiological stress?

erosive gastritis

a nurse is providing teaching for a client who has cirrhosis and a new prescription for lactulose. the nurse should include which of the following instructions in the teaching?

expect 2-3 soft stools per day.

Other than H. pylori infections and NSAID use, what are 3 risk factors for peptic ulcer disease?

family history smoking advanced age

a nurse is providing discharge teaching for a client who has a new prescription for medication to treat peptic ulcer disease. the nurse should identify that which of the following medications inhibits gastric acid secretion?

famotidine

a nurse is assessing a client who has Crohn's disease. which of the following findings should the nurse except?

fatty, diarrheal stools

What is the classic symptom of peptic ulcer disease?

gnawing, burning, aching, hunger like epigastric pain that can radiate to the back that is relieved by eating

A 67-year-old male is suspected of having a peptic ulcer. The nurse monitors for a decrease in which diagnostic value with GI hemorrhage in this patient?

hematocrit

a nurse is reviewing the laboratory values of a client who has colorectal cancer. which of the following findings should the nurse expect?

hemoglobin 9.1 g/dl decreased hemoglobin is an expected finding in a client who has colorectal cancer because of occult intestinal bleeding.

What are the 3 complications of PUD?

hemorrhage obstruction perforation

a nurse is assessing a client who is postoperative following a gastrectomy. the nurse should identify which of the following findings as an indication of acute gastric dilation?

hiccups

a nurse is providing discharge teaching for a client who has chronic hepatitis C. which of the following statements by the client indicates an understanding of the teaching?

i will avoid medications containing acetaminophen

a nurse is admitting a client who has acute pancreatitis. which of the following actions should the nurse take first?

identify the client's level of pain

The nurse implements which interventions to decrease the risk of dumping syndrome in the patient after esophageal resection for cancer?

increase protein in the diet

a nurse is reviewing the laboratory results of a client who has acute pancreatitis. which of the following findings should the nurse expect?

increased serum amylase serum amylase levels are increased in a client who has acute pancreatitis because of the pancreatic cell injury.

The nurse provides education to a patient who has a hiatal hernia and experiences GERD after eating. Which activity should the nurse instruct this patient to avoid?

laying flat after meals

The nurse has requested a dietary consult for a patient with GERD. What statements provide useful dietary information for this patient to manage the GERD symptoms? (Select all that apply.)

maintain an ideal body weight - avoid spicy foods - avoid fatty foods

The nurse includes which information in the discharge teaching plan for the patient who sustained esophageal trauma?

notify the provider if increased drainage is noted from the injured area

The nurse recognizes which as a risk factor for cholecystitis? (Select all that apply.)

obesity - female - caucasian

a nurse is assessing a client who has appendicitis. which of the following findings should the nurse except?

oral temp 101.1 F nausea and vomiting right lower quadrant pain

pH: 7.28, PCO2: 39, HCO3: 18

pH: ↓, PCO2: N, HCO3: ↓ Uncompensated Metabolic Acidosis

pH: 7.23, PCO2: 50, HCO3: 28

pH: ↓, PCO2: ↑, HCO3: ↑ Partially compensated Respiratory Acidosis

The nurse receives a patient from the operating room after he undergoes a Whipple's procedure. The nurse understands that the patient has which disorder?

pancreatic cancer

The nurse correlates a disorder of which type of cell in the patient with vitamin B12 deficiency?

parietal cells

The nurse assesses for which finding in a patient with a positive Cullen's sign?

periumbilical bruising

The nurse recognizes which risk factors for the development of stomatitis? (Select all that apply.)

radiation therapy - poor dental hygiene - smoking

Elevated ammonia levels can lead to hepatic encephalopathy. Which provider order best reduces this risk in patients with cirrhosis?

restrict protein intake

What are the 4 manifestations of an obstruction

sensation of epigastric fullness nausea/vomiting electrolyte imbalances metabolic alkalosis

The nurse correlates an increase in which laboratory value to the diagnosis of acute pancreatitis?

serum amylase

The most specific laboratory result in the patient with acute pancreatitis is an elevation in which laboratory value?

serum lipase

What are the 7 manifestations of a perforation

severe upper abdominal pain radiating into the shoulder RIGID BOARDLIKE ABDOMEN absence of bowel sounds diaphoresis tachycardia rapid shallow respirations fever

In assessing the patient with an ileostomy, the nurse focuses on which "three Ss"? (Select all that apply.)

skin - stoma - stool

What is a leading cause of pancreatic cancer?

smoking

The nurse places the patient in which position for a gastrointestinal assessment to promote relaxation of the abdominal muscles?

supine with arms at the side and knees flexed

a nurse is caring for a client who has colorectal cancer and is receiving chemotherapy. the client asks the nurse why blood is being drawn for a CEA level. which of the following responses by the nurse is appropriate?

the CEA determines the efficacy of your chemotherapy. the provider uses the CEA level to determine the efficacy of the chemotherapy. the client's CEA levels will decrease will decrease if the chemotherapy is effective

a nurse is caring for a client who has a duodenal ulcer. which of the following findings should the nurse expect?

the client states the pain occurs 1.5 to 3 hours after meals and during the night

a nurse is teaching a client how to prepare for a colonoscopy. which of the following instructions should the nurse including in the teaching?

the nurse should instruct the client to drink clear liquids for 24 hr prior to colonoscopy to promote adequate bowel cleansing. maintaining NPO status for 4 to 6 hr prior to colonoscopy preserves the bowel's cleansed state

A nurse working in an endoscopy clinic is screening patients for the risk of developing Barrett's esophagus. The nurse should consider which patient at greatest risk?

the patient who has had untreated GERD for 30 years

The nurse recognizes that an impairment of which CN makes a patient at risk for aspiration due to ineffective swallowing?

trigeminal

What are the 3 diagnostic tests used to diagnose PUD?

upper GI series gastroscopy H. pylori testing (serum, feces, urea breath test)

a nurse is caring for a client who has ulcerative colitis. the client has had several exacerbations over the past 3 years. which of the following instructions should the nurse include in the plan of care to minimize the risk of further exacerbations?

use progressive relaxation techniques. progressive relaxation techniques, a form of biofeedback, are recommended to help the client minimize stress, which can precipitate an exacerbation. arrange activities to allow for daily rest periods is correct. daily rest periods decrease stress and reduce intestinal motility. restrict intake of carbonate beverages. the client should avoid gastrointestinal stimulants, such as carbonated beverages, nuts, pepper, and smoking.

In a patient with cirrhosis, the nursing diagnosis "risk for injury and bleeding related to prolonged clotting factors" is most appropriate related to which disorder?

vitamin K deficiency


Kaugnay na mga set ng pag-aaral

Advanced A&P chapter 28 practice questions

View Set

3-MC review for ap world test #1

View Set

Spanish 2 Lesson 3A - Fin de semana-preterito yo form

View Set

The Hiding Place - Chapter 10-12 Review

View Set